Você está na página 1de 92

60 Days Orthobullets:

Answers Bank
Day 4

1.

PREFERRED RESPONSE 4
Patients with multiple injuries including a pelvic ring fracture who present with hemodynamic instability should have a pelvic
binder or circumferential pelvic sheet placed as part of their initial resuscitation.
A systematic approach to search for sources of bleeding and control ongoing hemorrhage is necessary for patients who present
with hemodynamic changes in the setting of a pelvic ring fracture. Management of continued hypotension after pelvic binder
placement is controversial and varies among trauma centers.
Krieg et al. prospectively evaluated 16 patients with unstable pelvic ring injuries initially managed with a novel circumferential
compression device. The authors found substantial reduction in pelvic width with the use of this compressive device in patients
with volume expanding pelvic ring fractures.
Croce et al. retrospectively compared patients with unstable pelvic ring injuries who were treated with either emergent pelvic
fixation (EPF) or a pelvic orthotic device (POD). The authors found that those patients treated with POD had decreased
transfusion requirements and shorter length of hospital stay.
Routt et al describe their technique for circumferential pelvic antishock sheeting (CPAS). The authors provide an illustrative case
and discuss the potential advantages of sheet application versus other techniques of pelvic stabilization.
Incorrect Answers:
Answer 1: External fixation of pelvic ring fractures can be used to assist with resuscitation but pelvic binder application should be
attempted first
Answer 2: The use of pelvic angiography is controversial and institution specific however some centers utilize pelvic angiography
as part of the algorithm for management of ongoing hemorrhage.
Answer 3: Pelvic packing is utilized in some centers to control ongoing pelvic hemorrhage however it is not used as initial
management of patients with hemodynamic instability
Answer 5: Percutaneous iliosacral screws can also be utilized as a form of resuscitation however they should not be used as as first
line of management

2.

PREFERRED RESPONSE 1
The clinical presentation is consistent for a mildly displaced parasymphyseal fracture in a pediatric patient with an open triradiate
cartilage. Weight bearing as tolerated is the most appropriate treatment.
In skeletally immature pelvic ring fractures, the majority of cases can be treated nonoperatively. Open reduction and internal
fixation is required for acetabular fractures with >2 mm of fracture displacement and for any intra-articular or triradiate cartilage
fracture displacement >2 mm. External fixation is necessary for pelvic ring displacement of >2 cm to prevent limb-length
discrepancies.
Holden et al. emphasize that children with open triradiate cartilage have different fracture patterns than do children whose
triradiate cartilage has closed. They report because of the immaturity of the pelvis, the iliac wing is weaker than the elastic pelvic
ligaments, resulting in bone failure before pelvic ring disruption has a chance to occur. For this reason fractures usually involve
the pubic rami and iliac wings and rarely require surgical treatment.
Spiguel et al. reviewed 2850 pediatric trauma admissions at their institution and reviewed cases with a pelvic ring fracture. They

found that although pelvic fractures are an uncommon injury in pediatric trauma patients, the morbidity associated with these
injuries is significant. They report while the majority of pelvic fractures in children are treated nonoperatively, more than one-half
of these patients have concomitant injuries requiring operative management.
Figure A shows an inferior rami fracture in a pediatric patient with an open triradiate cartilage.
Incorrect Answers:
Answers 1,3,4,5: These treatment options are not appropriate in a stable pelvic ring fracture in a child with open triradiate
cartilage.
3.

PREFERRED RESPONSE 5
The patient has an Anterior-Posterior Compression type 3 pelvic ring injury (APC3), and this injury places the patient at risk of
life- threatening hemorrhage. The most appropriate next step in the trauma bay is to place the patient in a pelvic binder in order to
minimize pelvic volume and impart stability to the injured hemipelvis to allow for clot formation.
Pelvic fractures are high energy injuries with a high association of concomitant musculoskeletal trauma and damage to multiple
organ systems. It is important that any patient with a high-energy pelvic ring injury undergo a complete work-up including a CT of
the chest abdomen and pelvis to look for alternative sources of bleeding. Application of a pelvic binder should occur once a pelvic
ring injury is identified as part of the ongoing resuscitation of the patient.
Karadimas et al. retrospectively reviewed 34 patients at a single center who underwent pelvic arterial embolization as part of their
resuscitation. APC injuries had the highest mean transfusion rate during the initial 24 hours, and the overall mortality for pelvic
fractures requiring embolization was 23.5% in this series.
Manson et al. conducted a retrospective case-controlled study, evaluating mortality factors on LC-1 fractures. They found that in
LC-1 fractures, the sacral fracture pattern does not predict mortality; however, mortality rate was increased in patients with a brain
injury, chest injury, or abdominal injury.
Figure A demonstrates an APC3 pelvic ring injury with widening of both the symphysis and the right SI joint. Illustration A
demonstrates the same injury as seen in Figure A after application of a pelvic binder with improved alignment of the pelvic ring.
Illustration B shows appropriate application of a pelvic binder in a multiply injured patient.
Incorrect Answers:
Answer 1: While this patient may need to go emergently to the OR for multiple reasons, the work-up needs to be completed.
However, the patients pelvis should be stabilized with a pelvic binder in the interim.
Answer 2: These images should be obtained, but the pelvis should be closed with a pelvic binder first.
Answer 3: While the pelvis may not be the only location of bleeding, the patient has a known source for bleeding, and it can be
quickly stabilized with a pelvic binder. After the pelvic binder is placed, continued resuscitation and investigation of other possible
locations of bleeding should occur.
Answer 4: While this patient may benefit from embolization, the first step is to close down the pelvis. Closing down the pelvis
may prevent the need for embolization.

4.

PREFERRED RESPONSE 2
Alternating single-leg-stance radiographs are used for the diagnosis of chronic or subtle pelvic instability.
Pelvic instability is a rare etiology of lumbar and low-back discomfort; patients report subjective instability and mechanical
symptoms. Static radiographs (AP pelvis, inlet pelvis, outlet pelvis) are often not adequate for diagnosis of this condition.
Garras et al. performed a study of healthy volunteers and reported on the normal range of physiologic motion with single leg
stance radiographs. They found that multiparous women exhibited the most symphyseal motion with alternating single leg stance
weightbearing AP pelvic radiographs, and up to 5mm of symphyseal translation was seen in healthy, asymptomatic patients.
Siegel et al. reviewed 38 patients with pelvic instability and pain. They found that single leg stance radiographs were more
indicative of instability than standard AP pelvis and inlet/outlet radiographs. They found that up to 5 cm of sympyhseal translation
can be present with these injuries.
Illustration A shows a single leg stance (left leg) AP pelvis radiograph with cephalad displacement of the left hemipelvis.
Illustration B shows a single leg stance (right leg) AP pelvis radiograph, with cephalad displacement of the right hemipelvis.
Incorrect Answers:
Answer 1,3,4,5: Standing alternating single-leg-stance radiographs are not used for diagnosis or evaluation of these disorders.

5.

PREFERRED RESPONSE 5
The lateral sacral view is used to place percutaneous iliosacral screws. Sacral alar morphology has been shown to be variable from
patient to patient. Therefore, intraoperative fluoroscopy is recommended. During placement of the screws, the L5 nerve root is at
risk.
Routt et al (1997) examined the sacral slope and sacral alar anatomy in cadavers and a series of patients. They determined that the
pelvic outlet and lateral sacral plain films provide the best plain radiographic view of the sacral ala. They recommended routine
usage of these views intraoperatively to guide screw placement.
Routt et al (2000) reported on the early complications of percutaneous placement of iliosacral screws for treatment of posterior

pelvic ring disruptions. While technically challenging, this technique leads to less blood loss and lower rates of infection
compared to traditional open techniques.
Barei et al described methods of anterior and posterior pelvic ring disruptions. They determined that successful placement depends
on accurate closed reduction, excellent intraoperative fluoroscopic imaging, and detailed preoperative planning. Early treatment
decreased hemorrhage, provides patient comfort, and allows early mobilization.
6.

PREFERRED RESPONSE 3
Figure A shows an unstable bilateral pelvic ring injury. Percutaneous posterior iliosacral screw fixation places the L5 nerve root at
risk as it courses across the sacral ala. Injury to the L5 nerve root would typically result in weakness in great toe extension and
sensory changes on the dorsum of the foot. It is important to notice that L5 often partially innervates tibialis anterior along with
L4, so weakness to ankle dorsiflexion may be present as well. Illustration A shows the post-operative films with bilateral iliosacral
screws.
Routt et al examined the sacral slope and sacral alar anatomy in cadavers and a series of consecutive patients. They determined
that the pelvic outlet and lateral sacral plain films provide the best plain radiographic views of the sacral ala. They recommended
routine usage of these views intraoperatively to guide screw placement.
In another study, Routt et al reported on the early complications of percutaneous placement of iliosacral screws for treatment of
posterior pelvic ring disruptions. While technically challenging, this technique leads to less blood loss and lower rates of infection
compared to traditional open techniques.
Illustration B displays the root diagrams for sensation, reflex, and motor of the L4-S1 nerves.
Incorrect answers:
1: Weakness to knee extension would be caused primarily by an injury to the L4 nerve root.
2: Decreased patellar reflex would be caused primarily by an injury to the L4 nerve root.
4: Weakness in ankle plantar flexion would be caused primarily by an injury to the S1 nerve root.
5: Decreased Achilles reflex would be caused primarily by an injury to the S1 nerve root.
Illustrations: A

7.

PREFERRED RESPONSE 5
The outlet view best guides superior-inferior screw orientation during percutaneous S1 screw placement. This is due to the relative
forward flexion of the sacrum and pelvis due to pelvic incidence. A lateral sacral view and an inlet pelvis view would best guide
anterior-posterior screw orientation.

Routt et al did a review of percutaneous techniques of pelvic surgery. Although anterior pelvic external fixation remains the most
common form of percutaneous pelvic fixation, iliosacral screws have the advantage of stabilizing pelvic disruptions directly while
diminishing operative blood loss and operative time. They stress importance of a thorough knowledge of pelvic osseous anatomy,
injury patterns, deformities, and their fluoroscopic correlations for surgical success.
Routt et al also looked at the complications that can result from percutaneous iliosacral screw placement. Complications ranged
from inability for adequate imaging due to patient obesity, L5 nerve root injuries, fixation failure, and sacral nonunions. They
support quality triplanar fluoroscopic imaging during iliosacral screw insertions to help accurately reduce injured posterior pelvic
rings.
Illustration A is an example of an outlet view image status post anterior pelvic ring plating and percutaneous iliosacral screw. This
outlet view allows superior S1 neural foramen visualization to help guide screw placement and avoid nerve injury.

8.

PREFERRED RESPONSE 4
APC II injuries are unstable injuries and occur as a result of high-energy trauma. Anatomic structures which are injured or torn
include the pubic symphysis, anterior iliosacral ligaments, and the sacrotuberous ligaments. The posterior sacroiliac ligaments are
spared in APC-II injuries, and differentiate an APC-II injury from an APC-III injury, in which the posterior ligaments are also
torn.
Burgess et al review the classifications of pelvic ring disruptions and their association with mortality. They concluded that APC
injuries required more blood replacement and were related to death more often than lateral compression, vertical shear, or
combined mechanism pelvic injuries.
Tile studied the anatomy of anterior to posterior pelvic ring injuries. Although the anterior structures, the symphysis pubis and the
pubic rami, contribute to 40% to the stiffness of the pelvis, clinical and biomechanical studies have shown that the posterior
sacroiliac complex is more important to pelvic-ring stability. The posterior sacroiliac ligamentous complex is more important to
pelvic-ring stability than the anterior structures and therefore, the classification of pelvic fractures is based on the stability of the
posterior lesion.

9.

PREFERRED RESPONSE 1
Lateral compression type II fractures (as described by the Young-Burgess Classification System) are associated with a crescent
fracture of the iliac wing located on the side of impact. A representative CT scan image and illustration of this injury are shown in
Illustrations A and B respectively. A table describing each pelvic injury and their associated complications is shown in Illustration
C. Illustration D shows each Young-Burgess pelvic injury type.
Burgess et al discuss the effectiveness of a treatment protocol as determined by their pelvic injury classification and hemodynamic
status. The injury classification system was based on lateral compression, anteroposterior compression, vertical shear, and

combined mechanical injury types. They found that their classification-based treatment protocols reduce the morbidity and
mortality related to pelvic ring disruption.
Tile discusses acute pelvic trauma and his classification system for pelvic injuries (ie. Types A, B, and C). He states that any
classification system must be seen only as a general guide to treatment, and that the management of each patient requires careful,
individualized decision making.
Incorrect Answers:
Answer 2: This describes an APC-II injury
Answer 3: This describes an APC-III injury
Answer 4: This describes and LC-I injury
Answer 5: This describes an LC-III injury (ie. "wind-swept pelvis")
10. PREFERRED RESPONSE 2
In the management of patients with multiple injuries, controversy often arises as to the appropriate method of initial pelvic
stabilization. It is generally agreed upon that applying an external frame is appropriate in the setting of an unstable patient with
intraperitoneal fluid and labile blood pressure. Ex-fix placement can support hemodynamic stabilization and assist the general
surgeons with their laparotomy procedure. Plate or screw fixation of the pelvis should be delayed because the laparotomy takes
precedence in a patient who is hemodynamically unstable, and internal fixation in the presence of bowel contamination can result
in increased rates of infection.
Furthermore, Tile noted increased septic complications with intrapelvic hardware fixation in the setting of intraperitoneal soft
tissue damage and bleeding. Angiography and embolization may help with the pelvic bleeding, but will not stabilize the pelvis
during the laparotomy. The review article by Tile et al discusses the assessment of the patient with a pelvic injury, and summarizes
the various methods of temporary and definitive pelvic fixation.
11. PREFERRED RESPONSE 4
Of the pelvic ring injuries, APC type III have the highest rate of mortality, blood loss, and need for transfusion. They also have a
high rate of urogenital injury and abdominal organ injury. Lateral compression injuries (especially type III) have the highest rate
of head injury. Vertical shear and combined injuries also have significant rates of concomitant injuries. The referenced article by
Dalal et al is a review of Shock Trauma's pelvic ring injuries; they found significant increases in associated injuries as the grade of
pelvic ring injury increased, regardless of mechanism/pattern. The aforementioned information was also found to be true with
their patient review.

12. PREFERRED RESPONSE 1


Unstable anteroposterior compression (APC) pelvic fractures are most appropriately managed with a pelvic binder or
circumferential pelvic sheeting as described by Routt et al in the emergency room prior to definitive treatment. Illustration A
demonstrates the utility of circumferential wrapping for the case shown in Figure A. Rapid, temporary fixation of unstable pelvic
fracture patients with hemodynamic instability can be performed in the trauma bay. Pelvic binders can remain in place during
further diagnostic tests such as pelvic vessel angiography.
Bottlang et al performed a cadaveric study in JBJS of Young-Burgess type-II and III anteroposterior compression fractures and
found that a pelvic binder reduced rotation instability by 61%.
The study by Krieg et al followed 16 patients treated with pelvic binders and found that the binder reduced the pelvic fracture
displacement by 9% which closely approximated the reduction achieved with definitive fixation.

The Bottlang article published in JOT is a cadaveric study which determined that 180 +/- 50 Newtons of circumferential
compression is needed to stabilize an unstable pubis symphysis diastasis. Tile Type A pelvic fractures are stable and include
avulsion, iliac-wing, anterior-arch fracture due to a direct blow, or transverse sacrococcygeal fractures.

13. PREFERRED RESPONSE 2


High energy pelvic injuries such as the one seen in Figure A continues to be a source of high mortality in orthopaedics. Active
involvment of the orthpaedic surgeon in managing these life threatening injuries remains critical. A sheet or pelvic binder needs to
be emergently applied in this clinical scenario. An aggressive resuscitation protocol must also be initiated. The review article by
Hak et al discussed the advances in prehospital, interventional, surgical, and critical care that have led to increase survival rates for
pelvic injuries. Gonzalez et al found that initial coagulopathy in trauma patients was associated with decreased survival. They
noted that hypothermia and acidosis was well managed but pre-ICU coagulopathy was the most difficult to treat. They
recommended early FFP in a FFP:PRBC ratio of 1:1
14. PREFERRED RESPONSE 4
According to the referenced article by Griffin et al, the risk of postoperative loss of reduction is greatest with a vertical sacral
fracture pattern (13%, all within 3 weeks). There was no significant association between failure and anterior fixation method,
iliosacral screw arrangement or length, or any demographic or injury variable.
Their conclusion: "Percutaneous iliosacral screw fixation is a useful technique in the management of vertically unstable pelvic
fractures, but a vertical sacral fracture should make the surgeon more wary of fixation failure and loss of reduction."
15. PREFERRED RESPONSE 4
Pelvic external fixation with supraacetabular pins through the AIIS can be utilized to stabilize a pelvic fracture. While using this
technique, care must be taken not to injure the lateral femoral cutaneous nerve (LFCN). Gardner et al describe the technique for
placement of supraacetabular external fixation pins and state that pins in this location are more stable biomechanically compared
to other locations in the iliac crest. Grothaus et al performed a cadaveric study to determine the anatomic detail and variation of
the LFCN and the distances it traveled from various landmarks.The found the nerve to potentially be at risk as far as 7.3 cm
medial to the anterior superior iliac spine along the inguinal ligament and as much as 11.3 cm distal on the sartorius muscle from
the anterior superior iliac spine. Riina et al performed a cadaveric study to define the neurovascular structures at risk with the
placement of anterior-posterior locking screws in the proximal femur. They found that risks to the neurovascular structures during
anterior-posterior locking in the proximal femur are diminished if locking is performed above the level of the lesser trochanter.
16. PREFERRED RESPONSE 2
Figure A, a coronal CT image, shows a vertical sacral fracture, which creates a vertically unstable pelvic ring. Percutaneous
iliosacral screw fixation is a useful technique in the management of vertically unstable pelvic fractures, but a vertical sacral
fracture should make the surgeon more wary of fixation failure and loss of reduction.
According to the referenced study by Griffin et al, fixation failure of iliosacral screws was significantly associated with vertical

sacral fractures and not with any of the other answers listed above. All cases of hardware failure occured within the first 3 weeks;
a lesser relationship between hardware failure and sacroiliac joint injury was noted.
17. PREFERRED RESPONSE 3
Figure A shows an APC III injury, which is a rotationally and vertically unstable injury, with damage to the anterior ring, pelvic
floor, and posterior ligamentous stabilizing structures.
The referenced study by Sagi et al found that biomechanically, a percutaneous iliosacral screw and anterior ring internal fixation
was the most stable construct. In addition, he found no biomechanical support for addition of a second iliosacral screw.
18. PREFERRED RESPONSE 5
Unstable pelvic fractures can be devastating injuries often resulting in significant morbidity and even death.
According to the referenced study by Smith et al, fracture pattern and angiography/embolization were not predictive of mortality
in patients with unstable pelvic injuries. The three factors they found to be predictive were: increased blood transfusions in the
first 24 hours, age >60 years, and increased ISS or RTS scores. Deaths were most commonly from exsanguination (<24 hours) or
multiorgan failure (>24 hours).
Incorrect Answers: Choices 1-4 do not correlate with increased blood transfusions to the extent of Option 5.
19. PREFERRED RESPONSE 2
Unstable anterior and posterior pelvic ring injuries are amenable to percutaneous treatment if reduction is able to be obtained in a
closed manner and appropriate radiographic visualization is able to be achieved. In the 1996 reference by Routt et al, proper SI
screw placement is described. Pelvic inlet, outlet, and lateral sacral images must be obtained to safely place a percutaneous
iliosacral screw. The iliac cortical density seen adjacent to the SI joint is the anterior edge of the insertion safe zone, and is only
able to be seen on the lateral image. Failure to place the screw behind this radiographic line would lead to an "in-out-in" screw (in
the ilium, and then exiting anterior to the sacral ala, only to re-enter in the sacral body), which would cause direct injury to the L5
nerve root.
In the 2000 reference by Routt et al, they state "a thorough knowledge of pelvic osseous anatomy, injury patterns, deformities, and
their fluoroscopic correlations are mandatory for percutaneous pelvic fixation to be effective."
Illustration A shows a representative lateral sacral radiograph, with the major anatomic landmarks labeled. Safe SI screw insertion
in the S1 body should be underneath the sacral ala line to minimize risk of a "in-out-in" screw that would come out in the area of
the ala and injure the L5 nerve root that sits directly on top of this structure. Dysmorphic pelvic rings will often have a more
vertical sacral line, or one that starts more inferiorly.

20. PREFERRED RESPONSE 2


The figure shows an anteroposterior pelvic ring injury. The most common urological injury with pelvic ring injuries remains the
posterior urethral tear, followed by bladder rupture.

Watnik et al notes lower urinary tract (bladder to end of urethra) injuries in up to 25% of patients with this injury. He reports that
when contaminated urine communicates with the anterior arch, the possibility of infection exists, and early repair of bladder
disruptions with simultaneous anterior arch plating minimizes this risk.
Routt et al notes that even with simultaneous treatment of these injuries, complications are common (late stricture in 44%,
impotence in 16%, delayed incontinence in 20% of females, anterior deep pelvic infection in 4%). Despite this, they report that
early urological repairs are easily performed at the time of anterior pelvic open reduction and internal fixation.
21. PREFERRED RESPONSE 4
Pelvic ring injuries are associated with a high incidence of mortality mainly due to retroperitoneal hemorrhage. Early stabilization
is an integral part of hemorrhage control. Temporary stabilization can be provided by a pelvic sheet, sling, or an inflatable
garment. However, these devices lack control of the applied circumferential compression.
Krieg et al showed a pelvic circumferential compression device (PCCD) significantly reduced the pelvic width by 9.9 +/- 6.0% of
external rotation (APC) pelvic injuries, and did not overcompress internal rotation (LC) injuries.
Bottlang et al determined that a widened pelvis can be effectively reduced in the emergency department with a pelvic strap
(binder). While the other choices are urgent as well, hypotension caused by pelvic widening demands the most immediate
attention.
22. PREFERRED RESPONSE 3
The posterior sacroiliac ligaments are not disrupted in an APC type II pelvic fracture.
Young and Burgess classification of pelvic ring injuries is largely based on the mechanism and energy of injury. An APC type I
involves slight widening of pubic symphysis and/or anterior sacroiliac (SI) joint. An APC II is a continuation of this force, and
additionally involves a disrupted anterior SI joint, as well as sacrotuberous and sacrospinous ligaments. An APC III also involves
disrupted posterior SI ligaments, causing complete SI joint disruption with potential translational and rotational displacement.
The reference by Young et al is a classic article that describes the Young and Burgess classification of pelvic ring injuries. They
retrospectively analyzed pelvic ring radiographs and discussed four patterns of injury: anteroposterior compression, lateral
compression, vertical shear, and a complex/combined pattern.
The reference by Burgess et al is a validation of the aforementioned classification and study, as they reviewed 210 consecutive
patients who sustained a pelvic ring injury. They validated the classification scheme and found that overall blood replacement
averaged: lateral compression, 3.6 units; anteroposterior compression, 14.8 units; vertical shear, 9.2 units; combined mechanical,
8.5 units. Overall mortality was: lateral compression, 7.0%; anteroposterior, 20.0%, vertical shear, 0%; combined mechanical,
18.0%.
Illustration A shows an APC-II injury pattern - (a) is an outlet radiograph, (b) is an axial CT cut, (c) is a 3-D CT cut, and (d) is a
representative fixation construct.
Incorrect answers:
1,2,4,5: An APC - 2 pelvic ring injury involves injury to all of these structures.

23. PREFERRED RESPONSE 3


The clinical presentation and radiograph is consistent with an open-book type parturition-induced pelvic dislocation.
The case series by Kharrazi et al reports four patients treated with open-book type parturition-induced pelvic dislocations. The
authors advocate nonoperative treatment with bedrest and a properly positioned pelvic binder in the acute setting for patients with
a symphyseal diastasis less than 4.0 cm. All four patients had significant symptoms and radiographic widening (anterior splaying)
of the sacroiliac joints. The three patients who had presented acutely were treated with closed reduction and application of a pelvic
binder, while two had closed reduction of their pelvic dislocation while anesthetized with a general anesthetic. At latest follow-up
the diastasis at the pubic symphysis reduced to an average of 1.7 cm (range: 1.5-2.0) The authors advocate nonoperative treatment
with bedrest and a properly positioned pelvic binder in the acute setting for patients with a symphyseal diastasis of 4.0 cm of less
and operative treatment for diastasis greater than 4cm.
24. PREFERRED RESPONSE 4
Care must be taken when placing a retractor on the anterior aspect of the sacrum, as the L4 and L5 nerve roots are both at risk.
Illustration A shows a diagram of the lumbosacral plexus, indicating the proximity of the L4 and L5 nerve roots to the anterior
sacrum and SI joint.
The first referenced article by Atlihan et al reported on the anatomy of the anterior sacroiliac joint and reported that the L4 nerve
root is within 1 cm of the joint at its inferior margin.
The second article by Ebraheim et al found that the L4 and L5 nerve roots are 10 mm medial to the sacroiliac joint at the pelvic
brim.

25. PREFERRED RESPONSE 1


An anterior approach to the sacroiliac (SI) joint is indicated with displaced SI joint dislocations that cannot be reduced with closed
or percutaneous techniques. One contraindication to anterior exposure of the SI joint is comminuted sacral fracture patterns.

Posterior pelvic ring injuries that are unable to be reduced by closed techniques may require open reduction via anterior or
posterior approaches. Relative contraindications to anterior approach include comminuted sacral fractures, morbid obesity, iliac
wing external fixation, and ipsilateral diverting colostomy. In the presence of a comminuted sacral fracture, aggressive medial
dissection would be required and would place the L5 nerve root at risk.
Simpson et al describe their initial results with open reduction and internal fixation of the SI joint via an anterior exposure in a
series of 16 patients. They note that sacral alar comminution is a contraindication to the anterior approach
Jones provides an overview of the operative treatment of posterior pelvic ring injuries. He demonstrates reduction and fixation
techniques via both anterior and posterior exposures.
Incorrect Answers:
Answer 2: Prior laparotomy is not a contraindication to open anterior approach if the bowel is in continuity and there is no
evidence of wound infection
Answer 3: Supracetabular external fixation does not interfere with anterior approach to the SI joint
Answer 4: Anterior pelvic ring injuries such as parasymphyseal fractures do not affect the choice of approach
Answer 5: An ipsilateral proximal femur fracture does not affect the choice of approach
26. PREFERRED RESPONSE 2
The injury shown in Video V reveals a right sided posterior ilium fracture, which is known as a crescent fracture. The presence of
a crescent fracture is consistent with a lateral compression type 2 injury; this differentiates this from a type I injury. The ipsilateral
anterior sacrum has a small impaction injury anteriorly while the contralateral SI joint has a minor amount of anterior sacral
impaction indicative of a lateral compression type I injury.
The reference by Burgess et al is the primary source of the mechanism classification of pelvic ring injuries. Overall blood
replacement averaged 5.9 units (lateral compression, 3.6 units; anteroposterior compression, 14.8 units; vertical shear, 9.2 units;
combined mechanical, 8.5 units). Overall mortality was 8.6% (lateral compression, 7.0%; anteroposterior, 20.0%, vertical shear,
0%; combined mechanical, 18.0%).
Incorrect answers:
1: The presence of a crescent fracture means this is at least a LC-2 injury. The left-sided fracture pattern is consistent with an LC-1
pattern.
3: A vertical shear fracture pattern would exhibit some vertical displacement and does not typically exhibit the crescent fragment.
4: The fracture pattern does not match an anterior-posterior compression pattern.
5: The fracture pattern does not match an anterior-posterior compression pattern.
27. PREFERRED RESPONSE 5
Penetration of an iliosacral screw through the sacral ala would injure the ipsilateral L5 nerve root (great toe dorsiflexion). This can
be avoided with proper understanding of the sacral anatomy as well as iliosacral screw starting points. The three required views
for placement of this screw are: lateral sacral, pelvic inlet, and pelvic outlet.
The referenced study by Ziran et al is an excellent review of fluoroscopic evaluation of screw placement. They reported that the
anterior border of the S1 body is best seen with overlap of the S1 and S2 anterior cortex while the superior aspect of the S1
foramen is best seen with overlap of the S2 foramen on the superior pubic ramus.
The referenced study by Routt et al reviewed 177 patients with pelvic ring injuries treated with these screws and found that quality

triplanar imaging decreased intraoperative and postoperative complications. They also recommend supplemental fixation of
iliosacral screws with posterior plating in noncompliant patients.
28. PREFERRED RESPONSE 5
This question is a simple review of anatomy and nerve innervation. The L5 root is at risk with an "in-out-in" screw, as described in
the question, as the nerve root is just anterior to the sacral ala as it enters the true pelvis. L5 is primarily evaluated by extensor
hallucis longus function. L4 is tested with tibialis anterior function and S1 by gastroc-soleus function (ankle plantarflexion).
29. PREFERRED RESPONSE 5
Figure A shows an intraoperative outlet view, which provides the best visualization of the neural foramina (and possible screw
placement into these foramina). This view provides information regarding cephalad-caudad placement of the screw, whereas the
inlet view provides information regarding the anterior-posterior position of the screw. The lateral sacral view provides information
regarding the sagittal curvature of the sacral ala and gives information regarding possible iatrogenic L5 nerve injury as it goes
over the sacral ala.
The referenced article by Routt et al is a review article regarding the safety and techniques of percutaneous pelvic ring fixation.
30. PREFERRED RESPONSE 5
The clinical scenario is consistent with a high-energy sacral fracture. The radiographs in figures A and B demonstrate a sacral
fracture with posterior displacement of the right hemipelvis seen on the inlet view. Figures C and D are axial and sagittal CT
images which show a displaced fracture of the right hemisacrum along with a transvere fracture component through the S3 body .
Diminished perianal sensation is concerning for an S2 nerve root injury.
Mehta et al reviewed the current management of sacral fractures. They note that the S1 and S2 nerve roots are more likely to be
injured with sacral fractures as they occupy 1/3 to 1/4 of the neural foramina, as opposed to S3 and S4, which only occupy 1/6 of
the neural foramina.
Robles reviewed the current literature to ascertain principles of evaluation and treatment for transverse sacral fractures. The author
notes that injury to nerve roots S2 to S5 is manifested by impairment of urinary and anal continence and sexual function.
The first illustration demonstrates the sacral nerve root dermatomal distribution. The second shows a pelvic cadaver dissection
demonstrating the sacral nerve roots as they exit the foramina.

31. PREFERRED RESPONSE 1


Answering this question relies on knowledge of the Denis classification of sacral fractures and their associated risks of nerve
injury. Figure A represents a Denis Zone 3 (medial to the foramina) sacral fracture, which has the highest associated risk of nerve
injury.
Denis et al outlined a novel classification system of sacral fractures based on the position of the fracture line relative to the sacral

foramina. The authors found a 56.7% incidence of nerve injury in fractures that extended medial to the sacral foramina (zone 3),
compared with 28.4% for fractures through the foramina (zone 2), and 5.9% for fractures lateral to the foramina (zone 1).
Mehta et al reviewed the current principles for management of sacral fractures. They note that bowel, bladder and sexual
dysfunction occur in 76% of patients with zone 3 sacral fractures.
Illustration A below demonstrates the Denis classification of sacral fractures.
Incorrect Answers:
2. Figure B shows a zone 1 sacral fracture, which has a 5.9% incidence of nerve injury
3. Figure C shows a zone 2 sacral fracture, which has a 28.4% incidence of nerve injury
4. Figure D shows a sacroiliac joint dislocation, not a sacral fracture
5. Figure E shows a zone 1 sacral fracture with an associated iliac fracture (crescent fracture)

32. PREFERRED RESPONSE 3


Combined iliosacral and lumbopelvic fixation (triangular osteosynthesis) for sacral fractures has the greatest stiffness when used
for an unstable sacral fracture.
The referenced article by Schildhauer et al is a cadaveric study that examined the biomechanical properties of different fixation
constructs under cyclic loading and demonstrates that triangular osteosynthesis for unstable transforaminal sacral fractures
provides significantly greater stability than iliosacral screw fixation under in-vitro cyclical loading.
Illustration below shows the radiographic appearance of lumbopelvic fixation. The addition of iliosacral fixation would complete
triangular osteosynthesis.

33. PREFERRED RESPONSE 4


The posterior wall is best visualized on the obturator oblique pelvic view.
The obturator oblique and iliac oblique views make up the Judet views that are used to evaluate acetabular fractures, along with a
standard AP pelvis radiograph. The obturator oblique pelvic view is best to view the anterior column and posterior wall in detail.
The iliac oblique shows the profile of involved iliac wing, the posterior column, and the anterior wall.
Letournel reviewed his classification and treatment protocols, based on his 22 years of experience at that time. He noted that
perfect anatomical reduction of the acetabulum led to the best outcomes.
Patel et al reviewed of the Letournel classification, and they found moderate to high inter- and intra-observer reliability with this
classification system. The presence of articular displacement, marginal impaction, incongruity, intra-articular fragments and
osteochondral injuries to the femoral head were found to have less reliability (intra- and interobserver).
Illustration A shows a right-sided obturator oblique radiograph, while Illustration B shows a right-sided iliac oblique radiograph.
Illustration C shows a diagram of the obturator oblique radiograph, with the radiographic lines marked out.
Incorrect Answers:
Answers 1: Inlet pelvic imaging is best for assessing pelvic ring injuries (rotation and anterior-posterior or medial-lateral
translation).
Answers 2: Outlet pelvic imaging is best for assessing pelvic ring injuries (proximal-distal translation, rotation).
Answers 3: AP pelvis is a good screening tool for pelvic and acetabular fractures.
Answers 5: Iliac oblique pelvis is best for assessing the posterior column and anterior wall of the acetabulum.

34. PREFERRED RESPONSE 4


Dynamic fluoroscopic examination of the affected hip under anesthesia is considered the best method of predicting hip stability.
Fragment size, which can be calculated using the Keith, Moed, or Calkins method, can be used to predict hip stability
radiographically, however they are not as accurate. In general it is thought that posterior wall fractures involving less than 20% of
the posterior wall are stable, whereas those involving greater than 40%-50% are unstable. Unfortunately, this leaves an
indeterminent zone (20-40%) which does not provide guidance in treatment.
Moed et al retrospectively reviewed 33 patients with posterior wall fractures who underwent dynamic fluoroscopic stress testing
and compared the results of this testing to the Moed, Calkins, and Keith method of hip stability prediction. They found that the
Moed method is the only reliable technique that is predictive of hip stability for small fracture fragments while also being
predictive of instability for large fracture fragments. However, they also stated that there remain a substantial number of fractures
involving 20% or more of the posterior wall that are both stable and unstable by examination under anesthesia. Therefore, they
recommend dynamic fluoroscopic examination for assessment of hip stability in the presence of a posterior wall fracture.
Tornetta et al conducted a study in which dynamic fluoroscopic stress views were taken of 41 acetabular fractures that met the
criteria for non-operative management to determine subtle signs of instability. Of the 41 fractures, 38 were found to be stable and
91% of these had good or excellent outcomes at 2.7 years. They concluded that dynamic stress views can identify subtle instability
in patients who would normally be considered for non-operative treatment.
incorrect answers:
1-> Keith Method - Depth of the fracture segment in injured hip is compared to the contralateral intact posterior wall depth at the
level of the fovea
2-> Moed - Depth of the fracture segment in the injured hip is compared to contralateral posterior wall depth at the level of the
greatest amount of fracture involvement
3-> Calkins - Length of posterior acetabular arc from each hip is compared at the level of the greatest amount of fracture
involvement.
35. PREFERRED RESPONSE 4
Joint stability is critical for successful nonoperative management of posterior wall acetabular fractures. Recent evidence has
established that dynamic fluoroscopic stress examination is the best method to determine joint stability in the setting of a posterior
wall fracture. The obturator oblique view allows for the best evaluation of hip joint stability during examination for posterior wall
fractures.
Grimshaw and Moed retrospectively reviewed the results of patients with posterior wall acetabular fractures managed
nonoperatively after evaluation with dynamic fluoroscopic stress tests. At two year follow up, all had good to excellent Merle
dAubigne clinical scores for hip function and no evidence of post-traumatic hip arthritis on AP pelvis radiographs.

Tornetta retrospectively reviewed his results managing patients with dynamic fluoroscopic stress examination for acetabular
fractures which met radiographic nonoperative criteria. Good-to-excellent clinical results were seen in 91% of patients managed
nonoperatively
Tornetta reviewed management of acetabular fractures and Tornetta and Mostafavi separately reviewed management of hip
dislocations. In both articles, emphasis is placed on dynamic examination of posterior wall fractures as instability has been seen
with fractures comprising as little as 15% of the posterior wall.
Illustration A demonstrates two fluoroscopic images from a dynamic stress exam of a patient with a posterior wall fracture. The
image obtained in the obturator oblique view clearly demonstrates that the femoral head loses congruency with the acetabular
dome.
Incorrect Answers:
Answers 1 & 2: Posterior wall fragment size less than 40% was historically used as an indirect measure of stability, however
measurements of fragment size may be unreliable and instability has been seen with fractures much smaller than 40%
Answer 3: A history of hip dislocation was thought to indicate a more unstable fracture. In the level IV study by Grimshaw and
Moed, patients with an associated hip dislocation who were stable under stress exam had no significant difference in outcome with
nonoperative management.
Answer 5: The iliac oblique view is used to evaluate the anterior wall and posterior column. Displacement or instability of the
posterior wall would not be seen with this view

36. PREFERRED RESPONSE 5


The obturator oblique-inlet view, as seen in Illustration A, best demonstrates the position of a supra-acetabular screw or pin
relative to the tables of the ilium.
Starr et al review their initial results and technique of closed or limited open reduction and percutaneous fixation of acetabular
fractures. They defined two groups of patients who may benefit from this technique; elderly patients with multiple comorbidities
to facilitate early mobilization and restore hip morphology, and young patients with elementary fracture patterns and multiple
associated injuries.
Starr et al describe their operative technique and outcomes for a case series of 3 patients using percutaneous acetabular fixation to

augment open reduction of acetabular fractures. The authors state that, for placement of an anterior colum ramus screw, an iliac
oblique-inlet (not obturator oblique-inlet) will ensure that the screw is within the medullary canal of the ramus and does not exit
anterior or posterior.
Gardner and Nork describe a technique for placement of a large femoral distractor in the supra-acetabular region to compress
displaced posterior pelvic ring injuries. They note that the obturator oblique-inlet view is necessary to view the entire length of the
pin as well as to ensure that pin remains in bone.
Incorrect answers:
Answer 1: Relationshiop of the screw to the acetabulum is best evaluated with the obturator oblique-outlet view as well as the
iliac oblique view
Answer 2: The iliac oblique view is used to ensure the trajectory of the screw is superior to the sciatic notch
Answer 3: The Obturator oblique-outlet view, otherwise known as the "teepee" or "tear drop" view, is used to identify the start for
supra-acetabular implant placement
Answer 4: The gluteal pillar is not utilized as a start point when placing supraacetabular fixation, and the obturator oblique-inlet
view would not be ideal to visualize this region of the pelvis
Illustrations: A

37. PREFERRED RESPONSE 4


Figures A through C depict and AP pelvis and Judet views of a T-type fracture of the right acetabulum. The ilioinguinal approach
provides access to the anterior wall and anterior column for fracture fixation, in addition to allowing fixation of the nondisplaced
posterior transverse fracture line. The lateral femoral cutaneous nerve (LFCN) is at risk in the superficial part of the dissection.
Another option for the approach would be the modified Stoppa, which would also allow excellent access to the anterior column as
well as the internal aspect of the iliac wing and quadrilateral plate.
Five basic and 5 associated acetabular fractures.

Incorrect Answers:
Answer 1. Kocher-Langenbeck: access for posterior wall and column fractures
Answer 2. Watson-Jones: anterolateral approach best for the hip, not the anterior column of the acetabulum.
Answer 3. Extended iliofemoral: visualization for both column fractures
Answer 5. Hardinge approach: lateral approach for THA

38. PREFERRED RESPONSE 1


The patients hip radiograph demonstrates an os acetabuli marginalis superior which is a benign accessory ossification center found
in the superior aspect of the acetabulum. This can be commonly confused with an acute fracture or avascular necrosis. Although
the os acetabuli marginalis superior occasionally persists into adult life, it usually fuses to the acetabulum by the time an
individual reaches age 20.
Caudle et al provide a case report of a a patient with a painful os acetabuli marginalis superior. This was successfully treated with
resection of the fragment, and bone grafting. This was noted to be a very unusual source of hip pain in adolescents.
Incorrect Answers
2-The fovea capitis is the depression on the head of the femur where the ligamentum teres inserts. This can appear as a small
ossicle on the surface of femoral heads in skeletally immature patients. An example of this is shown in Illustration A.
3-Myositis ossificans is soft tissue calcification which develops after trauma, or more rarely, surgery. An example of myositis
ossificans around the hip is shown in Illustration B.
4-Avascular necrosis of the femoral head classically occurs in patient with a history of alcoholism, steroid use, or sickle cell
disease. Radiographs can demonstrate femoral head sclerosis, and eventually collapse of the articular surface. An example of
femoral head AVN in a patient with sickle cell disease is shown in Illustration C.
5-Acetabular fractures occur in the setting of trauma, and are relatively rare in the pediatric population. An example of a left sided
acetabular fracture is shown in Illustration D. Illustration E shows a right sided acetabular fracture through the triradiate cartilage.

39. PREFERRED RESPONSE 4


Patient functional outcomes after acetabular fractures have been shown to be related to postoperative hip strength, regardless of
surgical approach.
The reference by Borrelli et al evaluated muscle strength and outcomes after acetabular surgery via an anterior approach. They
report that hip extension strength was affected least (6%), whereas abduction, adduction, and flexion strength was affected to a
greater degree. They note that hip muscle strength after operative treatment of a displaced acetabular fracture directly influences
patient outcome.
The reference by Engsberg et al is a review of patients that underwent ORIF of acetabular fractures through anterior or posterior
approaches. They report that maximizing hip muscle strength may improve gait, and improvement in hip muscle strength and gait
is likely to improve functional outcome. Worsening functional outcomes were correlated with decreased gait kinematics and stride
length.
REFERENCES:

1. Borrelli J Jr, Ricci WM, Anglen JO, Gregush R, Engsberg J. Muscle strength recovery and its effects on outcome after open
reduction and internal fixation of acetabular fractures. J Orthop Trauma. 2006 Jul;20(6):388-95. PMID:16825963 (Link to
Abstract)
2. Engsberg JR, Steger-May K, Anglen JO, Borrelli J Jr. An analysis of gait changes and functional outcome in patients surgically
treated for displaced acetabular fractures. J Orthop Trauma. 2009 May-Jun;23(5):346-53.PMID:19390362 (Link to Abstract)
40. PREFERRED RESPONSE 3
Operative treatment is indicated for most displaced acetabular fractures to allow early ambulatory function and to decrease the
chance of post-traumatic arthritis. Among the various surgical approaches, the Kocher-Langenbeck allows direct exposure of both
the posterior column and posterior wall. Indications for using this exposure include posterior wall fractures, posterior column
fractures, combined posterior wall/posterior column fractures, and simple transverse fractures.
REFERENCES:
1. Routt ML. Surgical treatment of acetabular fractures. In: Browner BD, Jupiter JB, Levine AM, Trafton PG, Krettek C, eds.
Skeletal Trauma: Basic Science, Management, and Reconstruction, 4th ed. Philadelphia, PA: WB Saunders; 2009:1171-1218.
2. Smith WR, Ziran BH, Morgan SJ, eds. Fractures of the Pelvis and Acetabulum, 2nd ed. New York, NY: Informa Healthcare
Publishers; 2007:362-374.
41. PREFERRED RESPONSE 3
As reviewed in the referenced article by Starr et al, when placing a retrograde pubic rami screw, the pelvic inlet iliac oblique view
will best determine the anteroposterior placement of the screw in the pubic ramus. To ensure placement outside of the joint, the
outlet obturator oblique is best, but all other views should be incorporated into determination of the position of fixation, as the
corridor for this screw placement is quite narrow.
Illustration A shows a left sided inlet iliac view on a pelvic bone model.

REFERENCES:
1. Riley MC. Fractures of the acetabulum. In: Bucholz RW, Tornetta P, Heckman JD, Koval KJ, Court-Brown CM, eds. Rockwood
and Green's Fractures in Adults, 6th ed. Philadelphia, PA: Lippincott Williams & Wilkins; 2006:1665-1715.
2. Starr AJ, Reinert CM, Jones AL. Percutaneous fixation of the columns of the acetabulum: a new technique. J Orthop Trauma.
1998 Jan;12(1):51-8. PMID:9447519 (Link to Abstract)
42. PREFERRED RESPONSE 3
Figure A is an AP radiograph of a hip hemiarthroplasty.
Contact pressures from an insturmented hip endoprosthesis can have important implications in both implant positioning and
rehabilitation protocols.
Hodge et al implanted a pressure-measuring Moore-type endoprosthesis in a patient who had sustained a displaced fracture of the
femoral neck. They used this prosthesis to determine the measurement and telemetry of contact pressures in the hip for 36 months
post-operatively. The highest pressure, eighteen megapascals, was recorded while the patient was getting up from a chair using the
affected leg and was localized in the posterior superior portion of the acetabulum. This can be important in the post-operative care
of acetabular fractures, as patients are at increased risk of loss of fixation with greater acetabular contact forces. Interestingly, peak
pressures in vivo were found to be considerably higher than previously measured pressures in vitro.
Incorrect Answers:
1,2,4,5,-These regions of the acetabulum have less contact pressure compared to the posterior superior portion.
REFERENCES:

1. Hertling D, Kessler R. Management of Common Musculoskeletal Disorders: Hip, 4th ed. Philadelphia, PA: Lippincott Williams
& Wilkins; 2005:441-486.
2. Hodge WA, Carlson KL, Fijan RS, Burgess RG, Riley PO, Harris WH, Mann RW. Contact pressures from an instrumented hip
endoprosthesis. J Bone Joint Surg Am. 1989 Oct;71(9):1378-86. PMID:2793891 (Link to Abstract)
43. PREFERRED RESPONSE 4
The "corona mortis" (translated as crown of death) artery is a vascular variant that joins the external illiac and the obturator
artery as it crosses the superior pubic ramus. Tornetta et al did a study where "fifty cadaver halves were dissected to determine the
occurrence and location of the corona mortis. Anastomoses between the obturator and external iliac systems occurred in 84% of
the specimens. Thirty-four percent had an arterial connection, 70% had a venous connection, and 20% had both. The distance
from the symphysis to the anastomotic vessels averaged 6.2 cm (range, 3-9 cm)." The corona mortis can be injured in superior
ramus fractures and iatrogenically while plating pelvic ring injuries using the ilioinguinal approach.
Illustrations: A

REFERENCES:
1. Tornetta P III, Hochwald N, Levine R: Corona mortis: Incidence and location. Clin Orthop Relat Res 1996;329:97101 PMID:8769440 (Link to Abstract)
2. Letournel E, Judet R: Fractures of the Acetabulum, ed 2. New York, NY, Springer-Verlag, 1993, pp 375-381
44. PREFERRED RESPONSE 4
The radiograph in Figure A shows a transverse acetabulum fracture. The iliopectineal (anterior column) and ilioischial lines
(posterior column) are interrupted, revealing bicolumnar involvement; however, this is different than the both column fracture, as
a transverse pattern has articular surface still in continuity with the axial skeleton via the sacroiliac joint.
The referenced article by Patel et al showed a wide variation of inter and intra-observer agreement in interpreting radiographs of
acetabular fractures, with high agreement for basic radiographic classification and only slight to moderate agreement for other
radiologic variables such as impaction.
The other referenced article by Letournel is a great review article regarding the initial classification of these fractures as well as a
quick summary of his outcomes.
REFERENCES:
1. 2. Letournel E: Acetabulum fractures: Classification and management. Clin Orthop Relat Res 1980;151:81-106 PMID:7418327
(Link to Abstract)
2. Patel V, Day A, Dinah F, et al: The value of specific radiological features in the classification of acetabular fractures. J Bone
Joint Surg Br 2007;89:72-76 PMID:17259420 (Link to Abstract)
45. PREFERRED RESPONSE 4

The pelvic spur sign is indicative of a both column acetabular fracture. It is best seen on an AP or obturator oblique x-ray. The
spur is the intact portion of the ilium, still attached to the axial skeleton and seen posterosuperior to the displaced acetabulum
(typically medially displaced).
Illustration A shows the spur sign (arrows) on a CT image, while illustration B shows an obturator oblique of the pelvis and the
spur sign is shown with the long tailed arrow (on the left of the image).
Illustrations: A & B

REFERENCES:
1. Letournel E, Judet R: Fractures of the Acetabulum, ed 2. Berlin, Heidelberg, Germany, Springer-Verlag, 1981
2. Vrahas MS, Tile M: Fractures of the acetabulum, in Bucholz RW, Heckman JD (eds): Rockwood and Greens Fractures in
Adults, ed 5. Philadelphia, PA, Lippincott Williams & Wilkins, 2001, pp 1513-1545
46. PREFERRED RESPONSE 5
CT scanning is indicated in acetabular fractures for determination of surgical approach and techniques, evaluation of marginal
impaction and presence of intra-articular loose bodies (especially after hip dislocation), and evaluation of fracture piece sizes and
relative positions.
Kellam et al reviewed their initial experience with CT scanning and acetabular fractures, and noted a 25% change in surgical
planning when CT was utilized versus plain radiographs; they also noted the ability to detect marginal impaction and fracture
size/position was improved with CT.
REFERENCES:
1. Kellam JF, Messer A. Evaluation of the role of coronal and sagittal axial CT scan reconstructions for the imaging of acetabular
fractures. Clin Orthop Relat Res. 1994 Aug;(305):152-9. PMID:8050224 (Link to Abstract)
2. Letournel E, Judet R: Fractures of the Acetabulum, ed 2. Berlin, Springer-Verlag, 1993, pp 29-61
47. PREFERRED RESPONSE 2
A both column acetabular fracture is defined as an acetabular fracture with no articular surface in continuity with the remaining
posterior ilium (and therefore, axial skeleton). The spur sign is a radiological sign seen with these fractures, and is the posterioinferior aspect of the intact posterior ilium. The spur sign and other radiographic findings consistent with a both column acetabular
fracture can be seen in Illustration A (AP), Illustration B (obturator oblique), and Illustration C (iliac oblique).
Illustrations: A

REFERENCES:
1. Letournel E, Judet R: Fractures of the Acetabulum. New York, NY, Springer-Verlag, 1993, pp 253-254

2. Vrahas MS, Tile M: Fractures of the acetabulum, in Bucholz RW, Heckman JD (eds): Rockwood and Greens Fractures in
Adults, ed 5. Philadelphia, PA, Lippincott Williams & Wilkins, 2001, pp 1513-1545
48. PREFERRED RESPONSE 3
Early fixation of acetabular fractures is associated with lesser organ dysfunction, so therefore answer three is not true.
Plaisier et al showed the timing of acetabular and pelvic ring fracture fixation greatly impacted patient outcome. Patients who had
fixation within 24 hours of injury showed shorter length of stay in the hospital and ICU (decreased number of ventilator days),
improved functional outcomes including a highly likelihood of being discharged to home as opposed to a rehabilitation facility,
and lesser organ dysfunction.
The reference by Matta et al is a classic article that shows that patients fixed within 3 weeks of injury showed both a higher rate of
anatomical reduction and lower overall complication rate than patients with similar fracture patterns treated after 3 weeks.
REFERENCES:
1. Baumgaertner MR, Tornetta P III (eds): Orthopaedic Knowledge Update: Trauma 3. Rosemont, IL, American Academy of
Orthopaedic Surgeons, 2005, pp 271-280
2. Plaisier BR, Meldon SW, Super DM, et al: Improved outcome after early fixation of acetabular fractures. Injury 2000;31:8184 PMID:10748809 (Link to Abstract)
3. Matta JM. Fractures of the acetabulum: accuracy of reduction and clinical results in patients managed operatively within three
weeks after the injury. J Bone Joint Surg Am. 1996 Nov;78(11):1632-45.PMID:8934477 (Link to Abstract)
49. PREFERRED RESPONSE 2
Figures A through D show a comminuted both column acetabular fracture. In this injury, both columns are involved, with the
acetabulum losing all connection to the axial skeleton (sacrum). This differentiates it from all other patterns, where at least part of
the acetabular cartilage maintains connection to the sacrum.
Figure C shows the ischial spur, which is classically known as the spur sign and most easily seen on the obturator oblique
radiograph.
Incorrect Answers:
Answer 1: This injury has axial skeleton attachment to the acetabular cartilage through the posterior column.
Answer 3: This injury has axial skeleton attachment to the acetabular cartilage through the anterior and posterior columns.
Answer 4: This injury has axial skeleton attachment to the acetabular cartilage through the anterior column as well as the posterior
column, depending on fracture pattern.
Answer 5: This injury has no posterior column involvement, and therefore the posterior column maintains the axial skeleton
attachment to the acetabulum.
REFERENCES:
1. Letournel E, Judet R: Associated transverse and posterior wall fractures, in Letournel F, Judet R (eds): Fractures of the
Acetabulum, ed 2. Berlin, Heidelberg, Springer-Verlag, 1993, pp 201-221
2. Reilly MC: Fractures of the acetabulum, in Rockwood and Greens Fractures in Adults, ed 6. Philadelphia, PA, Lippincott
Williams & Wilkins, 2006, pp 1665-1714
50. PREFERRED RESPONSE 2
Borrelli et al examined the intraneural pressure of the sciatic nerve with the hip and knee in various different positions. They
found that the "sciatic nerve appeared to exceed published critical thresholds for alterations of blood flow and neural function only
when the hip was flexed to 90 degrees and the knee was fully extended." As a result, the leg is typically position with the hip in
extension (or minimal flexion) and the knee in about 90 degrees of flexion when performing acetabular surgery via a posterior
approach.
51. PREFERRED RESPONSE 2
The axial CT cut and Judet radiographic view shown reveals a transverse fracture pattern according to the Letournel classification
system. This can be determined by the fact that the articular surface of the acetabulum is attached to the intact portion of the ilium,
which is connected to the axial skeleton posteriorly through the sacroiliac joint. This differs from a both-column fracture, in which
the articular surface of the acetabulum has no attachments to the axial skeleton due to fracture line(s). The axial CT scan also
shows a vertical fracture line which is typical of a transverse fracture pattern.

Durkee et al review the classification schemes for these injuries, as well as comment on the importance of quality images (Judet
views, CT, etc).
Figures A and B show a transverse acetabular fracture with mild displacement.

52. PREFERRED RESPONSE 1


Madhu et al showed time to surgery was a significant predictor of radiological and functional outcome for both elementary and
associated displaced fractures of the acetabulum. Both anatomic reduction and functional outcome significantly worsened as time
to surgery increased. It was found anatomic reduction was more likely when surgery was within 15 days for elementary fracture
and 5 days for associated.
Incorrect answers:
2: No data exists showing a decrease in heterotopic ossification as time to surgery increases.
3: Neurologic injury is more associated with the initial injury.
4,5: Multi-organ failure was not commented on, but infection showed a trend towards being more likely with longer time to
surgery.
53. PREFERRED RESPONSE 4
The external iliac and obturator artery anastomose to form the corona mortis. During the Stoppa or ilioinguinal approach to the
pelvis, you need to be careful of the corona mortis because the vessels can cause significant bleeding especially if they retract into
the pelvis. In the Tornetta et al article, fifty cadaver halves were dissected to determine the occurrence and location of the corona
mortis. Anastomoses between the obturator and external iliac systems occurred in 84% of the specimens. Thirty-four percent had
an arterial connection, 70% had a venous connection, and 20% had both. The distance from the symphysis laterally to the
anastomotic vessels averaged 6.2 cm. The Okcu et al article showed similar results in 150 cadavers: they found vascular
anastomoses between the obturator and external iliac systems in 61% of the sides, and anastomotic veins in 52% of the exposures.
The mean distance between the anastomotic arteries and the symphysis pubis was 6.4 cm, and 5.6 cm for the communicating
veins. There seemed to be no significant difference between genders in the incidence of corona mortis and the distance between
communicating vessels and the symphysis pubis.
54. PREFERRED RESPONSE 5
There are 5 simple and 5 associated fracture types according to the classification system created by Judet and Letournel. The key
feature which distinguishes both column fractures from other associated types is that all articular segments are detached from the
intact portion of the ilium, which remains attached to the sacrum through the SI joint.
Although the transverse plus posterior wall, T-shaped, and anterior plus posterior hemi-transverse fractures all show involvement

of the anterior and posterior columns, they are not both columns because a portion of the articular surface remains in its normal
position, attached to intact ilium.
The intact ilium is responsible for the "spur sign" noted most prominently on the obturator oblique radiograph.
Illustration A demonstrates the 10 types of acetabular fractures as created by Judet and Letournel. Illustration B is an example of a
both column acetabular fractures as seen on the obturator oblique radiograph.
Illustration A

Illustration B

REFERENCES:
1. Browner BD, Jupiter JB, Levine AM (eds): Skeletal Trauma. Philadelphia, PA, WB Saunders, 1992
2. Kellam and Tile, Fractures of the Pelvis and Acetabulum
55. PREFERRED RESPONSE 4
The patient described in this question has sustained an insufficiency fracture, with the radiograph showing significant osteopenia
with comminution of both columns, the dome and the medial wall. Treatment of this problem should include reduction and
fixation of the column(s) and placement of a total hip arthroplasty (THA), with use of flanged and/or custom acetabular
components as needed.
The first referenced article by Weber et al reviewed delayed THA in acetabular fractures, and reported a 78% 10-year survival
rate, with worse outcomes in patients < 50yrs, males, weight >80kg, and patients with large residual segmental acetabular defects.
The second referenced article by Jiminez et al reviews the the use of THA after acetabular fractures, either in a delayed or acute

fashion. He reviews the techniques of reduction and arthroplasty placement.


The third referenced article by Mears reviews acute THA in osteopenic acetabular fractures, with presentation of treatment
algorithms and techniques. He reviews available case series on the subject, with a review of the techniques utilized.
REFERENCES:
1. Weber M, Berry DJ, Harmsen WS: Total hip arthroplasty after operative treatment of an acetabular fracture. J Bone Joint Surg
Am 1998;80:1295-1305 PMID:9759814 (Link to Abstract)
2. Jimenez ML, Tile M, Schenk RS: Total hip replacement after acetabular fracture. Orthop Clin North Am 1997;28:435446 PMID:9208835 (Link to Abstract)
3. Mears DC. Surgical treatment of acetabular fractures in elderly patients with osteoporotic bone. J Am Acad Orthop Surg. 1999
Mar-Apr;7(2):128-41. PMID:10217820

56. PREFERRED RESPONSE 4


Negative outcome factors have been shown to include: increasing patient age, time from injury to surgery (>3 weeks),
intraoperative complications, femoral head bone or cartilage injury, and fracture reduction > 1-2mm from anatomic. Choice of
surgical approach has not been shown to affect patient outcomes.
The referenced study by Matta evaluated outcomes of displaced acetabular fractures. The overall clinical result was excellent for
104 hips (40 per cent), good for ninety-five (36 per cent), fair for twenty-one (8 per cent), and poor for forty-two (16 per cent).
The clinical result was related closely to the radiographic result. These findings indicate that in many patients who have a complex
acetabular fracture the hip joint can be preserved and post-traumatic osteoarthrosis can be avoided if an anatomical reduction is
achieved.
57. PREFERRED RESPONSE 4
In the cited study, researchers measured tissue fluid pressure within the sciatic nerve in cadaveric specimens using a pressure
transducer. The hip and knee were taken through a combination of ranges and found that the clinically relevant increase in
pressure happened with the hip flexed at 90 degrees and the knee fully extended. They concluded that increased intraneural
pressure was related to excursion of the nerve as linear distance between the greater sciatic notch and the distal leg increase.
Hence, according to the question stem, to avoid traction injury, the reverse position should be implemented (hip extension and
knee flexion).
58. PREFERRED RESPONSE 3
Letournel and Judet developed a schematic representation of the acetabulum as being contained within asymmetric long anterior
and short posterior arms of an inverted Y.
On the bony pelvis, the ilioischial component becomes that posterior column and the iliopectineal line becomes the anterior
column. The Judet-Letournel classification system is based on this scheme. By careful evaluation of landmarks on a standard AP
pelvis radiograph, as well as on 45-degree oblique obturator and iliac views, the extent of injury can be determined accurately.
The AP view usually demonstrates the six fundamental landmarks relatively well as seen in illustration A. The obturator oblique
view reveals additional information about the anterior column and posterior wall(see illustration A(B), B). In an obturator oblique
view the x-ray beam is centered on and almost perpendicular to the obturator foramen. The iliac oblique view visualizes the
posterior column and anterior wall (illustration A(C), C). This view also shows the best detail of the iliac wing as the radiographic
beam is roughly perpendicular to the iliac wing. Inclusion of the opposite hip is essential for evaluation of symmetrical contours
that may have slight individual variations and to evaluate the width of the normal articular cartilage in each view in a pelvic series
(AP, Judet's) .
Illustration A

Illustration B

Illustration C

59. PREFERRED RESPONSE 3


The radiograph and CT images shown in A-D show an acute both column acetabular fracture with segmental posterior column
comminution. For difficult fractures with anterior displacement in which access to the entire anterior column is required, the
ilioinguinal or Stoppa approach is ideal. These approaches allow access to the anterior column as far as the symphysis and
includes the quadrilateral plate. Most both-column fractures can also be managed through these approaches, but only if the
posterior fragment is large and in one piece. In this case, the posterior column is in several pieces and requires either two
approaches or an extended approach, such as the iliofemoral. The original description of the ilioinguinal approach makes
intraarticular visualization of the hip impossible. If visualization of the joint is required, a T extension of the incision just medial
to the anterior-superior iliac spine can be made. Most surgeons accept that the joint is reduced when the fracture lines inside the
pelvis are reduced, and thus this extension is very rarely used.
The extended iliofemoral approach gives excellent visualization of the outer table of the ilium, the superior dome, and the
posterior column. The anterior column can be visualized to the iliopectineal eminence. The exposure is similar to that provided by
the triradiate approach with the additional benefit of access to the bone above the sciatic notch. The approach can be extended to
provide exposure to the iliac fossa; however, this is very rarely necessary and should be avoided. Extending the approach to the
inside of the pelvis greatly increases the risk of devascularizing segments of the acetabulum.
REFERENCES:
1. Letournel E, et al (eds). Fractures of the Acetabulum, ed. 2. 1993
60. PREFERRED RESPONSE 1
Transverse acetabular fractures separate the innominate bone into two fragments, the superior iliac and the inferior ischiopubic, by
a single fracture line that crosses the acetabulum horizontally. The iliopectineal and ilioischial lines are disrupted on the AP pelvis
radiograph. Axial CT scan of this fracture pattern at the level of the dome will show a vertical anterior to posterior fracture line.
Illustrations A-C show AP and Judet pelvic radiographs of a transverse fracture. Illustration D demonstrates the axial CT
appearance of this fracture type. Answer choice 2 is describing a both column injury, and answer choice 3 describes a T-type
fracture pattern. Answer choices 4 and 5 describe an anterior column and posterior column injury respectively. Judet et al provide

one of the first comprehensive reviews on acetabular surgical approaches, fracture types, and radiographic anatomy. Illustration E
demonstrates the acetabular classification scheme developed by Judet.
Illustrations: A B C D E

REFERENCES:
1. Letournel E, Judet R (eds): Fractures of the Acetabulum, ed 2. Berlin, Germany, Springer Verlag, 1993.
2. Judet R, Judet J, Letournel E. Fractures of the acetabulum: Classification and surgical approaches for open reduction.
Preliminary Report. J Bone Joint Surg Am.1964 Dec;46:1615-46. PMID:14239854
61. PREFERRED RESPONSE 2
The images demonstrate a posterior column acetabular fracture. These are best surgically treated with a Kocher-Langenbeck
approach, which allows access to the posterior column and posterior wall. Figure A shows disruption of the ilioischial line with an
intact iliopectineal line which is diagnostic of this fracture pattern. The CT image in Figure D shows the characteristic horizontal
(coronal) orientation of the column fracture when viewed on an axial CT. Illustration A shows the radiographic landmarks used in
diagnosing acetabular fractures. Illustrations B and C show the orientation of column and wall fractures respectively. Ilioinguinal
and Stoppa approaches allow access for anterior column fixation and symphysis fixation respectively. The extended iliofemoral
approach can be used to treat both column injuries, but has high rates of post-operative heterotopic ossification.
Illustrations: A

62. PREFERRED RESPONSE 5


Moed et al performed a study to determine the clinical outcome in patients in whom a displaced fracture of the posterior wall of
the acetabulum had been treated by open reduction and internal fixation. They were able to show good to excellent clinical results

for patients who underwent anatomic reduction and internal fixation of posterior wall acetabulum fractures as assessed using
radiographs. Fractures in elderly patients and patients who sustained extensive comminution were more likely to have worse
clinical result.
In a separate study, Moed et al. evaluated the results of 67 patients who underwent ORIF of a posterior wall fractures by assessing
the accuracy of postoperative AP pelvis, obturator oblique films, iliac oblique films, and CT scans. They found that postoperative
pelvic CT scan was the most accurate way to judge final fracture reduction and was able to pick up residual fracture displacements
that were not seen on postoperative plain radiographs. They concluded that the accuracy of reduction as assessed on postoperative
CT scan was the most reliable indicator of clinical outcomes.

60 Days Orthobullets:
Answers Bank
Day 5

1)PREFERRED RESPONSE 2
The radiograph shown in Figure A reveals a left hip dislocation, with some obscuring of detail secondary to the trauma backboard.
CT scans should be obtained following a hip dislocation to evaluate for fractures or impacted areas of the femoral head or
acetabulum, as well as noncongruent reductions and free intraarticular joint fragments.
The referenced study by Brumback et al comments on the importance of post-reduction CT scans and found that 23% of their
posterior wall fractures had associated marginal impaction, with 94% of these discovered via preoperative CT scan.
2) PREFERRED RESPONSE 1
Traumatic hip dislocation results from the dissipation of a large amount of energy about the hip joint. Clinically, these forces often
are first transmitted through the knee en route to the hip. It is therefore logical to look for coexistent ipsilateral knee injury in
patients with a traumatic hip dislocation.
Schmidt, et al, prospectively evaluated the ipsilateral knee of all patients who had a traumatic hip dislocation and found that 93%
had abnormalities on MRI of the knee, with effusion (37%), bone bruise (33%), and meniscal tear (30%) being the most common
findings. They suggest liberal use of MRI to the ipsilateral knee if injury is suspected.
3) PREFERRED RESPONSE 4
Figure D represents a Pipkin II femoral head fracture, which is defined as a fracture which is superior to the fovea. Differentiation
between Pipkin I and Pipkin II fractures can be important, as suprafoveal injuries often require surgical fixation. Illustration A
demonstrates the Pipkin fracture types.
Droll et al review femoral head fracture evaluation and treatment. They discuss non-operative indications (typically reserved for
Pipkin I injuries) which include an anatomic or near anatomic reduction (<2 mm) of the femoral head fragment, a stable hip, and
no interposed fragments preventing a congruent hip joint. They also state that an anterior surgical approach is preferred for
fixation of Pipkin II type injuries.
Pipkin discusses the evaluation and treatment of hip fracture dislocations as defined by the Stewart and Milford classification
scheme. He focuses on Grade IV injuries, which at the time lacked an appropriate treatment algorithm.
Incorrect Answers:
1,2-Figures A and B show Pipkin IV fractures, due to the presence of an associated acetabular fracture.
3-Figure C shows a Pipkin I fracture, as the fracture is infra-foveal
5-Figure E shows an example of a Pipkin III injury due to the presence of an associated femoral neck fracture.
4) PREFERRED RESPONSE 3
Based on the injury sustained and the risk factors (namely chronic renal failure) shown, the patient will likely have a postoperative
mortality of 45% at 2-years post operatively.
Hip fractures often occur older patients from low energy type mechanisms of injury. In femoral neck fractures, the relative preinjury mobility is the most significant determining factor for the postoperative survival.
Bhattacharya et al. reviewed factors affecting acute mortality in patients undergoing orthopaedic procedures. They found that
patients with history of chronic renal failure had an univariate mortality rate of 9%. The most predictive factors of death were:
chronic renal failure, CHF, COPD, hip fracture and an age greater than 70 years of age.
Karaeminogullari et al. reviewed clinical outcomes of operative treatment of hip fractures in patients on chronic hemodialysis. A

total of 29 patients sustained 32 hip fractures. The mortality rate, with an average follow up of 23 months, was found to be 45%.
They found a significant association between age and risk of mortality.
Figure A shows an AP radiograph demonstrative of a displaced femoral neck fracture. Figure B shows a postoperative radiograph
with an appropriately placed hip hemi-arthroplasty. Illustration V is a video that provides a brief overview of the evaluation and
management of femoral neck fractures.
Incorrect Answers
1, 2, 4, 5: Based on the current literature, the post operative mortality rates at close to 2 years of follow up are 45%
5) PREFERRED RESPONSE 5
Figure A shows a displaced right femoral neck fracture in an active, healthy elderly patient. Treatment of her hip fracture with
total hip arthroplasty (THA) has shown to have the lowest re-operation rates and best functional outcome scores when compared
to internal fixation devices and hemiarthroplasty.
Large studies have shown the incidence of femoral head AVN to be approximately 30-45% with displaced femoral neck fractures
(Garden III-IV). For this reason, treatment of these injuries in elderly patients have supported arthroplasty over ORIF. Treatment
of patients with THA vs. hemiarthroplasty have also been investigated. Studies have shown that THA has lower re-operation rates
and improved functional outcome scores in younger, active elderly patients compared to hemiarthroplasty.
Avery et al. prospectively followed a cohort of 81 patients treated with THA vs. hemiarthroplasty in high functioning elderly
patients with displaced femoral neck fractures. They showed a lower mortality rate (p = 0.013) and trend towards superior
function in patients treated with THA. Advantages with THA vs hemiarthroplasty must be traded off against a slightly higher risk
of dislocations.
Hedbeck et al. performed a randomized controlled trial involving 120 elderly patients with acutely displaced femoral neck
fractures that were treated with either bipolar hemiarthroplasty or THA. They showed Harris hip scores and EQ-5D scores in
favour of THA. They suggested treatment with THA in elderly, lucid patients with displaced femoral neck fractures.
Figure A is a AP pelvic radiograph. The most obvious finding is a displaced femoral neck fracture.
Incorrect Answers:
Answers 1,2,3: Arthroplasty of any type has been shown to have the least amount of complications and greatest functional
outcome scores compared to internal fixation devices. Indications for treatment of femoral neck fractures with internal fixation
include: (1) stable or unstable fractures in young or physiologically young patients (2) stable fragility fractures (Garden I and II)
in low demand elderly patients.
Answer 4: Hemiarthroplasty is most appropriate for displaced femoral neck fractures (Garden III or IV) in low-functional demand
elderly patients.
6) PREFERRED RESPONSE 3
A femoral neck fracture has been shown to have an estimated mortality rate of 20% or more at one year after injury, and estimates
regarding loss of independence are at 50%.
Most literature on the topic points out the highest mortality rate is in the first 3-6 months, with mortality rates of 14-36% at one
year; mortality rates have been found to return to normal, age-matched controls after that.

Egol et al. provide a review of the factors involved in functional recovery of patients with femoral neck fractures. They report the
successes of integrated care pathways and review the risk factors (institutionalization, comorbidities, etc.) that go into the
outcomes of these patients. They recommend comanagement of these patients with a medical or geriatric service in order to
improve patient outcomes.
Incorrect Answers:
Answer 1,2,4,5: These can be debilitating injuries, but no evidence exists to show these injuries are associated with these levels of
morbidity and mortality at one year.
7) PREFERRED RESPONSE 4
The clinical presentation is consistent with a femoral neck nonunion, which is supported by the presence of new varus collapse
and shortening which was not noted on prior radiographs. The most appropriate method to treat this complication is valgus
intertrochanteric osteotomy of the femur with blade plate fixation.
Valgus intertrochanteric osteotomies function by making a vertical fracture more horizontal, converting shear into compressive
forces. It also helps correct the varus position of the fracture nonunion.
Watson et al performed a retrospective review of the complications associated with combination femoral neck/shaft fractures and
found 13 patients who had healing complications develop after their index surgical procedure. Eight femoral neck nonunions
occurred, and these healed after either valgus intertrochanteric osteotomy (seven patients) or compression hip screw fixation (one
patient).
Jackson et al state that nonunion may occur in up to 43% of displaced intracapsular femoral neck fractures. The authors present an
evidence-based algorithm regarding procedures for treatment of femoral neck nonunion and the roles of refixation, osteotomy,
grafting, and prosthetic replacement when indicated.
Angelini et al provide a review on salvage procedures after failed fixation of hip fractures. The authors state that in the setting of a
nonunion in the younger patients with a well-preserved hip joint, treatment should typically involve revision internal fixation with
or without osteotomy or bone grafting. They conclude that overall, salvage of nonunions of femoral neck and intertrochanteric hip
fractures in properly selected patients can provide patients with good to excellent results.
Figure A shows a femoral neck nonunion with varus collapse. Illustration A shows an example of a valgus intertrochanteric
osteotomy with blade fixation.

Illustrations: A

8) PREFERRED RESPONSE 1
The strongest portion of the femoral neck is the posterior inferior neck in the region of the femoral calcar. The optimal
biomechanical configuration includes an inverted triangle pattern with the single screw in the inferior aspect of the femoral neck
adjacent to the calcar.

Booth et al performed a cadaveric study comparing central versus calcar (cortical-adjacent) fixation. The results demonstrated
significant improved stability, load, stiffness, and displacement in all tested parameters for the group with calcar-adjacent screw
fixation.
Lindequist and Trnkvist performed a Level 4 study of 72 femoral neck fractures. They found that all 5 of their nonunions had
screws placed greater than 3mm from the femoral calcar. Additionally, 16 of 18 fractures healed in the group of displaced fractures
where both the fixating screws were placed within 3 mm from the femoral neck cortex.
Gurusamy et al performed a Level 4 study of 395 patients undergoing femoral neck fixation. They found a reduced spread of the
screws on the lateral view was associated with an increased risk of nonunion of the fracture.
Illustration A depicts the optimal configuration of an inverted triangle with the single screw being inferior and all of the screws
being cortical adjacent.
Illustrations: A

9) PREFERRED RESPONSE 2
The femoral neck fracture seen in Figure A is considered by most to be a surgical emergency in a 25-year old due to the at risk
blood supply of the femoral head. Due to the intra-capsular hematoma and hemarthrosis that occurs, the involved extremity is
often found to be in external rotation and flexion as the intra-capsular volume is the highest in this position. Debate exists of the
significance of the pressure caused by the fracture hemarthrosis as it is believed by some to cause a local compartment syndrome
adding further insult to the already tenuous blood supply. Advocates of early fixation have proposed that reduction maneuvers
without capsulotomy can compromise the circulation of the femoral head by increasing the hip joint pressure. Maruenda et al
showed in their study of 34 consecutive patients with femoral neck fractures that the mean intracapsular pressure was the highest
with the hip in extension and internal rotation.
Bonnaire et al in their prospective study of 55 patients with intracapsular femoral neck fractures found the lowest pressure to be at
70 degrees of flexion. Both Maruenda and Bonnaire's studies showed no significant intracapsular pressure difference based on
fracture displacement.
Corollary to adult fracture patients, it is also noted that pediatric patients with a septic hip hold their hip in a flexed and externally
rotated position to maximize intracapsular volume as shown in Illustration A.
Illustrations: A

10) PREFERRED RESPONSE 4


The ability to define the location of a femoral neck fracture aids in determining the optimal treatment. The addition of a "tractioninternal rotation" view (as described in Answer 4) to standard hip radiographs may assist with classification of femoral neck
fractures by accounting for the anteversion of the femoral neck.
Koval et al performed a study in which proximal femur fractures were classified with standard radiographs (AP pelvis, AP hip,
cross-table lateral), followed by the addition of a physician-assisted view with traction and 15 degrees of internal rotation. The
inclusion of the traction-internal rotation view led to increased agreement in classification between the authors, and was especially
helpful for differentiating displaced femoral neck fractures versus stable intertrochanteric fractures.
11) PREFERRED RESPONSE 4
An AP pelvis radiographs with a displaced femoral neck fracture is seen in Figure A. It important to note that degenerative
changes are seen on this image. Both references suggest that elderly active individuals should be treated with a primary total hip
after displaced femoral neck fractures.
In the first study by Blomfeldt et al, the group reviewed a series of patients who underwent either an acute primary total hip
arthroplasty for a femoral neck fracture or a delayed primary hip after an attempt at ORIF. They found that the group treated with
an acute primary total hip arthroplasty had better Harris hip and quality of life scores.
The second reference from Blomfeldt et al, studies a population of active elderly patients randomized to either a total hip
arthroplasty or bipolar for femoral neck fractures. The group found no mortality or dislocation difference between the groups, but
higher Harris hip scores at 1 year in patients treated with a total hip
arthroplasty.
12) PREFERRED RESPONSE 5
Femoral neck fractures in young patients are difficult to treat, and AVN is a significant concern. Despite advances in both imaging
and implants, this injury often leads to functional impairment.
Haidukewych et al followed treatment of femoral neck fractures in young patients. They found almost 10% of displaced fractures
were associated with the development of nonunion, where as 27% were associated with the development of osteonecrosis. Their
results were influenced by fracture displacement and the quality of reduction. Varus malreduction most closely correlates with
failure of fixation after reduction and cannulated screw fixation.
Swiontkowski reviews both the treatment and post operative complications in intracapsular hip fractures. In this Current Concept
Review, the rate of AVN was discussed as being related to the pre-operative degree of displacement seen on radiographs.

Incorrect Responses:
Answers 1 & 4: While each of these complications do occur, they are less common and are related to the approach and degree of
surgical dissection.
Answer 2: Hip instability is relatively uncommon.
Answer 3: Nonunion rate is significant but lower than the AVN rate. It is has been associated with the degree of initial
displacement and varus malreduction.
13) PREFERRED RESPONSE 3
The answer is total hip arthroplasty (#3).Both references suggest that elderly active individuals should be treated with a primary
total hip after displaced femoral neck fractures.
Keating et al randomized 207 patients to be either treated with ORIF, bipolar hemiarthroplasty, or total hip arthroplasty. There was
no mortality difference among the three groups, however the rate of secondary surgery was highest in the ORIF group (39%
compared with 5% in the group treated with bipolar hemiarthroplasty and 9% in the group treated with total hip replacement).
Furthermore, the fixation group had the worst hip-rating-questionnaire and EuroQol scores at four and twelve months.
Tidermark et al in a randomized controlled trial, studied the difference between ORIF and total hip replacement in 102 patients.
The total hip replacement group showed a lower overall complication rate (36% versus 4%) and higher hip function scores in
regard to pain, movement and walking.
Illustration A shows the division of proximal femur fractures according to location.
Illustrations: A

14) PREFERRED RESPONSE 4


Figure B shows a bipolar hemiarthroplasty that was performed for a displaced femoral neck fracture. The advantages of
hemiarthroplasty, compared with total hip arthroplasty, for the treatment of displaced femoral neck fractures include the more
limited nature of the procedure (decreased blood loss and operative time) and the lower risk of instability. The disadvantages
include the possible development of pain in the groin and acetabular erosion which increases the risk for revision surgery. Sim et
al reviewed 126 consecutive hybrid total hip arthroplasties done for acute femoral neck fractures. Minimum follow up was 10.1
years. They noted a high dislocation rate (10%), yet overall good clinical outcome with 87/102 patients who were alive at latest
follow up reporting either no or only minimal pain.
15) PREFERRED RESPONSE 4
Several studies have shown that only patient age and pre-injury functional independence measure scores were independent
predictors of functional outcome after hip fracture.
The Soderqvist et al study showed that a Short Portable Mental Status Questionnaire score of <3 and male gender were associated
with an increased mortality rate during the first twelve months. Moreover, patients with a score of <3 had a significantly worse
outcome with regard to the ability to walk and to perform the activities of daily living.
The referenced study by Holt et al is a prospective review of 1000 hip fractures and reported that pre-injury mobility to be the

most significant determinant for post-operative survival.


The referenced study by Cornwall et al found that six-month mortality was lowest for patients with nondisplaced femoral neck
fractures (5.7%) and highest for patients with displaced femoral neck fractures (15.8%), but multivariate analysis only identified
preinjury function as an independent predictor of mortality.
16) PREFERRED RESPONSE 2
Figure A shows a displaced femoral neck fracture.
Moran et al studied 2660 elderly patients who underwent surgical treatment of a hip fracture to determine whether a delay in
surgery affects postoperative mortality. The 30-day mortality for patients for whom the surgery had been delayed for more than 4
days was 10.7%, compared to 7.3% in those delayed 1-4 days. The group delayed >4 days also had significantly increased
mortality at 90 days and 1 year. Patients who had been admitted with an acute medical comorbidity that required treatment prior to
the surgery had a 30-day mortality of 17%, which was nearly 2.5 times greater than that for patients without and acute
comorbidity. The study concluded that patients with medical comorbidities that delayed surgery had 2.5 times the risk of death
within 30 days after the surgery compared with patients without comorbidities. Mortality was not increased when the surgery was
delayed up to four days for patients who were otherwise fit for hip fracture surgery, however, a delay of more than four days
significantly increased mortality.
Healy et al examined 120 patients who underwent surgical treatment of 186 displaced femoral neck fractures with either internal
fixation, hemiarthroplasty, or total hip arthroplasty. Arthroplasty was associated with more independent living, and was more costeffective than internal fixation. There was no difference in rates of reoperation or mortality, but arthroplasty produced a longer
interval to reoperation or death. They concluded that total hip arthroplasty was the best treatment for displaced fractures of the
femoral neck in their series.
Illustration A shows a scoring system developed by Rogmark et al to aid in decision making.
Illustrations: A

17) PREFERRED RESPONSE 4


An ipsilateral femoral neck fracture occurs in approximately 6% to 9% of all femoral shaft fractures. A comminuted midshaft
femoral fracture secondary to axial loading should alert the treating physician to the possibility of an associated femoral neck
fracture. As a result, trauma CT scans should be reviewed for non to minimally displaced femoral neck fractures during the initial
work up.
Watson et al did a retrospective review of 13 patients who had healing complications develop after their index surgical procedure

for ipsilateral femoral shaft and neck fractures. Six of the eight (75%) femoral neck nonunions occurred after the use of a second
generation, reconstruction-type intramedullary nail. Factors contributing to nonunion of the femoral shaft were the presence of an
open fracture, use of an unreamed, small diameter intramedullary nail, and prolonged delay to weightbearing. Lag screw fixation
of the femoral neck fracture and reamed intramedullary nailing for shaft fracture stabilization were associated with the fewest
complications.
Peljovich et al discuss that several treatment options are described in the literature, but no clear consensus exists regarding the
optimal treatment of neck/shaft fractures. Due to the the potentially devastating complications of the femoral neck fracture in
young patients (avascular necrosis, nonunion, and malunion), the neck fracture should be treated first followed by the shaft.
Current recommendations involve treating the neck with a sliding hip screw versus cannulated screws followed by intramedullary
nailing of the femoral shaft.
18) PREFERRED RESPONSE 3
After a femoral neck fracture, patients often present with their injured hip in a flexed, abducted, and externally rotated position
due to decreased pain from minimization of the capsular distension from fracture hematoma (if the capsule isn't disrupted).
In the referenced study by Bonnaire et al, extension and internal rotation had the highest intracapsular pressure. They found that
the greatest decrease in pressure was found with flexion, abduction and external rotation. This is a possible etiology to the
common position in which these patients will present to the emergency room. Traction was shown to increase pressure in the joint
capsule.
Incorrect Answers: 1,2,4,5: All of these positions have increased pressure as compared to the position of flexion, abduction, and
external rotation.
19) PREFERRED RESPONSE 1
Figure A shows a displaced femoral neck fracture. Avascular necrosis (AVN) and nonunion are the two most common
complications after femoral neck fractures in the young adult.
Dedrick et al found nonunion of the fracture site was observed in 20% and avascular necrosis in 36% of young patients with
femoral neck fractures. In addition, they reported that of patients with subcapital fractures, 83% developed nonunion or avascular
necrosis, compared to 21% with a more distal femoral neck fracture.
20) PREFERRED RESPONSE 3
Figure C shows a femoral neck fracture that underwent closed reduction and percutaneous pinning with three screws, that has
gone on to femoral neck nonunion. Figure A shows slipped capital femoral epiphysis (SCFE). Figure B shows a subtrochanteric
fracture status post intramedullary nailing. Figure D shows a hip labral tear. Figure E shows hip avascular necrosis.
In the case of femoral neck nonunion, a valgus intertrochanteric osteotomy helps to convert any shear force of a vertical fracture
line (especially with a high Pauwel's angle) into a horizontal compressive force (Illustration A.)
In their series of 50 patients with femoral neck nonunions, Marti et al treated all patients with Pauwels valgus intertrochanteric
osteotomies. At an average followup of seven years, they reported that only 7 patients needed prosthetic replacement. However,
nearly half of the patients showed radiographic signs of avascular necrosis (only 3/22 were symptomatic). They conclude that
valgus intertrochanteric osteotomy yields good results for young patients with femoral neck nonunions.
Illustrations: A

21) PREFERRED RESPONSE 4


In the scenario of an elderly patient with questionable fixation into the femoral head and a non-healed femoral neck fracture,
proper treatment is arthroplasty. In a physiologically younger patient, reduction and fixation of the fractures (femoral neck and
subtrochanteric, if present) with methods such as a valgus producing osteotomy at the level of the subtrochanteric fracture are
recommended.
Figure A shows cannulated screw fixation of a right femoral neck fracture.
The referenced study by Oakey et al evaluated strength of proximal femurs after cannulated hip screw placement and found that
placement of an inverted triangle had a higher ultimate load to failure than placement in a standard triangle format (two screws
distal).
22) PREFERRED RESPONSE 1
TIp-apex distance (TAD) as it relates to a lag screw in the femoral head is the summation of the distance between the end of the
screw and the apex of the femoral head on AP and lateral radiographs. This is shown in Illustration A.
TAD is a useful intraoperative indicator of deep and central placement of the lag screw in the femoral head, regardless of whether
a nail or a plate is chosen to fix a fracture. A TAD of <25mm has been shown to minimize the risk of fixation cut-out in stable and
unstable intertrochanteric hip fractures.
Baumgaertner et al examined factors leading to the failure of sliding hip screws (SHS) in the treatment of 198 intertrochanteric
fractures. They determined that the tip-apex distance (TAD) is a reproducible, standard measurement to predict SHS failure. The
average TAD for successful fractures was 24mm while the average TAD for failures was 38mm. No screw with a TAD <25mm
failed. Calculation of the TAD is shown in Illustration B.
Kyle et al reviewed 622 intertrochanteric fractures. For unstable patterns, a SHS was superior to a fixed angle nail. Early
ambulation and weight bearing contributed to improved results.
Illustrations B and C show a lag screw with an excessive TAD, and subsequent failure of fixation.
Illustrations: A

Illustrations:B

Illustrations:C

23) PREFERRED RESPONSE 2


Left-sided unstable intertrochanteric hip fractures are at increased risk of malreduction compared to unstable right-sided fractures
fixed with dynamic hip screws. In left-sided fractures the rotational torque imparted to the proximal head and neck fragment can
cause loss of reduction leading to potential failures of fixation. With these left sided injuries, the rotational torque can cause an
anterior spike, whereas with right-sided injuries the rotational torque causes compression and reduction of the fracture. In
addition, if a nail is used for these injuries and the proximal fracture fragment is not being held by the nail itself, this phenomenon
can be seen as well.
Mohan et al conducted a study to assess the effect of clockwise rotational torque onto the fracture configuration in unstable and
stable intertrochanteric fractures fixed with a dynamic hip screw construct. They found that 11 out of 30 unstable fractures showed
an anterior spike (flexion malreduction) in left-sided fixations due to clockwise torque. This malreduction was not present in rightsided or stable fractures.
Illustrations A and B are images from Mohan et al's study showing the rotational affect on the fracture with placement of a
dynamic hip screw.
Illustrations: A

Illustrations:B

24) PREFERRED RESPONSE 2


Clinton et al performed a Level 2 study of 8049 women that demonstrated that proximal humeral fractures independently
increased the risk of a subsequent hip fracture. The risk was more than five times in the first year after the humeral fracture but
was not associated with a significant increase in the hip fracture risk in subsequent years.
Johnell et al performed a Level 4 review that found that men and women had an increased risk of hip, forearm and spine fractures
following a prior spine, hip or shoulder fracture.
Schousboe et al performed a Level 2 investigation of 9516 community-dwelling elderly women and found that 521 hip fractures
occurred after 10 years of follow-up. They found that prior non-spine fractures, non-hip fractures, and prevalent moderate to
severe radiographic vertebral fractures were modestly associated with incident hip fracture.
25) PREFERRED RESPONSE 3
Illustration A found below depicts the method to calculate Tip-apex distance (TAD). The tip-apex distance is the sum of the
distances from the tip of the lag screw to the apex of the femoral head as seen on the AP and lateral radiographs.
Baumgaertner et al reported that he had no screws cut out if the tip-apex distance was less than 25mm. Tip-apex distance was the
strongest predictor of cutout. Increasing age of the patient, poor reduction, use of a high angle sideplate, and unstable fracture
were weaker predictors of cutout.
Kyle et al demonstrated that obtaining an anatomic reduction when using a sliding hip screw with intertrochanteric fractures leads
to the best radiographic and clinical outcomes.
Illustrations: A

26) PREFERRED RESPONSE 1


Baumgaertner et al in their classic study in 1995 determined that the position of the lag screw in the femoral head influenced the
risk of cutout of a dynamic hip screw construct in treatment of intertrochanteric fractures. They had no cutouts if the tip-apex

distance on the combined AP and lateral radiographs was less than 25 millimeters. Subsequent studies demonstrated a decreased
cutout rate once people were aware of the tip-apex distance importance.
27) PREFERRED RESPONSE 1
The use of intramedullary (cephalomedullary) devices has increased in the last ten years despite a lack of evidence to support
superiority over extramedullary implants (sliding hip screws)
Intertrochanteric hip fractures remain one of the most common injuries managed by Orthopaedic surgeons. The optimal form of
surgical stabilization for these injuries has been a topic of debate, however several recent studies have demonstrated equivalent
outcomes with long cephalomedullary nails and sliding hip screws.
Anglen et al. reviewed the database of orthopaedic surgeons taking their oral board examination. The authors found that the use of
intramedullary nails for intertrochanteric hip fractures dramatically increased from 3% in 1999 to 67% in 2006. The authors calls
attention to the fact that reimbursement was higher until 2010 for intramedullary nails despite a lack of evidence demonstrating
superiority.
Forte et al. evaluated geographic variation in the use of intramedullary nails to treat intertrochanteric hip fractures. The authors
found significant regional variation in the use of these devices despite similarities in the treatment populations.
Barton et al. conducted a Level 1 prospective randomized controlled study comparing long cephalomedullary nails with sliding
hips screws in the treatment of unstable intertrochanteric fractures (AO/OTA 31-A2). The authors found no significant difference
in any of the measured variables when comparing the two devices.
Incorrect Answers:
Answer 2: The use of the sliding hip screw has decreased despite equivalence with cephalomedullary nails
Answer 3: Until 2010 Medicare reimbursement was more for cephalomedullary nails.
Answer 4: Intramedullary nails have not been shown to have superior outcomes in multiple studies
Answer 5: Sliding screws have been shown to have worse outcomes for reverse obliquity fractures
28) PREFERRED RESPONSE 3
Figure A shows superior cutout of the lag screw from the sliding hip screw as well as the superior cannulated screw used for an
"antirotation" device.
In the referenced review article by Haidukewych and Berry, salvage of failed treatment of hip fractures in the elderly is limited by
bone quality and comorbidities. They recommend total hip arthroplasty in this instance to restore function, decrease pain, and
limit periods of immobilization. They mention that the major challenges for arthroplasty are: assessing the need for acetabular
resurfacing, selecting the femoral implant, and managing the greater trochanter.
29) PREFERRED RESPONSE 5
The image shows a reverse obliquity intertrochanteric hip fracture.
According to the referenced article by Haidukewych et al, unstable peritrochanteric hip fractures have a worse outcome (failed in
9/16 cases) if treated with a sliding hip screw. Two additional factors that were found to have a strong correlation with
postoperative failure (nonunion, loss of reduction) were poor reduction and poor implant placement. In this study, fixed angle
devices were superior. Intramedullary fixation has the added advantage of a shorter lever arm and less potential for fracture
collapse and limb shortening. The IMN also acts as a medial buttress.

According to Sanders et al, the dynamic condylar screw (DCS) can also be used in subtrochanteric models, but should not be used
if extensive comminution is seen, as they reported a high failure rate with DCS in these fractures if highly comminuted. They
report a 77% overall union rate with this device.
30) PREFERRED RESPONSE 2
Anterior perforation of the femur has been attributed to a simple mismatch in the radius of curvature of implants and the apex
anterior bowed femur.
The radius of curvature is generally smaller (114-120 cm) than many earlier generation femoral nails (up to 300 cm), and the
referenced article by Ostrum et al describes a case series of 3 such patients with subtrochanteric fractures. He noted that the
difference in femoral anteroposterior bow between the bone and the implant is a contributing factor to distal femoral anterior
cortex penetration in intramedullary nailing of subtrochanteric fractures.
Illustration A shows an example of a nail penetrating the anterior femoral cortex.
Illustrations: A

31) PREFERRED RESPONSE 5


Intertrochanteric hip fractures with lateral wall fractures should be treated with an intramedullary device as opposed to a sliding
hip screw, as the intact lateral wall provides a buttress for the proximal fragment facilitating fracture impaction as well as
rotational and varus stability.
Palm et al showed that 22% of patients with a fractured lateral femoral wall underwent reoperation for collapse of fracture
compared to 3% with an intact lateral femoral wall. Interestingly, 74% of the lateral proximal femoral wall fractures were
iatrogenic during the procedure itself.
Gotfried et al reported on 24 patients with postoperative intertrochanteric hip fracture collapse and noted that this complication
followed fracture of the lateral wall in every instance and resulted in a protracted period of disability until fracture healing. They
recommend care when drilling at the base of the lateral wall intraoperatively.
Lindskog et al review the diagnosis, treatment, as well as biomechanical reviews of treatment options for unstable
intertrochanteric hip fractures.
Incorrect Answers:
Answer 1, 2, and 3: No difference in collapse has been shown between long or short intramedullary devices and a sliding hip
screw in stable intertrochanteric hip fractures.
Answer 4: Early postoperative weightbearing is the goal after repair, and no differences have been shown in collapse rates with
different weight bearing protocols.
32) PREFERRED RESPONSE 1

The ASA classification (detailed in Illustration A) was initially developed in 1963 and has been shown to be predictive of postsurgical mortality in hip fracture patients. Basic categories are as follows: 1= normal, healthy; 2= mild systemic disease; 3= severe
systemic disease, not incapacitating; 4= severe incapacitating systemic condition, constant threat to life; 5= moribund patient; 6 =
brain dead, organs being donated.
The study by Richmond et al looked at 836 patients treated for a hip fracture and found that this injury is not associated with
significant excess mortality amongst patients older than age 85. However, in younger patients, those with ASA classifications of 3
or 4 have significant excess mortality following hip fracture that persists up to 2 years after injury.
Illustrations: A

33) PREFERRED RESPONSE 5


The radiographs demonstrate a reverse obliquity intertrochanteric femur fracture. Compared to the more stable intertrochanteric
femur fracture, a reverse oblique intertrochanteric hip fracture is not optimally treated with a sliding hip screw. Compression along
a sliding hip screw is designed to create compression along the plane of the fracture, however in a reverse obliquity fracture
pattern as seen here, shear force is created causing medial displacement of the femoral shaft and screw cutout.
Haidukewych et al showed in their retrospective review of 55 consecutively treated reverse obliquity intertrochanteric fractures,
that patients treated with a sliding hip screw had nearly a 56% failure rate (9/16). The failure rate of patients treated with a blade
plate was only 13%.
Sadowski et al showed in their prospective randomized trial in patients with a reverse obliquity or transverse intertrochanteric
fracture who were randomized to either a 95 degree screw-plate or cephalomedullary nail a much higher failure rate for the platescrew implant. Implant failure was seen in 7/19 patients treated with the 95 degree screw plate and only 1/30 in the intramedullary
nail group. Both articles support the use of a blade plate or cephalomedullary nail for reverse obliquity fractures.
An example of screw cutout and medial displacement is seen in Illustration A.
Illustrations: A

34) PREFERRED RESPONSE 3


Currently, cephalomedullary nails are used widely for reverse obliquity fractures because they limit medialization of the shaft
fragment unlike sliding hip screws.
The Haidukewych et al study quoted demonstrated the superiority of fixed angle devices such as blade-plates or dynamic condylar
screws over the sliding (or dynamic) hip screws. Reverse obliquity intertrochanteric fractures of the femur are recognized as
biomechanically different from standard intertrochanteric fractures. The rate of failure of internal fixation for this fracture pattern
was higher than the rates in most reports of internal fixation of intertrochanteric fractures devices.
35) PREFERRED RESPONSE 2
A two part stable intertrochanteric femur fracture can be treated with a sliding hip screw, with good biomechanical and clinical
results.
The referenced article by Bolhofner et al reviews a series of 69 patients with a sliding hip screw and two hole side plate and notes
that they did not have any failure of the side plate construct.
The referenced article by McLoughlin et al is a biomechanical evaluation of 2 versus 4 hole plates and found that peak load in the
failure test was not found to be statistically different between the two-hole and four-hole designs. In cyclic testing, the two-hole
configuration exhibited statistically smaller fragment migration in both shear and distraction than the four-hole design.
36) PREFERRED RESPONSE 4
The patient's injury is most consistent with a bisphosphonate induced atypical femur fracture.
Atypical subtrochanteric femoral fractures have been identified as a potential complication of long-term bisphosphonate therapy
for the treatment of osteoporosis. Prodromal symptoms of thigh pain are common prior to fracture, and bony failure is usually
associated with low energy mechanisms.
Puhaindran et al. performed a retrospective review of the imaging studies and case notes for patients with skeletal malignant
involvement who received a minimum of twenty-four doses of intravenous bisphosphonates. Patients were classified as having an
atypical subtrochanteric femoral fracture if they had a transverse subtrochanteric fracture following low-energy trauma or an
impending fracture, together with radiographic findings. In the study cohort of 327 patients, four patients developed an atypical
subtrochanteric femoral fracture. All four patients were female, three had breast cancer, and one had myeloma.
Feldman et al. reviewed the imaging presentations on routine radiographs, alternate imaging modalities, and associated pitfalls in
nine atypical femur fractures in six patients. The author states that effects may differ with each bisphosphonate's route of
administration and prolongation of activity, despite discontinuation. The review also discusses the theoretical mechanisms of

bisphosphonates as a class rather than with a specific alendronate association and provides a broader basis for evaluating the
recently observed clinical and radiographic complications.
Figure A shows a transverse subtrochanteric femur fracture. Figure B shows diaphyseal cortical thickening and cortical beaking at
the subtrochanteric area.
Incorrect Answers:
Answer 1: The radiographic findings are not consistent with a fibrous cortical defect.
Answer 2: The radiographic findings are not consistent with the presence of a metastatic lesion.
Answer 3: Subtrochanteric fractures are typically associated with high energy trauma, and this patients mechanism of injury is
atypical.
Answer 5: There is no radiographic or clinical evidence supporting the presence of chronic osteomyelits.
37) PREFERRED RESPONSE 1
In order for a fracture to be successfully treated with tension band principles the bone must be eccentrically loaded, the construct
must be applied on the tensile side, and the opposite cortex must be able to withstand compressive forces. Of the fractures seen in
Figures A-E, the subtrochanteric fracture seen in Figure A best meets the criteria for stabilization according to tension band
principles.
Kinast et al retrospectively compared their results with 95 degree blade plate fixation of subtrochanteric fractures utilizing the
blade plate as a dynamic tension band. They performed either wide exposure of the fracture site with autogenous bone grafting
according to AO technique at the time (group 1), or indirect reduction techniques without bone grafting the medial side as
advocated by Mast et al (group 2). The authors found 100% union rates at six months with indirect reduction techniques without
bone grafting (group 2), and emphasize the key concepts of preservation of the medial soft tissues and intraoperative
pretensioning of the plate.
Illustration A demonstrates the principles of tension band fixation specifically applied to the femur. Within Illustration A, Figure C
shows the correct application of a plate along the lateral cortex to resist tensile forces, along with incorrect application of the plate
along the medial cortex (Figure D) or in a fracture pattern with an absent opposite cortex (Figure E). Illustration B shows an
example of blade plate fixation of a subtrochanteric fracture.
Incorrect Answers:
Answer 2. Figure B demonstrates a comminuted proximal tibia fracture. Although the tibia is eccentrically loaded and an implant
applied to anterior cortex could function as a tension band, the posterior communition would lead to collapse.
Answer 3. Figure C demonstrates a comminuted distal humeral shaft fracture. Again,the comminution prevents application of a
tension band construct
Answer 4: Figure D shows a valgus impacted proximal humerus fracture. Eccentric loading is absent for this type of fracture
Answer 5: Figure E demonstrates a comminuted olecranon fracture with extension distal to the coronoid process. Although tension
band constructs are commonly used for olecranon fractures, the comminution and distal extension of this fracture would prevent
application of a tension band.
Illustrations: A

Illustrations: B

38) PREFERRED RESPONSE 2


The image in Figure A shows an unstable intertrochanteric fracture and the image in Figure B shows perforation of the anterior
cortex of the femur by the intramedullary implant. This complication is due to a mismatch of the curvature of the nail with the
anterior bow of the femur, and was likely caused by a nail of a greater radius of curvature (eg, straighter than the femur).
Egol et al measured the radius of curvature for 474 matched cadaveric femurs and found the average anterior radius of curvature
to be 120cm (+/- 36cm). In contrast, the radii of curvature for the measured intramedullary nails ranged from 186cm to 300cm,
demonstrating that the nails were straighter than the femurs. The authors advocate for a decreased radius of curvature (more
curve) for intramedullary nails, especially larger diameter implants designed for fractures about the hip
Ostrum and Levy present a case series of 3 patients with subtrochanteric fractures who had anterior penetration of the femoral
cortex. They state that the mismatch in femoral bow between the bone and the implant is a contributing factor to distal femoral
anterior cortex penetration in intramedullary nailing of subtrochanteric fractures.
Simonian et al present 4 iatrogenic femoral neck fractures that occured during a series of 315 femoral nails. The authors attempted
to reproduce the iatrogenic fractures with cadaveric femurs and felt that the iatrogenic fractures may be due to a combination of a
valgus femoral neck and impingement from the AO insertion jig used at the time.
Harper and Carson examined 14 cadaveric femurs and intramedullary implants at the time. Similar to Egol et al, they found a
mismatch between the radius of curvature of the femurs and the intramedullary nails.
Illustration A shows the difference between a lesser and greater radius of curvature. Illustration B demonstrates how to calculate

radius of curvature based on an implant with an exaggerated bow. Illustration C shows the anterior bow of a synthetic femoral
model compared with several intramedullary implants.
Illustrations: A

Illustrations:B

Illustrations:C

39) PREFERRED RESPONSE 2


Subtrochanteric fractures will cause a proximal fragment to be flexed, abducted, and externally rotated due to the imbalanced
proximal muscular attachments. The proximal fragment would likely have to be extended, adducted, and internally rotated to
obtain a proper reduction.
Lundy did a review on subtrochanteric fractures. He reviews that these fractures can be effectively stabilized with 95 degrees
plates, femoral reconstruction nails, or trochanteric femoral nails. Although intramedullary nails produce very stable constructs
that are a great treatment option for this fracture, 135 degrees hip screw-plates are not suitable in the treatment of subtrochanteric
femoral fractures due to the high risk of loss of fixation and fracture displacement.
Illustration A depicts the applicable deforming muscle forces.
Illustrations: A

40) PREFERRED RESPONSE 4


The patient is presenting with a reverse obliquity peritrochanteric fracture nonunion, as shown in Figure A. 135-degree
compression plate implants (such as the one used in this patient) are designed to stabilize intertrochanteric femoral fractures, and
they may be unable to resist the deforming forces inherent in subtrochanteric fractures. When this device is used, the distal
fragment often displaces medially and proximally as the fracture settles. The proximal fragment also may rotate on the
compression screw because the plate design allows only for one screw in the proximal fragment. Cephalomedullary nails, such as
that shown in Illustration A, have been shown to offer biomechanical superiority and diminished risk of implant failure when
compared to plating of these injuries.
Lundy provides a review article on the evaluation and treatment of subtrochanteric femur fractures. He states that these fractures
can be effectively stabilized with 95 plates, femoral reconstruction nails, or trochanteric femoral nails with interlocking options.
With regards to plates, he states that a 135 hip screw-plate is not suitable in the treatment of subtrochanteric femoral fractures,
and that use of these implants may result in loss of fixation and fracture displacement.
Menezes et al reviewed 155 consecutive patients who were treated with a proximal femoral nail from 1997 to 2001 to determine
the rate of implant specific complications. They concluded that low rates of femoral shaft fractures and failure of fixation support
the use of the proximal femoral nail for treatment of unstable trochanteric and subtrochanteric fractures.
Robinson et al used the long Gamma nail to treat a consecutive series of 302 local patients who had sustained a subtrochanteric
fracture during low-energy trauma over an 8 year period. They found that trochanteric-entry cephalomedullary nails are associated
with an acceptable rate of perioperative complications and favorable functional outcomes.
Incorrect Answers:
1-Increasing the length of the the 135-degree compression plate to 6-holes will not improve its biomechanical properties in this
fracture pattern.
2-Addition of autograft would not improve the biomechanical stability of the fracture, and is not appropriate during the index
procedure.
3-Long strut allografts are not indicated in the initial treatment of reverse obliquity subtrochanteric fractures.
5-Addition of a de-rotation screw would not change the stability of the fixation construct.
Illustrations: A

41) PREFERRED RESPONSE 1


Figure A demonstrates a displaced subtrochanteric femur fracture with an intact lesser trochanter. The pull of iliopsoas on the
lesser trochanter as well as the intact external rotators and gluteal musculature results in the the proximal fragment being in a
flexed and externally rotated or abducted position (the most common post operative deformity). Reduction manuevers must be
biologically friendly but also counteract the flexion/abduction moment. Lundy's review article discusses evaluation and treatment
of subtrochanteric fractures. The review article details the various implants often used which include 95 degrees plates, femoral
reconstruction nails, or trochanteric femoral nails with interlocking options. Lundy's article discourages the use of the 135 degree
screw and side plate combo due to high failure rates in these fracture patterns. Bedi et al also review treatment of these fractures
and discuss common problems of malunion, nonunion, and implant failure. The article reviews reduction techniques that are soft
tissue friendly, as well as the use of appropriate implants in these fracture types.
42) PREFERRED RESPONSE 4
Based on the references provided, the advantages of the lateral position include: facilitates the retraction of the vastus lateralis,
allows hip flexion to aid reduction, improves access to the proximal segment (easier to get starting point). Disadvantages of the
lateral position include: intraoperative imaging may be more difficult, rotation is more difficult to judge, and lateral positioning
may not be practical in the polytraumatized patient.
Advantages of the supine position include: may help protect a potentially unstable spine, facilitates access to sites other than the
injured femur, shorter setup time, rotational and angulatory deformities may be more easily appreciated. Disadvantages of the
supine position include: starting point localization may be more difficult.
43) PREFERRED RESPONSE 4
The gluteus medius attaches to the greater trochanter, leading to abduction, while the iliopsoas attaches to the lesser trochanter,
leading to flexion. French et al evaluated forty-five Russell-Taylor Type 1B subtrochanteric femoral fractures which were
stabilized using an interlocked cephalomedullary nail. The intraoperative complication rate was 13.5%; and the most frequent
complication was a varus malreduction. The primary reason for this was failure to counteract the muscle forces acting on the
proximal fragment combined with the adducted position of the distal femur during portal creation. This problem can be avoided if
the position of the proximal fragment is evaluated carefully and reduced before guidewire insertion.
44) PREFERRED RESPONSE 1
The most common deformity after antegrade nailing of a subtrochanteric femur fracture is varus and procurvatum (or flexion).
This is caused by the hip abductors and iliopsoas pulling the proximal fragment into abduction and flexion, while the distal
fragment is pulled into adduction from the adductors.
The reference by French et al is a review on 45 patients with subtrochanteric fractures treated with cephalomedullary interlocked
nailing. Based on femoral neck-shaft angle, 61% of the fractures were reduced in at least 5 varus. The authors attributed this
malalignment to failure to counteract muscle forces acting on the proximal fragment, combined with the adducted position of the
distal femur during portal creation.
The reference by Ricci et al is a report of 403 femoral shaft fractures treated with intramedullary nailing. Patients with proximal

femoral shaft fractures were found to have the highest incidence of malalignment. The most common deformity in this group was
varus, followed by procurvatum (or flexion).

60 Days Orthobullets:
Answers Bank
Day 6

1. PREFERRED RESPONSE 2
In the setting of a severe closed head injury such as a bifrontal cerebral contusion with elevated intracranial pressures, external
fixation of a femoral shaft fracture is indicated to limit the risk of intraoperative hypotension and decreased cerebral perfusion
pressure.
Immediate reamed nailing remains the standard treatment for the vast majority of femoral shaft fractures, however patients with
multiple injuries with incomplete resuscitation and patients with severe intracranial trauma may benefit from a damage control
approach with external fixation.
Anglen et al retrospectively reviewed the intracranial pressure (ICP) and cerebral perfusion pressure (CPP) in patients undergoing
femoral nailing. The authors found a significant decrease in intraoperative CPP, especially in those patient undergoing femoral
nailing in the first 24 hours, however they were unable to demonstrate a link between the decreased CPP and poor patient
outcomes.
Pietropaoli et al examined the effects of intraoperative hypotension on patients with blunt head trauma. The authors found that
32% of patients experienced intraoperative hypotension (systolic blood pressure less than 90mm Hg) and those patients with a
hypotensive episode had an 82% mortality and significantly worse outcomes on the Glasgow Outcomes Scale.
McKee et al conducted a retrospective cohort study comparing matched groups of patients with femoral shaft fractures with and
without a closed head injury. In contrast to previous studies, the authors found no significant difference in outcome between the
groups including mortality, hospital length of stay or neuropsychologic testing.
Illustration A shows a femoral shaft fracture treated with external fixation.
Incorrect Answers:
Answer 1, 3-5: Immediate reamed nailing would not change the outcome of any of these injuries

2. PREFERRED RESPONSE 1
Figures A and B show displaced ipsilateral femoral neck and shaft fractures. Fixation with a single implant, such as an antegrade
cephalomedullary device, has been shown to have the highest rate of fracture malreduction with displaced fractures.
Approximately 5% of femoral shaft fractures are accompanied by ipsilateral neck fractures. Fixation methods to stabilize these
fractures may be accomplished by using a single implant or two separate implants. Single implant techniques are thought to
reduce operative time and blood loss by simultaneously fixing nondisplaced fractures. With displaced fractures, single implant
techniques have been shown to have a higher rate of malreduction of at least one of the two fractures. Two separate implant
devices are recommended in these scenarios.
Bedi et al. examined a retrospective cohort of 40 patients with ipsilateral femoral neck and shaft fractures. They showed that using
a single cephalomedullary device for fixation of both femoral shaft and neck fractures led to a significantly higher rate of fracture
malreduction (3 of 9), in comparison to a staged, two implant strategy (0 of 28) (P = 0.01).

Peljovich et al. reviewed the presentation and management of ipsilateral femoral neck and shaft fractures. To reduce complications
of AVN, malunion and nonunion, they suggest obtaining anatomic reduction and rigid fixation of the femoral neck fracture first
with 3 cannulated screws, blade plate, or sliding hip screw. The shaft fracture can then be reduced and stabilized with either
retrograde intramedullary femoral nailing or plating.
Incorrect Answers:
Answers 2,3,4,5: Fixation of femoral neck and shaft fractures using two separate implant devices has a lower rate of malreduction.
No study to date has conclusively demonstrated superiority of any particular combination of devices in long-term studies.

3. PREFERRED RESPONSE 5
This patient has an external rotation deformity of 40 of the distal fragment of the right femur. Correction would entail rotating the
distal fragment internally by 40.
The right femoral neck (RFN) is internally rotated (IR) by 13 to the horizontal (IR13). The left femoral neck (LFN) is externally
rotated (ER) by 13 to the horizontal (ER13). The right distal fragment (RDF) is ER17. The left distal fragment (LDF) is ER3.
Bringing both femoral necks to ER0/IR0 gives the absolute amount of rotation of the distal fragment to the horizontal. To do this,
the RFN has to EXTERNALLY rotate 13 and the LFN has to INTERNALLY rotate 13. Thus, RDF has a total of
ER(13+17)=ER30, and LDF has IR(13-3)=IR10 to the horizontal. To correct the RDF from ER30 to IR10, internal rotation of 40
must occur.
Malrotation is the most common cause of limb deformity after nailing. To avoid this, patients should be examined for rotation and
limb length after insertion of static interlocks, before leaving the operating room. Correction is easier to perform prior to fracture
union. Drill-hole cutout is possible if correction<20 if the previous distal locking site is to be used because of the proximity of the
new interlock to the previous interlock. To avoid this, (1) use alternative locking holes or the dynamic locking slot, or (2) advance
or retract the nail to avoid previous locking sites.
Lindsey et al. reviewed rotational malalignment after femoral nailing. The incidence of rotational malalignment was 27.6%.
Normal femoral neck anteversion (angle of the femoral neck relative to the transverse axis through the femoral condyles) is 1113. Some patients have up to 15 difference in version between limbs.
Jaarsma et al. reviewed rotational malalignment after nailing of 76 femoral fractures. The incidence of rotational malalignment
>=15 was 28%. Patients with an ER malalignment (n = 12) had more functional problems than patients with a IR malalignment
(n = 9).
Incorrect Answers:
Answer 1: This answer would be correct if LFN was IR13, and LDF was IR3, giving absolute LDF rotation of ER10. Then to
correct ER30 (right) to ER10 (left), internal rotation of 20 would be needed.
Answers 2, 4: The RDF is more externally rotated. Correction must involve internal rotation.
Answer 3: This answer would be correct if LFN was IR13, giving absolute LDF rotation of ER16. Then to correct ER30 (right) to
ER16 (left), internal rotation of 14 must occur.

4. PREFERRED RESPONSE 4
This patient has an external rotation deformity of 40 of the distal fragment of the right femur. Correction would entail rotating the
distal fragment internally by 40.
The right femoral neck (RFN) is internally rotated (IR) by 13 to the horizontal (IR13). The left femoral neck (LFN) is externally
rotated (ER) by 13 to the horizontal (ER13). The right distal fragment (RDF) is ER17. The left distal fragment (LDF) is ER3.
Bringing both femoral necks to ER0/IR0 gives the absolute amount of rotation of the distal fragment to the horizontal. To do this,
the RFN has to EXTERNALLY rotate 13 and the LFN has to INTERNALLY rotate 13. Thus, RDF has a total of
ER(13+17)=ER30, and LDF has IR(13-3)=IR10 to the horizontal. To correct the RDF from ER30 to IR10, internal rotation of 40
must occur.

Malrotation is the most common cause of limb deformity after nailing. To avoid this, patients should be examined for rotation and
limb length after insertion of static interlocks, before leaving the operating room. Correction is easier to perform prior to fracture
union. Drill-hole cutout is possible if correction<20 if the previous distal locking site is to be used because of the proximity of the
new interlock to the previous interlock. To avoid this, (1) use alternative locking holes or the dynamic locking slot, or (2) advance
or retract the nail to avoid previous locking sites.
Lindsey et al. reviewed rotational malalignment after femoral nailing. The incidence of rotational malalignment was 27.6%.
Normal femoral neck anteversion (angle of the femoral neck relative to the transverse axis through the femoral condyles) is 1113. Some patients have up to 15 difference in version between limbs.
Jaarsma et al. reviewed rotational malalignment after nailing of 76 femoral fractures. The incidence of rotational malalignment
>=15 was 28%. Patients with an ER malalignment (n = 12) had more functional problems than patients with a IR malalignment
(n = 9).
Incorrect Answers:
Answer 1: This answer would be correct if LFN was IR13, and LDF was IR3, giving absolute LDF rotation of ER10. Then to
correct ER30 (right) to ER10 (left), internal rotation of 20 would be needed.
Answers 2, 4: The RDF is more externally rotated. Correction must involve internal rotation.
Answer 3: This answer would be correct if LFN was IR13, giving absolute LDF rotation of ER16. Then to correct ER30 (right) to
ER16 (left), internal rotation of 14 must occur.

5. PREFERRED RESPONSE 1
Comminuted femoral shaft fractures treated with statically locked intramedullary nails of appropriate diameter can be treated with
immediate weight-bearing, with little risk of nail/screw breakage or deformity. Immediate range of motion and weight-bearing can
be extremely beneficial to short-term patient outcomes, especially in polytrauma patients.
Brumback et al.(1988) reviewed 133 dynamically locked femoral nails and report that 10.5% lost fixation and/or reduction
postoperatively. They recommend reviewing high-quality radiographs to determine fracture characteristics, and note that dynamic
fixation only be considered for transverse fracture patterns.
Brumback et al.(1988) performed a prospective series of 97 patients with statically locked femoral nails, and they report that 98%
of these went on to successful union without additional procedures, and the 2% with nonunions were successfully treated with
later conversion to dynamic interlocking. They also found that no implant failure or deformity occurred with early walking or
weight-bearing.
Brumback et al.(1999) reviewed the biomechanics of immediate weight-bearing after statically locked intramedullary nails are
used in a segmental femur model, and they found that it would be safe. They then looked at immediate weight-bearing of these
fractures after statically locked nail insertion in a series of 35 patients, and found no loss of reduction or implant failure with
immediate weight-bearing.
Figures A and B show a comminuted, segmental femoral shaft fracture, while Figures C and D show the immediate postoperative
radiographs of this patient after intramedullary nailing.
Incorrect Answers:
Answers 2-5: Delay in weight-bearing is not required for this injury pattern, if treated with an appropriate diameter statically
locked intramedullary nail.

6. PREFERRED RESPONSE 4
Interlocking of intramedullary nails using fluoroscopy requires attention to detail. A true lateral of the intramedullary nail is
present when "perfect circle" views of interlocking holes are present. Once perfect circles are obtained, the drill can be advanced
parallel to the fluoroscopic beam.
Knowledge of the implications of the appearance of the interlocking holes when "perfect circles" are not present can be helpful in
minimizing the number of manipulations and fluoroscopic exposure. Widening of the interlocking hole in the proximal-distal

direction (as is seen in this case) signifies the need for an adjustment in the abduction/adduction plane. Similarly, widening of the
interlocking hole in the anterior-posterior plane signifies the need for an adjustment in the internal/external rotation plane (Answer
3).
Raising or lowering the leg (Answer A and B) should not have major effects of the appearance of the interlocking hole. Similarly,
magnification of the C-arm (Answer D) will not affect the appearance. Internal/external rotation will result in widening in the
anterior-posterior plane (as this dimension is currently adequate). Abduction or adduction will result in creating "perfect circles"
(Answer 4). The position of the leg may hint to which of these is correct. If further widening occurs in the proximal-distal
direction, the opposite maneuver will correct to the proper position.

7. PREFERRED RESPONSE 1
Computer-assisted navigation has been shown to reduce radiation exposure for surgeons when performing interlocking of
medullary nails compared to free-hand technique.
Ricci et al compared two fluoroscopic navigation tracking technologies, optical and electromagnetic versus standard freehand
fluoroscopic targeting, in a standardized foam block model for placement of interlocking screws. They found that fluoroscopy
time (seconds) and number of fluoroscopy images were significantly less when using the computer-guided systems than for
freehand-unguided insertion. Average distance of pin placement from the target in the foam blocks was significantly greater for
controls than for each of the navigated systems.
Suhm et al performed a prospective controlled clinical study to compare fluoroscopic guidance with fluoroscopy-based surgical
navigation for distal locking of intramedullary implants. The surgical navigation group showed increased procedure time, but
equivalent precision with reduced radiation exposure. There was no significant difference in the technical reliability between both
groups.

8. PREFERRED RESPONSE 2
Branches of the deep femoral artery and femoral nerve are most at risk during placement of anterior to posterior interlocking
screws below the level of the lesser trochanter as seen in Figure B.
Riina et al performed a cadaveric study examining the neurovascular structures at risk during proximal interlocking of retrograde
femoral nails and found that the first division of the femoral nerve crossed the femur on average approximately 4cm distal to the
piriformis fossa. In addition, the femoral artery was closest to the medial aspect of the femur 4cm distal to the lesser trochanter.
The authors recommend placement of interlocking screws proximal to the lesser trochanter to minimize risk of iatrogenic
neurovascular injury.
Brown et al attempt to define relative safe zones (RSZs) for placement of both lateral-medial and anterior-posterior interlocking
screws of femoral nails in patients with acetabular fractures with reference to the location of the femoral neurvascular structures at
the level of the lesser trochanter. The authors found that the available safe zone for placement of anterior-posterior screws
decreased more than 50% in the setting of hematoma from acetabular fracture, and they recommend blunt dissection to bone and
use of a single incison between the proximal interlocking holes to minimize risk to these structures.
Handolin et al present two case reports of injury to the deep femoral artery with proximal interlocking screws during retrograde
nailing. Based on their experiences, the authors advocate for blunt dissection to bone to avoid entrapment of critical structures
when places these screws.
Illustration A shows a cadaveric dissection and illustration of the numerous branches of the deep femoral artery and femoral nerve
as they cross the femur from medial to lateral on the anterior surface just below lesser trochanter. The view is of the anterior thigh
from the medial aspect. The sartorius has been cut and reflected and the retractors are beneath the rectus femoris.

9. PREFERRED RESPONSE 3
Femoral neck fractures are seen less than 10% of the time with femoral shaft fractures, but they are frequently missed on initial
evaluation. When present, the pattern is typically nondisplaced, vertical, and basicervical.
The review article by Peljovich and Patterson note that the femoral shaft component of the combined injury is typically in the
middle third and is often comminuted.
The article by Tornetta et al reports that they reduced the delay in diagnosis of concomitant femoral neck fractures by 91% by
instituting a protocol that included: dedicated AP internal rotation plain radiograph, a fine (2-mm) cut CT scan through the femoral
neck, an intraoperative fluoroscopic lateral radiograph prior to fixation, as well as postoperative AP and lateral radiographs of the
hip in the operating room prior to awakening the patient.
The article by Wiss et al noted that 18% of their ipsilateral femoral neck/shaft cohort developed a symptomatic varus nonunion
requiring a valgus osteotomy. It is significant to note that in this study, the shaft fractures were fixed prior to definitive neck
stabilization, and the review article by Peljovich emphasizes that the neck fracture should be treated first and the shaft fracture
second.

10.

PREFERRED RESPONSE 4
Femoral malrotation after intramedullary nailing is unfortunately a possibility with either antegrade or retrograde nailing
techniques.
According to a review by Hufner et al, malrotation (internal or external >15 degrees) was seen in 22% of their patients via CT
scan after intramedullary nailing. There was a significant difference depending on the time of surgery, with significantly more
malrotation during the night shift. Increased fracture comminution also significantly increased malrotation rates. No significant
increases were seen with the other answers listed above.

11.
PREFERRED RESPONSE 2
Ipsilateral femoral neck fractures are seen in 1-9% of femoral shaft fractures and the femoral neck must be properly imaged either
preoperatively or intraoperatively in any patient with a femoral shaft fracture. Dedicated hip films, possibly including an internal
rotation AP, should be obtained before entering the OR.
Daffner et al reported that in 11 of 20 cases of combined femoral shaft and neck fractures, the initial preoperative radiographs did
not demonstrate the femoral neck fracture. Intraoperative fluoroscopy should also be used to evaluate for a femoral neck fracture
both before (to evaluate for unrecognized fx) and after (to evaluate for iatrogenic fx) IM nailing.
Tornetta et al also describe using preoperative CT scans to evaluate for a femoral neck fracture and found that they were able to
reduce the number of missed ipsilateral femoral neck fractures.

12.

PREFERRED RESPONSE 2

Figure A shows a femoral shaft fracture treated with an antegrade femoral nail. Long term deficits are weakness with knee
extension (quadriceps) and hip abduction (glutei muscles).
The referenced study by Kapp et al noted long term quadriceps weakness as well as decreased bone mineral density in the femur
(femoral neck by 9%, the lateral cortex by 20% and the medial cortex by 13%). It is unclear whether this is due to the injury,
treatment, or a combination of both.
The second referenced study by Archdeacon et al also noted weakness in hip abduction, which showed time dependent
improvement. He reports that increased early ipsilateral trunk lean is associated with worse recovery of abduction strength.

13.
PREFERRED RESPONSE 5
In the referenced study by Ricci et al, antegrade femoral nailing was shown to have an increased rate of hip pain as compared to
retrograde femoral nailing, while having a similar rate of union, time to union, rate of malalignment, and operative time. Hip pain
was signficantly higher in the antegrade nailing group, while knee pain was significantly greater in the retrograde group.
The referenced study by Winquist et al noted a 99.1% union rate with intramedullary nailing.
The referenced study by Moed et al noted a 6% nonunion rate in non-reamed retrograde femoral nailing with nail dynamization at
6-12 weeks and early weightbearing.

14.

PREFERRED RESPONSE 5
According to the study by Egol et al, the average femoral anterior radius of curvature was 120 cm (+/- 36 cm), and currently
available femoral nails have a greater radius of curvature (i.e. more straight). This mismatch has been shown to lead to an
increased risk of perforation of the anterior distal femur as the nail is impacted into the canal.
The referenced study by Tencer et al noted an increased risk of iatrogenic femoral fracture with anterior starting point >6mm from
the anatomic axis. They recommend starting in line with the femoral axis, or just a few millimeters anterior in order to minimize
this risk.
Illustration A depicts anterior femoral cortex penetration secondary to nail/femur radius of curvature mismatch.
Illustrations: A

15.
PREFERRED RESPONSE 5
Reamed intramedullary femoral nailing is associated with a higher rate of union than nonreamed femoral nailing.
The reference by the Canadian group randomized 224 patients to reamed vs. unreamed femoral nails and found that the relative
risk of nonunion was 4.5x greater without reaming, and nonunion was also greater with the use of a small-diameter nail.
The referenced article by Tornetta et al randomized 81 patients to reamed or unreamed nails and found more intraoperative
technical complications in the group without reaming. There was no statistical difference in OR time, transfusion requirement or
pulmonary complications between the groups. This study showed the overall union rate was similar but when they selected out
distal femur fractures, the reamed group healed faster.
The reference by Brumback et al is a review of reamed v. nonreamed nailing, with discussions of reaming techniques and the
importance of proper reamer technology and usage.

16.
PREFERRED RESPONSE 4
Patients with retrograde femoral nails commonly have knee pain, while antegrade nails commonly have hip pain, abductor
weakness and heterotopic ossification of the abductors.
Ostrums randomized prospective study of 100 patients with reamed femoral nails found 22% of antegrade nail patients had
proximal hip pain, weak hip abductors or trendelenburg gait. No significant difference was found in set-up time, operative time,
knee motion or pain, or infection rates.
Ricci performed a retrospective study of 293 fractures and found that the antegrade femoral nail group had more hip pain (10% vs
4%) and the retrograde nail group had more knee pain (36% vs 9%). There was no difference in healing, malunion, non-union or
other complications.
Tornetta performed a randomized controlled comparison of 69 femur fractures and found more problems of length and rotation
using a retrograde nailing. There was no difference in time to union, operating time, blood loss, complications, size of nail or
reamer, or transfusion requirements.

17.
PREFERRED RESPONSE 1
Heterotopic ossification (HO) prophylaxis with indomethacin has been shown to increase the risk of long-bone nonunion.
Indomethacin therapy has been shown to be an effective means of preventing HO formation, however literature has shown that it
increases the risk of long bone and acetabular nonunion. Indomethacin works primarily by inhibiting IGF-1, which is a different
mechanism from other NSAID's which typically inhibit the COX enzymes. IGF-1 is important for bone healing, and its inhibition
may be a risk factor for delayed bone healing.
Burd et al performed a study to determine if patients with an acetabular fracture, who received indomethacin for prophylaxis
against HO, were at risk of delayed healing or nonunion of any associated fractures of long bones. The study group consisted of
112 patients who had sustained at least one concomitant fracture of a long bone; of which 36 needed no prophylaxis, 38 received
focal radiation and 38 received indomethacin. When comparing patients who received indomethacin with those who did not, a
significant difference was noted in the rate of long bone nonunion (26% vs 7%).
Jordan et al performed a study to document the efficacy of variable treatment durations with indomethacin prophylaxis for HO
and its effect on union of the PW in operatively treated acetabular fractures. Patients were randomly assigned to one of four
treatment groups: (1) placebo for 6 weeks, (2) 3 days of indomethacin followed by placebo for a total of 6 weeks, (3) 1 week of
indomethacin followed by 5 weeks of placebo, and (4) 6 weeks of indomethacin and followed for 1 year. The authors concluded
that the use of prophylactic postoperative indomethacin increases the incidence of symptomatic nonunion of the PW as assessed
by CT scan and pain VAS.
Incorrect Answers:
2-Based on the Jordan et al reference, indomethacin increases the risk of posterior wall nonunion
3-There is no evidence that treatment with indomethacin decreases time to union
4-There is no definitive evidence that indomethacin is superior to radiation in the prevention of HO. Recent data actually is in
favor of radiation treatment both to prevent nonunion, and its superiority in preventing HO formation.
5-Indomethacin increases the risk of nonunion, which would therefore increase the need for re-operation.
18. PREFERRED RESPONSE 1
Usage of a piriformis (straight) nail through a greater trochanteric entry portal will bring the fracture into varus, as the greater
trochanteric entry site's axis is lateral to the femoral shaft, and advancement of the nail causes the two axes to become colinear,
leading to varus. The referenced study by Ostrum notes that usage of a greater trochanteric starting point is safe in obese patients;
he recommends usage of a larger incision and maximum leg adduction.
The referenced study by Winquist et al is a classic review of femoral nailing, and emphasizes the importance of starting point
selection and fracture reduction to maximize clinical outcomes (99.1% union rate in their series of 520 patients).

19. PREFERRED RESPONSE 1


The greatest amount of injury to hip abductor musculature is seen with piriformis starting points as compared to the other options
listed above. Increased rate of injury to the piriformis tendon, medial femoral circumflex artery branches, gluteus minimus, and
superior gluteal nerve branches are noted with the piriformis starting site. Increased injury to the gluteus medius is seen with a
greater trochanteric starting point.
The referenced study by Dora et al noted increased injury to the piriformis tendon with a piriformis starting point (as compared to
a more lateral insertion site).
The classic referenced article by Johnson et al notes that anterior placement of the starting point >6mm over the recommended
start leads to increased hoop stresses and possible burst-type fractures.
The classic referenced study by Winquist et al reviewed their series of 520 femur fractures treated by antegrade nailing; they
report a 99.1% union rate.
Illustrations: A

20. PREFERRED RESPONSE 3


Ipsilateral femoral neck and shaft fractures occur in high energy injuries, with a reported incidence of 2.5-9%. The diagnosis of
neck fracture is delayed in 19%-31% of patients. The neck fracture line is almost vertical and nondisplaced, or minimally
displaced in 26% to 59% of cases. Two major complications, AVN of the femoral head and non-union of the neck result from neck
fracture; therefore, it takes precedence. Dedicated protocols of femoral neck fracture detection have been described and include:
dedicated preoperative hip radiographs, pelvic/hip CT scan, and intraoperative fluoroscopic examination.
The referenced article by Bennet et al is a case series of 42 patients with ipsilateral femoral neck-shaft fractures; 31% of the neck
fractures were initially missed but none developed femoral head AVN.
The referenced article by Wolinsky et al is an excellent review of this injury. They also report that evaluation of these fractures
with plain radiographs can be nondiagnostic 19% to 50% of the time when the majority of the fractures are minimally or
nondisplaced.
Illustration A depicts a radiograph of an ipsilateral femoral shaft and vertical, minimally displaced femoral neck fracture.
Illustrations: A

21. PREFERRED RESPONSE 2


Figure A shows a proximal (supraisthmal) femoral shaft fracture.

The referenced article by Stephen et al is a randomized controlled trial between manual traction and fracture-table traction for the
reduction and nailing of femoral shaft fractures in terms of quality of the reduction, operative time, complications, and functional
status of the patient in eighty-seven patients. Internal malrotation was significantly more common when the fracture table had
been used: twelve (29%) of the forty-two femora were internally rotated by >10 compared with three (7%) of the forty-five
treated with manual traction. Mean operative time was also less in the manual traction group.
The referenced study by Wolinsky et al also found that use of a traction table significantly increased the anesthesia time, total
operating room time, prep and drape time, and overall surgical time as compared to manual traction.
22. PREFERRED RESPONSE 4
Hypoxia and hypotension are associated with lower GCS scores in polytrauma patients with major head injuries, but whether early
fracture fixation adversely affect CNS outcomes had been controversial. New studies, however, have found no association
between early surgery and decreased discharge GCS scores.
The referenced study by Scalea et al reviewed 171 patients with pelvic or lower extremity fractures and head injuries; they showed
no difference in CNS outcomes or mortality in patients who underwent early fixation.
The second reference by Brundage et al showed improved outcomes (including high GCS scores at time of discharge) in those
who had early fixation of femoral shaft fractures in the head-injured patient.
The last referenced study by Jaicks et al found a lower discharge GCS in the early fracture fixation group compared with the late
group. However, they also found that this association was due to hypoxemia and hypotension.
23. PREFERRED RESPONSE 2
Figure A shows a femoral shaft fracture, which can be appropriately treated with an intramedullary nail.
The referenced article by Ostrum et al reported that retrograde nailing had an increased rate of symptomatic distal interlocking
screws, an increased rate of need for dynamization, longer union time, and less thigh pain than antegrade nailing. Union rate and
knee range of motion were not significantly different between these two techniques.
Subsequent studies have found that union time does not significantly differ between antegrade and retrograde nailing.
24. PREFERRED RESPONSE 4
Figure A shows a complex pelvic ring injury, while Figures B and C show bilateral femur fractures. Appropriate treatment of an
unstable, head-injured patient with the above injuries includes prompt, judicious external fixation of his bilateral femoral fractures
and pelvic ring injury. The advantages of early fracture fixation in patients with multiple injuries have been challenged recently,
particularly in patients with head injury. External fixation (EF) has been used to stabilize pelvic fractures after multiple injury. It
potentially offers similar benefits to intramedullary nail (IMN) in long-bone fractures and may obviate some of the risks. EF is a
viable alternative to attain temporary rigid stabilization in patients with multiple injuries. It is rapid, causes negligible blood loss,
and can be followed safely by IMN when the patient is stabilized.
The referenced article by Scalea et al found that external fixation for femur fractures is a viable alternative to attain temporary
rigid stabilization in patients with multiple injuries.
25. PREFERRED RESPONSE 5
In multiple studies, femoral intramedullary reaming debris has been shown to have similar biochemical characteristics as iliac
crest autograft. Intramedullary reaming products have osteogenic potential with viable cells while BMP's are osteoinductive
cytokines.
Hoegel et al found that the reamings had alkaline phospatase activity, indicating living osteoblasts. The amount of activity was
independent of the reamer sizes and reamer design.

Frolke et al concluded that reaming debris supports callus building (healing) as much as conventional iliac crest bone grafting in
an animal fracture gap model.
26. PREFERRED RESPONSE 1
Figure A shows an intertrochanteric fracture treated with a cephalomedullary device. A starting point slightly anterior to the
piriformis fossa (starting point for standard antegrade femoral nail) has the benefit of improved placement of screws through the
nail and into the femoral head. This is due to the fact that the hip is anteverted and the femoral neck arises from the anterior
portion of the proximal femur. Therefore, by moving the nail anterior, that will increase the distance between the head screw and
the posterior cortex of the neck and lead to a straight shot into the center of the femoral head.
Johnson et al investigated the effect of starting hole position, fracture component length, reamed diameter, and nail type on the
potential for femoral bursting and fracture instability. They found the most significant factor in the proximal femoral component
was found to be the position of the starting hole. They found excessive anterior displacement greater than 6 mm from the neutral
axis of the medullary canal consistently caused high hoop stresses at the level of the fracture, which can increase the possibility of
iatrogenic fracture. Posterior starting points increase the risk of possible distal femur anterior cortex impingement/fracture.
Ostrum et al showed that lateral starting points should be avoided in order to avoid varus reduction when using a trochanteric
antegrade nail in subtrochanteric fractures. They recommended a slightly medial starting point.
27. PREFERRED RESPONSE 4
The patient is hemodynamically stable, has no other injuries, and is medically cleared for the operating room. Therefore, there is
no need for damage control fixation.
Ostrum et al conducted a review of 20 patients treated by percutaneous stabilization for ipsilateral fractures of the femur and tibial
shafts. All patients were treated with a retrograde femoral intramedullary nail and a small diameter tibial intramedullary nail
through a 4-cm medial parapatellar tendon incision. Six of the tibial shaft fractures required revision surgery, and no patients
reported signs or symptoms of knee pain. Ostrum concluded that although this is an excellent treatment option for patients with
ipsilateral femoral and tibial shaft fractures, the tibial fracture complication rates remain high.
Franklin et al reviewed 38 cases of open ankle fractures that had been treated with immediate splinting, antibiotics, debridement,
and internal fixation. They found that all of the fractures united, but three patients required subsequent ankle fusion because of
cartilage damage noted at the initial operation. Of the thirty-five ankles with complete follow-up, the functional result was
excellent in twenty-six and fair or poor in nine.
28. PREFERRED RESPONSE 4
Usage of an anterior starting point that is too anterior leads to creation of significant hoop stresses in the proximal segment,
potentially leading to iatrogenic fracture of the proximal segment. The referenced study by Johnson et al reviews the topic of
femoral bursting and he notes that even shifting 6mm too far anteriorly can lead to proximal femoral fracture creation. He also
reported that overreaming the canal by at least 0.5mm diameter is necessary to decrease hoop stresses throughout the femur, likely
due to a mismatch in the radius of curvature of the femur and intramedullary nail.
29. PREFERRED RESPONSE 3
Bilateral femur fractures have not been shown to have increased rates of rotational deformity. They have been shown to have
increased rates of initial hypotension, mortality, open skull fractures, and pelvic fractures.
Due to their high-energy nature, bilateral femur fractures have increased rates of initial hypotension or hemodynamic instability,
mortality, head injuries, abdominal injuries, pulmonary injuries, and other orthopaedic injuries.
Copeland et al. performed a retrospective analysis using their trauma registry data on consecutive blunt trauma patients with
unilateral (800 patients) or bilateral (85 patients) femoral shaft fractures. Patients with bilateral femoral fractures had a
significantly higher Injury Severity Score (30.2 versus 24.5, p < 0.05) and higher mortality rate (25.9 vs 11.7%, p < 0.014) than
patients with unilateral femoral fractures. Bilateral fracture patients also had significantly more closed head injuries, open skull

fractures, intra-abdominal injuries requiring surgical intervention, and pelvic fractures. Regression analysis of variables evident on
admission revealed a significant correlation between bilateral femoral fractures and death; however, other factors (shock, closed
head injury, and thoracic injury) had much stronger correlations with mortality.
Kobbe et al. also reviewed their trauma registry data on 776 patients with unilateral and 118 patients with bilateral femoral shaft
fractures. They found that bilateral femur fracture patients has a higher ISS score, higher incidence of pulmonary failure and
multiple organ failure, and higher mortality. They also noted that patients with bilateral femoral shaft fractures have significantly
more often severe abdominal injuries as well as severe blood loss which may account for the increased mortality rate.
Incorrect Answers:
Answer 1: Increased rates of hypotension upon admission are seen in the bilateral group.
Answer 2: Increased mortality rates have been reported in bilateral femur patients.
Answer 4: Increased rates of open and closed head injuries are noted in bilateral femur patients.
Answer 5: Increased rates of pelvic and other orthopaedic injuries are reported in the bilateral group.
30. PREFERRED RESPONSE 2
Implant C (locking compression plate, LCP) affords better control of coronal plane fractures than Implant A (95-degree angled
blade plate, ABP) and Implant B (dynamic condylar screw, DCS).
The LCP allows for better control of coronal plane and multi-fragmented fractures because the multiple locking screws at the
distal end secure the plate at multiple points and allow capture of fracture fragments in different planes. Newer polyaxial locking
plates have even greater versatility in screw positioning.
Vallier et al. reviewed their experience with the ABP and LCP in distal femur fixation. They note that complications and
secondary procedures (treatment of infection, nonunion, malunion, prominent implant removal) were more frequent in LCP than
ABP patients.
Gwathmey et al. reviewed the fixation of distal femoral fractures. They state that the LCP is biomechanically superior to the ABP
in cyclic loading and ultimate strength. However, the LCP has less fixation strength in torsional loading.
Figure A shows a 95-degree angled blade plate. Figure B shows a dynamic condylar screw. Figure C shows a locking compression
plate. Illustration A shows a coronal plane fracture (Hoffa fracture, OTA 33-B3).
Incorrect Answers:
Answer 1: Implant C (LCP) is better able to control fractures with a small distal segment than Implants A (ABP) and B (DCS).
Answer 3: Insertion of the lag screw component of Implant B (DCS) requires removal of a greater amount of bone than Implant A
(ABP) and Implant C (LCP).
Answer 4: Implant A (ABP) has greater subsidence and lower load to failure compared with Implant C (LCP).
Answer 5: Implant A (ABP) demonstrates greater fixation strength in torsional loading compared with Implant C (LCP).

31. PREFERRED RESPONSE 1


Figure A illustrates a comminuted distal femur fracture. AP fluoroscopic imaging with the leg in 30 degrees of internal rotation is
important to prevent intercondylar screw prominence.
Hardware irritation is a common post-operative complication of distal femoral plate fixation. Two common sites of pain are
laterally where the iliotibial band is in contact with the plate, and medially where intercondylar screws may penetrate the cortex if
they are of inappropriate length. Iliotibial irritation most commonly presents with activities requiring knee flexion and extension.
It is important to remember that the lateral metaphysis of the distal femur is angulated 10 degrees from the sagittal plane, and the
medial metaphysis is angulated 25 degrees from the sagittal plane. Therefore, if a straight AP view is obtained, a distal screw can
appear to be inside the bone even if it is too long. In order to assess the exact length of the screw, one must obtain an AP view with
30 internal rotation of the lower extremity.
Gwathmey et al discuss distal femoral fractures in their review article. They state that the goal of surgical management is to
promote early knee motion while restoring the articular surface, maintaining limb length and alignment, and preserving the soft-

tissue envelope with a durable fixation that allows functional recovery during bone healing. They describe a variety of surgical
exposures, techniques, and implants developed to treat these injuries, including intramedullary nailing, screw fixation, and
periarticular locked plating, possibly augmented with bone fillers.
Illustration A demonstrates the sagittal plane angulation of the medial and lateral cortex of the distal femur. Illustration B shows a
knee in external rotation, with the intercondylar screw appearing to be of appropriate length. Illustration C shows the knee in
internal rotation, which indicates that the screw is penetrating the medial cortex.
Illustrations: A

32. PREFERRED RESPONSE 4


The most common variation of a Hoffa fracture is a coronal fracture of the lateral femoral condyle. The most appropriate screw
placement of the above answer choices in the treatment of the most common Hoffa fracture variant would be anterior to posterior
screws across the lateral condyle for fixation.
Hoffa fractures are coronally oriented fractures of the femoral condyles, with most occurring in the lateral condyle. They are
commonly associated with high-energy fractures of the distal femur and can often be overlooked during the assessment and
treatment of distal femur fractures. Hoffa fractures are best evaluated using CT scans.
Nork et al. studied the association of supracondylar-intercondylar distal femoral fractures and coronal plane fractures. Of 202
supracondylar-intercondylar distal femoral fractures, they found coronal plane fractures were diagnosed in 38%. A coronal
fracture of the lateral femoral condyle was involved more frequently than the medial condyle. Eighty-five percent of these coronal
fractures involved a single lateral femoral condyle.
Holmes et al. looked at five cases of coronal fractures of the femoral condyle. All cases received open reduction and internal
fixation with lag screws through a formal parapatellar approach. They reported good results with all fractures healing within 12
weeks without complications with final range of motion at least 0 degrees to 115 degrees.
Illustration A shows sagittal and axial CT scan cuts showing a Hoffa fracture of the lateral femoral condyle. Illustration B shows
multiple anterior to posterior, and posterior to anterior oriented screws for ORIF of the Hoffa fracture.
Illustrations: A

33. PREFERRED RESPONSE 5


The injury shown in Figures A-D represents a comminuted metaphyseal distal femur fracture with a sagittally oriented intraarticular split in osteoporotic bone. Because of the intra-articular nature of this injury, the best fixation construct for treatment of
this fracture in an otherwise healthy and active patient is lag screw fixation followed by locked plate application.
Egol et al performed a systematic review of the literature to compare and contrast the function and roles of conventional unlocked
plates to locked plates in fracture fixation. They concluded that locked plates may increasingly be indicated for indirect fracture
reduction, diaphyseal/metaphyseal fractures in osteoporotic bone, bridging severely comminuted fractures, and the plating of
fractures where anatomical constraints prevent plating on the tension side of the bone.
Perren et al discuss the treatment of osteoporotic fractures in the elderly population. They state that in this group of patients,
plating with locked screws improve the biology and the mechanics of internal fixation. Furthermore, when this fixation method is
used as an 'internal ex fix' (bridging construct) it may stimulate early callus formation because of the inherent flexibility of the
construct.
Illustrations A and B show intraoperative fluoroscopic images during fracture fixation. Illustration C shows an AP radiograph of
the distal femur 3 months after fixation with a locked plate construct.

34. PREFERRED RESPONSE 2


Figure B demonstrates an isolated medial femoral condyle fracture. This is an AO B type (partial articular fracture). This fracture
is best treated with open reduction internal fixation through a medial approach, with lag screw and buttress plate fixation.
The supracondylar distal femur fractures illustrated in Figures A,C,D, and E can be treated with ORIF with a fixed-angle device
such as lateral locked plating. Non-locked plating for type C (complete articular) distal femur fractures has been associated with
varus malalignment.
35. PREFERRED RESPONSE 4
Oligotrophic femoral supracondylar nonunions have been shown to be best treated with open reduction and plating (revision if
previous surgery) and usage of autologous bone grafting.

The first study by Bellabarba et al is a case series of 100% union rate (n=19) of supracondylar femoral nonunions treated with
revision ORIF and autografting.
The second study by Chapman et al is a case series of 100% union rate (n=18) of supracondylar nonunions treated with single or
double plating and autograft.
The referenced study by Rodriguez et al is an excellent review of the general principles and concepts of nonunions and their
treatment.
36. PREFERRED RESPONSE 4
The "Hoffa fracture" is a coronal plane fracture of the femoral condyle that is often missed on plain radiographs of supracondylar
and intercondylar femur fractures. It involves the lateral condyle more frequently than the medial. Identification is important as it
may impact operative planning and likely require screw fixation in the anteroposterior plane.
Nork et al. reviewed 202 supracondylar-intercondylar distal femoral fractures and found a 38% prevalence of associated coronal
plane fractures. The authors recommend CT scan imaging of all supracondylar and intercondylar fractures.
Ostermann et al reported on 24 unicondylar fractures of the distal femur treated with open reduction internal fixation with a screw
construct. Twenty-three patients acheived satisfactory results at 5 year follow-up. Illustrations A and B are another example of a
supracondylar femur fracture with an associated Hoffa fracture identified on CT scan.
Illustrations: A

37. PREFERRED RESPONSE 3


Conventional plating provides stable internal fixation when fractures are anatomically reduced. Stability of this type of fixation
relies on the plate/bone interface and the friction that develops between this interface. Locked plates rely on the plate/screw
interface, and each provides not only axial stability but also angular stability; each screw acts as a fixed angle device. Indications
for locked plating for indirect reduction include: 1. metaphyseal/diaphyseal fractures 2. comminuted diaphyseal fractures 3.
comminuted metaphyseal fractures. 4. short segment fixation. Locked plates are not indicated for displaced articular fractures
unless anatomic rigid fixation of the articular surface is done first (locking technology cannot reduce fractures/lag segments
together).
The referenced article by Gardner et al reviews locking technology and reminds us that compression technology using nonlocking screws and plates is still needed for many fractures and is even required for proper treatment of some fractures.
The referenced article by Wagner is an instructional paper on how to use hybrid plating technology and reviews concepts such as
the necessity of lag screw fixation before locking.
The referenced study by Egol et al is a review paper that notes that locked plates and conventional plates rely on completely
different mechanical principles to provide fracture fixation and in so doing they provide different biological environments for
healing. They report that locked plates are indicated for: indirect fracture reduction, diaphyseal/metaphyseal fractures in
osteoporotic bone, and with bridging severely comminuted fractures.

38. PREFERRED RESPONSE 2


The clinical presentation is consistent with and inferior pole patella fracture. Open reduction and internal fixation is the most
appropriate treatment of this injury pattern.
Whenever possible, salvage of the inferior pole through open reduction internal fixation is favored over simple excision and
patellar tendon advancement, as this has been shown to be associated with improved outcomes. This is not always possible,
however, and pole resection can be performed if the inferior comminution precludes fixation. Inferior outcomes of the partial
distal patellectomy and patellar tendon advancement are (aside from the possible resulting patella baja) probably not directly
related to the patellofemoral articulation. On the undersurface, the proximal 75% of the patella is covered with articular cartilage;
however, the distal 25% is not, and does not articulate with the femoral trochlea.
Kastelec et al. performed a retrospective review of ORIF v. pole resection followed over 4.6 years postoperatively. There was a
significant increase (better) in patellofemoral score with ORIF; significant differences in knee pain, tolerated activity levels, and
ROM were also noted. Patella baja was frequently noted with pole resection and correlated with worse functional outcomes.
Matejcic et al. performed a retrospective review of ORIF v. pole resection followed over 5.3 years postoperatively. Results were
excellent/good in 90.1% of the ORIF and only 73.1% of the pole resection patients. In addition, significant differences between
the groups were noted with regard to knee pain, swelling, level activity, compression pain, range of motion, muscular atrophy,
muscular strength, and final patellofemoral score (all better with ORIF).
Figure A is a lateral knee radiograph showing an inferior pole patella fracture.
39. PREFERRED RESPONSE 5
Symptomatic hardware following surgical fixation of patella fractures is the most common complication, with ~18% of patients
electing to undergo hardware removal.
Patella fractures are treated in a variety of ways. Non-displaced fractures with an intact extensor mechanism are typically treated
non-operatively, displaced transverse fractures are typically treated with a tension-band construct, and severely comminuted
fractures of the superior and inferior pole may be treated with partial patellectomy. When using the tension-band construct, either
K-wires or cannulated screws may be used. Studies have shown decreased rates of symptomatic hardware when cannulated screws
are used.
Melvin et al. reviewed patella fractures in adults. They cite the most common complication of operative fixation is symptomatic
hardware, with literature reporting variable rates from 0-60%. This is increased in open fractures likely secondary to added insult
to the soft tissues at the time of injury.
Eggink et al. looked at 60 patients who underwent tension-band wiring for patella fractures. Group 1 had their k-wires bent
proximally and distally, while group 2 only had their k-wires bent proximally (see article for pictures). All failures (9) occurred in
the group that had the k-wires bent proximally only.
Figures A and B show the post-operative films of a cannulated-screw tension band construct with a check-rein wire inserted in the
tibial tubercle. Illustration A shows an example of a K-wire tension band construct used to treat a comminuted patella fracture.
Incorrect Answers:
Answer 1: Patellofemoral arthritis is a known complication following surgical fixation of patella fractures, but is less common
than symptomatic implants.
Answer 2: Knee stiffness is a common complication following patella fractures, however the clinical symptoms in this patient are
consistent with prominent hardware
Answer 3: Nonunion following patella fractures is a rare complication for closed injuries.
Answer 4: Intrarticular corticosteroid injection would not be indicated in this clinical scenario.
Illustrations: A

40. PREFERRED RESPONSE 3


Partial patellectomy is a recommended treatment for a comminuted superior or inferior pole fracture measuring <50% of the
patella's height that are not amenable to ORIF.
Comminuted patellar fractures are challenging to manage. Older studies show that in severely comminuted fractures, partial
patellectomy is the recommended treatment. Newer studies are showing improved outcomes with ORIF. Therefore, if possible,
proceed with ORIF. Unfortunately, when the comminution is severe, ORIF is often not possible and partial patellectomy is
required.
The area can be excised and a technique of quadriceps tendon (superior pole) or patellar tendon (inferior pole) repair can then be
undertaken.
Saltzman et al. reported maintenance of range of motion and strength (85% of the contralateral) with partial patellectomy at an
average of 8.4 years follow up.
Marder et al. showed that change in patellofemoral forces is minimized by attaching the patellar tendon/quad tendon anteriorly on
the remaining patella. Their study also showed that by resecting more of the inferior pole that contact stress increased as the size
of the discarded portion of the patella increased.
Matejcic et al report, in a retrospective study of 71 patients that either had basket plate osteosynthesis v. partial patellectomy, that
osteosynthesis by basket plate provides better clinical results. They recommend this technique if possible.
41. PREFERRED RESPONSE 3
Figure C shows a Schatzker IV tibial plateau fracture, or medial fracture-dislocation of the knee. Of the fracture patterns shown,
Schatzker IV tibial plateau fractures have the highest incidence of vascular injury and most often require measurement of an
ankle-brachial index (ABI) to rule-out associated vascular injury.
Schatzker IV tibial plateau fractures (fracture of the medial plateau) are rare and are most commonly associated with high-energy
trauma after a varus/axial load. At the time of initial injury, the fracture pattern produces a temporary dislocation of the knee,
placing tension on the peroneal nerve and popliteal artery. Because of the likelihood of associated popliteal artery injury, anklebrachial indices, frequent neurovascular checks, and arteriography are commonly performed following injury.
Berkson et al. review high-energy tibial plateau fractures. They state that Schatzker IV fractures are usually the result of highenergy trauma, and have a high incidence of popliteal artery and peroneal nerve injury. In contrast, Schatzker V and VI are more
commonly associated with compartment syndrome.
Gardner et al. review 103 tibial plateau fractures. 77% of fractures had an associated rupture of either cruciate or collateral
ligaments. 86% of Schatzker IV fractures had an associated medial meniscus tear.
Illustration A is a worksheet for calculating the ankle brachial index. An ABI less than 0.90 has been shown to have a sensitivity
exceeding 87% and a specificity exceeding 97% for identifying lower-extremity arterial injury.
Incorrect Answers:
Answer 1: Figure A shows a non-displaced Schatzker I tibial plateau fracture. These are more commonly associated with lateral
meniscal pathology.
Answer 2: Figure B shows a Schatzker V tibial plateau fracture. While it occurs through a high-energy mechanism, vascular
compromise is less common than Schatzker IV fractures.

Answer 4: Figure D shows a distal 1/3 tibial shaft fracture. These should be monitored carefully for compartment syndrome.
Answer 5: Figure E shows an ankle fracture-dislocation. After successful reduction, vascular status usually returns to normal and
further diagnostic imaging is not needed unless the foot is avascular.

42. PREFERRED RESPONSE 5


The above clinical scenario is consistent with a bicondylar tibial plateau fracture. Bicondylar tibial plateau fractures are best
definitively treated with dual incision technique using separate lateral plateau and posteromedial plates.
Yoo et al perfored a biomechanical study comparing locking and nonlocking single and dual plating constructs in maintaining
posteromedial fragment reduction in a bicondylar tibial plateau fractures. They found that the posteromedial fracture fragment
tolerated higher loads with a posteromedial conventional plate construct. The superiority of this dual plating construct may be
caused by unreliable penetration of the posteromedial fragment by the lateral locking screws alone.
Zeng et al also performed a biomechanical study comparing the biomechanical strength of four different fixation methods for the
posteromedial tibial plateau split fracture. The four following constructs: anteroposterior lag-screws, an anteromedial limited
contact dynamic compression plate (LC-DCP), a lateral locking plate, or a posterior T-shaped buttress plate were studied. They
found that the posterior-based buttress plating technique was the most biomechanically stable fixation method allowing the least
amount of fracture subsidence for posteromedial split tibial plateau fractures.
Figure A shows an AP radiograph of a bicondylar tibial plateau. Illustration A shows a post-operative radiograph following the
dual-plating technique for bicondylar tibial plateau fractures.
Incorrect answers:
Answer 1: External fixation is not the most appropriate definitive care for bicondylar tibial plateau fractures.
Answers 2-4: Not as biomechanically stable as dual plating technique for bicondylar tibial plateau fractures.

43. PREFERRED RESPONSE 3


Figure A shows a Schatzker II tibial plateau fracture. Joint widening of 6mm is commonly asociated with a lateral meniscal injury.
Tibial plateau fractures are commonly associated with soft tissue injuries and the operative surgeon needs to be aware of these
commonly associated injuries.

Gardner et al. (2005) examined 62 consecutive Schatzker type II fractures with radiographs and MRIs preoperatively. They found
that joint depression greater than 6mm and widening of greater than 5mm was associated with a lateral meniscal injury over 80%
of the time.
Gardner et all. (2006) that looked at MRIs for 103 consecutive patients with all types of tibial plateau fractures, only one patient
had no soft tissue injuries. 94 (91%) patients had a lateral meniscal injury, 79 (77%) patients had cruciate ligament injury and 70
(68%) patients had a posterior lateral corner injury.
Figure A: shows a Schatzker type II fracture of a tibial plateau.
Incorrect Answers:
Answer 1: The above mentioned articles only found associations with lateral meniscal injuries when joint depression was at least
6mm.
Answer 2: No known association with soft tissue injuries after tibial plateau fractures.
Answer 4: No known association with soft tissue injuries after tibial plateau fractures.
Answer 5: Suggestive of an ACL injury, not an injury to the lateral meniscus.
44. PREFERRED RESPONSE 2
This patient sustained a high-energy injury to the left knee, including a tibial plateau fracture as well as both anterior cruciate
ligament (ACL) and posterolateral corner (PLC) injuries as indicated on the physical examination findings. In addition to ORIF of
the plateau fracture, the surgical plan should include ACL reconstruction as well as posterolateral corner (PLC) reconstruction,
specifically with lateral meniscal repair and allograft reconstruction of the lateral collateral ligament (LCL,asterisk in Figure C)
and popliteofibular ligaments.
The PLC consists of static (LCL, popliteus tendon, popliteofibular ligament, lateral capsule) and dynamic (biceps femoris,
popliteus muscle, IT band, lateral head of the gastrocnemius) structures. Failure to identify a PLC injury associated with an ACL
injury often leads to failure of ACL repair.
Stannard et al. reported on the clinical outcomes of 22 patients with PLC injuries (7 isolated) who underwent modified 2-tailed
reconstruction of the popliteofibular ligament and LCL utilizing transtibial and transfibular bone tunnels. At an average 29.5
months post-operatively, the authors noted excellent results with restoration of range of motion and stability in both the isolated
and multiligamentous injured groups.
Stannard et al. reported on a separate cohort of 56 patients with PLC injuries either undergoing direct repair or modified 2-tailed
reconstruction. The authors noted significantly inferior results in the repair group (37% failures) compared to the reconstruction
group (9%) failures, and concluded that reconstruction is the procedure of choice for the majority of patients who sustain highenergy PLC injuries.
Levy et al. reported on 28 patients with multiligament knee injuries undergoing either direct PLC repair with staged cruciate
ligament reconstruction or delayed single-stage multiligament reconstruction. The authors noted a significantly higher rate of
failure in the repair/staged group compared to the delayed reconstruction group, and deemed reconstruction to be a more reliable
option than repair alone in the multiligamentously injured knee.
Figure A includes AP and lateral radiographs of the left tibia demonstrating a comminuted lateral tibia plateau fracture. Figure C
demonstrates a gross dissection of the posterolateral corner (left knee), with the asterisk on the lateral collateral ligament (LCL)
Incorrect answers:
Answer 1: Figure B demonstrates a gross dissection of the posterolateral corner (left knee), with the asterisk on the IT band
Answer 3: Figure D demonstrates a gross dissection of the posterolateral corner (left knee), with the asterisk on the biceps femoris
Answer 4: Figure E demonstrates a gross dissection of the posterolateral corner (left knee), with the asterisk on the peroneal nerve
Answer 5: Figure F demonstrates a gross dissection of the posterolateral corner (left knee), with the asterisk on the lateral head of
the gastrocnemius

45. PREFERRED RESPONSE 1


Figure A demonstrates a comminuted tibial plateau fracture with significant intra-articular involvement. Basic science evidence
has demonstrated that post-operative gentle compressive loading may have a positive impact on articular cartilage healing;
however, excessive shear loading may be detrimental.
Irrgang et al provide guidelines for rehabilitation following surgical management of articular cartilage lesions of the knee. They
state that after articular cartilage repair, exercises to enhance muscle function must be done in a manner which minimizes shear
loading of the joint surfaces in the area of the lesion. The authors also discuss the benefits of gentle compressive loading and
motion of the joint, and its positive effects on chondrocyte nutrition. Furthermore, they recommend a period of protected weight
bearing as often being necessary, and that this should be followed by progressive loading of the joint.
Illustration A is a diagram showing the different layers of joint cartilage.

Illustrations: A
46. PREFERRED RESPONSE 4
Buttress plating is best indicated for simple partial articular fractures, such as shown in Figure D. Buttress plates can support a
metaphyseal fragment and neutralize the shear and compressive forces across the cancellous bone. The actual buttress effect is
only on the side of the plate. Pre-shaped plates are frequently used as buttress plates, as they conform to local anatomy, however
exact contouring of the plate is necessary. Buttress plating is not appropriate for articular depression fractures, such as those
shown in Figures A, C, and E. Furthermore, buttress plating would provide inadequate fixation for a metadiaphyseal fracture, such
as that shown in Figure B.
Illustration A shows an example of a buttress plate used to treat a tibial plateau fracture.
Illustrations: A

47. PREFERRED RESPONSE 2


Figure B represents a medial plateau fracture/dislocation pattern (Schatzker IV). This fracture typically requires more energy to
occur than the corresponding lateral plateau injury, which is due to the more dense bone on the medial side. A fracture-dislocation
of the knee must be suspected with these injuries, as the femur will sometimes follow the displaced medial tibial condyle. Along
with a proper vascular exam, ankle brachial indices (ABI) must be immediately taken and if abnormal further vascular testing is
warranted. Furthermore, these injuries which are a hybrid of a dislocation and a fracture will often have a benign appearance on
radiographs, but a high rate of vascular complications.
Berkson et al present a review article and they stress the importance of safeguarding tissue vascularity and while emphasizing
joint reduction and restoration of the mechanical axis of the limb.
Ottolenghi et al in their review article showed a vascular injury rate of 2% for tibial plateau fractures.

Stark et al in their review showed a very high incidence of acute compartment syndrome in Schatzker IV and VI injuries. In their
series, 18% of Schatzker VI and 53% of Shatzker IV fractures developed compartment syndrome.
48. PREFERRED RESPONSE 2
The injury is a Schatzker type 2 with a significant split and depressed lateral tibial plateau fracture. The anteromedial cortex and
medial plateau remain intact and connected to the tibial shaft making this a partial articular fracture (AO type B) and obviating the
need for medial plate fixation. The best fixation strategy includes reduction of the articular surface with metaphyseal support with
bone graft or bone substitute and a lateral plate for buttress support and subchondral screw support of the articular fragment.
Marsh et al revisited the OTA and AO fracture and dislocation classifications to unify coding of these fractures. Illustration A
shows the Schatzker classification of tibial plateau fractures.
Illustrations: A

49. PREFERRED RESPONSE 4


Figures A through D show a medial tibial plateau fx, best classified as a Schatzker IV. This injury is high-energy in nature and
often requires a staged approach incorporating initial reduction and spanning external fixation. Appropriate treatment of this injury
involves a medial buttress plate to hold the medial tibial condyle in position. This fracture pattern can be associated with a knee
dislocation (see Illustration A). In a fracture-dislocation, the femur often displaces with this medial condylar fragment and is
involved with a significant rate of neurovascular injury/compartment syndrome. Along with a proper vascular exam, ankle
brachial indices (ABI) must be immediately taken and if abnormal, further vascular testing is warranted.
Illustrations: A

50. PREFERRED RESPONSE 5


Lipohemarthrosis is formed when an intraarticular fracture occurs and can be detected with arthrocentesis or imaging such as xray,
MRI, ultrasound, or CT. It is most commonly seen with occult tibial plateau fractures but can be associated with any intra-articular
fractures. Up to three layers are visible on an MRI (fat/serum/cellular parts of blood), and this separation may take up to 3 hours to
appear after injury. An example of hemarthrosis as seen on CT is shown in Illustration A. The referenced article by Ahn et al is a
review of MRI findings in intraarticular knee injuries. They note that detection of lipohemarthrosis on an MRI is very sensitive

and specific for intraarticular fracture. The referenced article by Schick et al reports that MRI can be as sensitive as arthrocentesis
in detecting lipohemarthrosis (occult fracture).

Illustrations: A

51. PREFERRED RESPONSE 2


Maintenance of mechanical axis correlates most with a satisfactory clinical outcome when managing an intra-articular fracture of
the proximal tibia.
According to the study of plateau fractures with up to 27 year follow-up by Rademakers et al, malalignment of the limb by greater
than 5 degrees tripled the rate of degenerative osteoarthritis (27% v. 9%). Age at time of injury had no effect on outcome; 31% had
joint space narrowing but 64% of those knees were well tolerated.
Weigel and Marsh's study looked at high energy plateau fractures treated with staged external fixation followed by internal
fixation, and noted a low rate of severe arthrosis even with mild to moderate joint incongruity.
Stevens et al noted a worse outcome with increasing age at presentation with these injuries; fracture type had a small influence
and adequacy of reduction had no significant influence on outcome.
Figure A is a coronal CT image showing a lateral tibial plateau fracture with significant joint depression.
52. PREFERRED RESPONSE 2
Figures A and B show a bicondylar tibial plateau fracture, with a typical appearing lateral fracture line and a posteromedial
fracture line. The posteromedial sheared fracture piece is difficult, and/or sometimes impossible, to achieve appropriate stable
fixation with a single lateral locking plate, as there will be limited screw purchase and fixation into the posteromedial fragment.
The referenced article by Georgiadis notes that a dual incision approach is safe and is associated with improved outcomes over
their historical comparisons. They describe the dual incisions and approaches in length, and review risks/issues with each
approach.
The other referenced study by Bhattacharyya et al notes that these fractures have a typical appearance of the posteromedial
fracture piece and that articular reduction quality is correlated with short-term results. They recommended buttress-type fixation
of these fracture pieces.
53. PREFERRED RESPONSE 3
In treating tibial plateau fractures, calcium phosphate has been shown to have the least amount of articular subsidence on followup examinations.
The referenced study by Russell et al noted a significantly increased rate of subsidence at 12 months with autograft as compared
to calcium phosphate cement (in types I-VI).
The other referenced study by Lobenhoffer et al noted improved radiographic outcomes and earlier weightbearing with usage of
calcium phosphate cement.

54. PREFERRED RESPONSE 3


Figures A and B show a bicondylar tibial plateau fracture with a large posteromedial fracture piece. This has clinical importance,
as currently available plate/screw constructs often have poor fixation of this fracture segment, and this pattern often requires a
second, posteromedial, approach and placement of a second plate/screw construct.
The referenced article by Barei et al notes a prevalence of posteromedial fracture pieces of nearly 33% of all bicondylar tibial
plateau fractures. They also recommend supplementary or alternative fixation techniques when this pattern is recognized.
The referenced article by Higgins et al notes a 59% incidence of this fracture pattern (consisting of nearly 25% of the total joint
surface) in bicondylar tibial plateau fractures, and recommends appropriate fixation to combat the vertical shear instability
through a separate approach.
The last referenced study by Higgings et al notes a significantly increased rate of late fracture displacement in a biomechanical
model with a single lateral locking plate as compared to a dual plate construct.
55. PREFERRED RESPONSE 3
The medial tibial plateau is more concave and more distal relative to the lateral tibial plateau.
Watson et al report "the medial tibial plateau has a more concave shape and is larger in both length and width than the lateral tibial
plateau, which has a slightly convex shape. The lateral tibial plateau lies proximal to the medial plateau. The convexity of the
lateral plateau helps differentiate it from the medial plateau on a lateral radiograph of the proximal tibia."
Illustration A shows the relative concavity of the medial and lateral proximal tibia.
Illustrations: A

56. PREFERRED RESPONSE 3


There are 4 main types of plating techniques: 1. Bridging 2. Neutralization 3. Dynamic Compression 4. Buttress plating. Plates
can utilize locking or non-locking screws.
Buttress plating is appropriate for a Shatzker Type I (see illustration C), as it can prevent collapse and axial deformity from shear
or bending forces.
Karunakar et al showed that there was no significant difference between split depression tibial plateau fractures (Shatzker II) fixed
with either a buttress plate with rafting screws versus a periarticular plate with built in rafting screw hole options, similar to the
commonly used pre-contoured periarticular locking plate.
57. PREFERRED RESPONSE 1
Figure A illustrates a depressed lateral tibial plateau. One of the key components of fixing a depressed articular segment is
maintaining the reduction.
Trenholm et al reviewed an experimental study where a split depression (Schatzker II) fracture was created in a cadaveric model.
The stiffness of the elevated fragment in cadavers using calcium phosphate cement versus cancellous bone graft as support
showed no difference, but calcium phosphate cement was found to have greater compressive strength than cancellous bone alone.
The review article by Hak reviews the composition, advantages, and disadvantages of commerically available bone graft
substitutes.
Incorrect answers:

2: Tricalcium phosphate is a bone graft substitute that is osteoconductive, but has less compressive strength.
3-4: Cancellous bone has less compressive strength than calcium phosphate.
5: rhBMP-7 is not used as a bone filling agent in this instance, as it is typically provided on a easily compressible collagen sponge.
58. PREFERRED RESPONSE 4
Figure D shows a medial tibia plateau fracture (Shatzker IV). Gardner et al reviewed a 103 consecutive tibial plateau fractures to
evaluate for associated soft tissue injury prevelance. The authors found that patients had a medial meniscal tear when they had a
medial fracture (Shatzker IV) 86% of the time. To review the Shatzker classification of tibial plateau fracture: I) lateral split II)
lateral split depression III) lateral compression (no split) IV) medial fracture V) bicondylar VI) metaphyseal dissociation. The
correct choice makes intuitive sense since one would expect concomitant soft tissue injury with the bony fracture. Lateral
meniscus pathology is the most common overall soft tissue injury with tibial plateau fractures (91%).
59. PREFERRED RESPONSE 3
Figure A shows a displaced Schatzker II (lateral split-depression) tibial plateau fracture. These injuries are commonly associated
with peripheral meniscal tears.
According to the referenced article by Abdel-Hamid et al, the overall incidence of associated soft tissue injuries of the knee in
tibial plateau fractures is 71%. The incidence of each injury in their study was: meniscal tears (57% - usually peripheral tears),
ACL (25% - more common in more severe fractures), PCL (5%), LCL (3%), MCL (3%), peroneal nerve (1%). No vascular injury
was seen in their collection of 98 patients.
60. PREFERRED RESPONSE 1
The Figure shows a bicondylar tibial plateau fracture. The goals that need to be met when treating tibial plateau fractures are the
following: restoration of mechanical axis alignment (most important aspect), restoration of condylar width, articular reduction,
and restoration of knee stability. Since the soft tissue envelope is favorable, open reduction internal fixation with dual incisions
and dual plates will provide the best probablity of achieving those goals.
Gosling et al did a biomechanical evaluation in cadavers comparing lateral locked plating with a combined medial and lateral
plate and found no difference in resistance to vertical subsidence even with loads exceeding the average body weight. However,
this was a cadaveric study with no mention and capability of analyzing articular reduction. Lateral locked plating only allows for
indirect reduction of the medial plateau.
Barei et al in a retrospective review found that comminuted bicondylar tibial plateau fractures can be successfully treated with
open reduction and medial and lateral plate fixation using 2 incisions, and postulate that the use of 2 incisions may contribute to a
lower wound complication rate. A two incision approach allows not necessarily for a stronger construct as some studies are
controversial, but for a more accurate reduction and restoration of alignment.
61. PREFERRED RESPONSE 4
Figures A and B show a plateau fracture with a lateral split and depression of the articular surface. In treating tibial plateau
fractures, calcium phosphate has been shown to have the least amount of articular subsidence on follow-up examinations due to its
high compressive strength.
The study by Lobenhoffer et al noted improved radiographic outcomes and earlier weightbearing with usage of calcium phosphate
cement. Welch and Zhang reproduced tibial plateau fractures in goats and compared cancellous autograft to calcium phosphate
cement augmentation. At 24 hours, four of five specimens treated with autograft had subsidence of the fragment. Only two
specimens from limbs treated with cement showed minimal subsidence; the remaining were congruent.
Yetkinlers study compared cement to no cement treatment in a model of depressed plateau fractures. Calcium phosphate cement
of high compressive strength provided equivalent or better stability than conventional open reduction and internal fixation with
either auto/allograft bone due to its low compressive strength and reduced mechanical stability.
62. PREFERRED RESPONSE 2

The radiographs and CT scan images show a depressed lateral tibial plateau fracture, which is correctly classified as a Schatzker
III tibial plateau fracture. This fracture typically occurs as the result of the femoral condyle directly impacting the articular surface
in older patients with osteopenia.
The referenced article by Bennett et al reviews the associated soft tissue injury with tibial plateau fractures. They found a 56%
frequency of associated soft tissue injuries overall, with MCL injured in 20%, the LCL in 3% , the menisci in 20%, the peroneal
nerve in 3%, and the anterior cruciate ligaments in 10%. Schatzker type IV and type II fracture patterns were associated with the
highest frequency of soft tissue injuries.
63. PREFERRED RESPONSE 4
Figure A shows a significantly displaced, high-energy proximal tibia fracture with intra-articular extension. Appropriate initial
treatment includes application of a spanning external fixation device with fasciotomy if needed.
The referenced article by Egol et al noted a low rate of wound infection, improved access to soft tissues, prevention of further
articular damage, and osseous stabilization. They reported the downside being residual knee stiffness.

64. PREFERRED RESPONSE 1


The clinical presentation and imaging studies are consistent with a tibial plateau fracture. Restoration of joint stability has been
shown to be the strongest predictor of long term outcomes.
Honkonen reviewed 131 tibial condyle fractures and determined that articular stepoff <3mm and tibial widening <5mm did not
negatively effect outcomes. In contrast, 70% of knees with moderate to severe malalignment went on to functionally unacceptable
outcomes. They suggested operative fixation for all medial uni and bicondylar fractures, any lateral fractures with >5 degrees of
valgus tilt, >3mm of articular depression, >5mm of condylar widening, or >5 degrees of valgus malalignment.
In the Marsh et al JAAOS symposium review, the authors noted that fractures with up to 10mm of articular depression and joint
stability obtained acceptable functional outcomes. They also cited a 20 year follow-up which indicated that articular step-off alone
was not a predictor of poor long-term results. More importantly, when instability is present with other factors, including step-off
and central depression, poor results followed.
Illustrations A and B show the intraoperative films. Illustration C reviews the Schatzker classification system.
Illustrations:
A
B
C

65. PREFERRED RESPONSE 2


In patients with multitrauma, combining reamed femoral nailing with fracture fixation (ie. tibial shaft) under tourniquet control
has been shown to increase pulmonary morbidity.
Limb reperfusion after tourniquet ischemia causes pulmonary microvascular injury. Similarly, microembolization, like that
associated with reamed femoral nailing, can induce pulmonary microvascular injury. Both processes result in increased pulmonary
capillary membrane permeability and edema, and ultimately increased pulmonary morbidity.
Karunakar et al showed in a canine model that there is no significant difference in the heat generated during reaming with and

without a tourniquet. The factor that made the most difference was related to the size of the reamer used compared with the
diameter of the isthmus. They concluded that the risk of thermal necrosis appears to be related more to the process of
intramedullary reaming than to the tourniquet.
Giannoudis and associates performed a prospective randomized trial on 34 patients to measure the rise of temperature during
reaming of the tibia before intramedullary nailing with and without the use of a tourniquet. The factor that generated the most heat
was using large reamers (11 mm to 12 mm) in a patient with a small isthmus (8 mm to 9 mm). Use of a tourniquet, steroid use, and
knee flexion during reaming were not shown to be associated with diaphyseal necrosis after reamed tibial nailing.
Pollak et al evaluated the association between femoral nailing followed by tourniquet ischemia and clinical lung injury. They
reviewed 72 patients with femoral shaft fractures and tibial or ankle fractures requiring internal fixation over a six year period. All
femoral shaft fractures were treated with reamed intramedullary nails, and the patients were divided into groups, based on whether
the tibial or ankle injury was managed surgically with or without a tourniquet. They noted increased pulmonary morbidity in the
group where a tourniquet was used.
Figure A shows a femoral shaft fracture at the junction of the middle and proximal one-third of the femoral shaft. Figure B shows
a contralateral tibial shaft fracture.
Incorrect Answers:
1-Tourniquet use has not been shown to lead to thermal necrosis of the bone during reaming of the tibial shaft.
3-The cortical temperature does not increase to a greater degree with tourniquet use versus non-tourniquet use during reaming.
4-There is no evidence that use of a tourniquet affects tibial shaft union rates after reaming and intramedullary nailing.
5-Tourniquet use has been associated with increased pain post-operatively.
66. PREFERRED RESPONSE 2
Figure B shows a midshaft tibia fracture with an intact fibula, which places the fracture at increased risk of varus malalignment
during healing.
Nonsurgically treated tibial shaft fractures with an intact fibula have an increased risk of varus malunion when treated
nonsurgically. The fibula acts as a strut, preventing valgus collapse, but predisposing to varus angulation. Limb-length
discrepancies can also occur.
Teitz et al. found that 26 percent of patients over the ago of 20 years old with isolated tibial shaft fractures treated nonsurgically
went on to varus malunion. They also found that when the fibula remains intact, a tibiofibular length discrepancy can develop and
cause altered strain patterns in the tibia and fibula leading to later ipsilateral joint sequeale.
Bonnevialle et al performed a retrospective study to assess the outcome after reamed nailing of tibial fractures with an intact
fibula. Thirty-eight fractures were evaluated at a minimum of one year from surgery. Healing was achieved after the first intention
treatment in 30 patients, after dynamization in 6. A non-union in 2 patients was also successfully managed with new nailing and
dynamization. The authors concluded that nailing is a reliable technique for the treatment of tibial fractures with an intact fibula.
Sarmiento et al reviewed 1,000 consecutive closed diaphyseal tibial fractures, treated with prefabricated functional below-knee
braces, to determine factors predictive of final fracture outcome. They found that final angulatory deformity in any plane was < or
= 6 degrees in 90% of patients. They also noted that the presence of an intact fibula was a relative contraindication for functional
fracture bracing because angulatory deformity was more likely to develop.
Figure B shows a midshaft tibial shaft fracture with an intact fibula.
Incorrect Answers:
Answer 1: Figure A shows a distal tibia and fibula fracture which are not specifically prone to varus malunion. Rather, if treated
with IM nailing, fracture alignment is dictated by placement of the guide-wire and path of reaming which must be center-center
distally.
Answer 3: Figure C shows a midshaft fibula and tibia fracture at the same level. The fibula, in this case, would not predispose the
fracture specifically to varus malalignment.

Answer 4: Figure D shows a proximal tibial shaft and associated fibula fracture. This fracture pattern is predisposed to valgus and
procurvatum.
Answer 5: Figure E shows a segmental tibial shaft fracture which are prone to shortening, translational, and rotational
malalignment.
67. PREFERRED RESPONSE 2
The most important factor shown to reduce the risk of infection at the site of an open fracture is early intravenous antibiotic
administration.
Infection risk after Gustilo Type II open fractures ranges from 10-20% in large studies. Antibiotic treatment initiated within 3
hours from the time of injury has shown to significantly reduce the rate of infection. Antibiotic coverage for Type II open fractures
should cover gram positive bacteria. Soil-contaminated wounds should include anaerobic coverage. The dose of antibiotic given
must be within a therapeutic range and titrated to the patient's weight (e.g. Ancef 2 g IV for >70 kg). Duration of antibiotic therapy
has been suggested to be between 1 and 3 days, although there is no agreement on a firm end point.
Pollak et al. reviewed a large cohort of open fractures treated at Level I trauma centers. They demonstrated a significant decrease
in infection rate with either early direct admission (<2 hours) or transfer (<11 hours) for ONLY type III open tibia fractures. They
did not not discuss timing of antibiotic treatment because this was not prospectively collected. Although they did not collect data
on antibiotic treatment, the authors theorize that early transfer potentially resulted in earlier administration of antibiotics.
Patzakis et al. examined a series of 1104 open fractures to determine the factors contributing to infection. They showed the most
important factor in reducing the infection rate was the early administration of antibiotics.
Illustration A is table showing the Gustilo classification of open fractures.
Incorrect Answers
Answers 1,4: These responses have been shown to reduce the risk of infection in open fractures, however, the most important
factor has been shown to be early antibiotic therapy.
Answer 3: Reamed intramedullary nailing has not been shown to decrease infection risk in open tibia fractures, however, skeletal
stability will prevent ongoing soft tissue damage
Answer 5: Vacuum-assisted wound dressings (or negative pressure wound therapy) are controversial as to whether they provide
any protective effect against infection.

68. PREFERRED RESPONSE 4


This patient has spiral distal tibia and proximal fibula fractures. Dedicated imaging of the ankle should be performed to exclude a
posterior malleolus fracture. Imaging options include either dedicated ankle radiographs or CT scans.
Tibial shaft fractures arise from high- or low-energy injuries. Low-energy injuries are characterized by (1) torsional mechanism of
injury, (2) spiral pattern, (3) fibula fracture at a different level. Surgical options for closed shaft fractures include IM nailing and
plate fixation. Concomitant ankle fractures should be treated during the same surgery to improve outcome.
Boraiah et al. examined the association of posterior malleolus fractures with spiral distal tibial fractures. They found that 39% had

posterior malleolus fractures. In simple fractures (92%), none of the posterior malleolar fracture lines were contiguous with the
diaphyseal fracture lines. In comminuted fractures (8%), an occult fracture line extended into the posterior tibial lip. Diagnosis
was missed in 5% before CT scanning was initiated, and 0% after.
Stuermer et al. examined tibial fractures with associated ankle injury. Of spiral tibial fractures, 37% extended into the ankle, 5%
involved the medial malleolus, 26% involved distal fibula, 8% had syndesmotic disruption, and 16% had posterior malleolar
fracture. They recommend ankle radiographs for rotational trauma, spiral distal third fractures, Maisonneuve fractures, and
fractures associated with an intact fibula.
Figures A and B are AP and lateral radiographs showing a spiral distal tibial fracture with a proximal fibula fracture. Illustration A
is an axial CT scan showing the posterior malleolar fracture not seen on plain radiographs. Illustration B shows a missed posterior
malleolar fragment in a fracture treated with IM nailing that subsequently displaced.
Incorrect Answers:
Answers 1, 2, 3, 5: There is no association between spiral distal tibial fractures and injuries of the hip, knee or calcaneus.

69. PREFERRED RESPONSE 1


The center of rotation of angulation(CORA) in diaphyseal tibial deformity is defined as the intersection of the proximal
mechanical(PMA) or anatomical axis(PAA), and the distal mechanical(DMA) or anatomical axis(DAA).
Angular deformity of the femur or tibia involves angulation not only of the bone but also of its axes. When a bone is divided and
angulated, the mechanical and anatomic axis of the bone are also divided into proximal and distal segments. The pairs of proximal
and distal axis lines intersect to form an angle. The point at which the proximal and distal axis lines intersect is called the CORA.
The axis line of the proximal bone segment is called the PMA or PAA, and axis line of the distal segment is called the DMA or
DAA. In the tibia, because the mechanical and anatomical axes are almost the same, the PMA and PAA lines overlap, as do the
DMA and DAA lines.
Illustration A shows the CORA as it relates to the axes of an angulated tibia.
Incorrect Answers:
2-5:These do not describe the CORA or any other specific anatomical points as they related to long bone deformity.

70. PREFERRED RESPONSE 4


Decreased extensor hallucis longus strength is the physical exam finding most consistent with transient peroneal nerve
neurapraxia.
Transient peroneal nerve neuropraxia has been noted in up to 5% of patients undergoing closed nailing of tibial shaft fractures.
This is of unknown etiology, although injury to the peroneal nerve branches can be from placement of the interlocking screws into
the nail. In this syndrome, extensor hallucis longus weakness is noted and decreased sensation is seen in the 1st dorsal webspace
(deep peroneal nerve distribution). These symptoms generally begin improving by 3 months and have variable rates of recovery.
Robinson et al. reported on 'dropped hallux' syndrome, with weakness of extensor hallucis longus and numbness in the first web
space, without clinical involvement of extensor digitorum longus or tibialis anterior. They found this in 5% of their patients after
tibial nailing of closed tibia fractures, and all recovered either partially or completely by 4 months.
Lawrence et al. performed an anatomic study that reported the deep peroneal nerve was located superficial to the anterior tibial
artery between the tibialis anterior and extensor hallucis longus muscles in the distal one third of the leg. They also found that the
deep peroneal nerve crossed deep to the extensor hallucis longus tendon to enter the interval between the extensor hallucis longus
and extensor digitorum longus at an average distance of 12.5 mm proximal to the ankle.
Figure A shows a tibial shaft fracture with mild displacement.
Incorrect Answers:
Answer 1: Lateral hindfoot sensation is generally via the sural nerve.
Answer 2: Achilles reflex is via S1 roots.
Answer 3: The peroneus longus is innervated by the superficial peroneal nerve.
Answer 5: Plantar forefoot sensation is via the tibial nerve along with the medial and lateral plantar branches.
71. PREFERRED RESPONSE 4
Figures A and B demonstrate an open, segmental tibia shaft fracture with extensive soft-tissue injury. Recent evidence has
demonstrated that time to transfer to a definitive trauma center has a significant effect on the incidence of infection for highenergy, open lower extremity fractures.
Pollak et. al analyzed a subgroup of 315 patients with high-energy, open lower extremity fractures from the Lower Extremity
Assessment Project (LEAP study). Time to admission to a definitive trauma center for treatment was a significant, independent

predictor of infection, with patients transferred 11-24 hours following injury having a significantly increased risk of major
infection as compared with patients transferred within 3 hours of injury.
Werner et. al reviewed the existing literature surrounding the urgency of surgical debridement for open fractures, specifically
relating to the "6 hour rule". The authors found limited evidence in the current literature to support emergent debridement within 6
hours of injury, and recommend urgent debridement (within 24 hours) once the patient is physiologically stable, life threatening
emergencies have been addressed, and adequate surgical staff and resources are available.
Incorrect Answers:
1. External fixator application, although important for limb stability and ongoing assessment of the soft-tissue envelope, has not
been shown to affect the rate of infection for open fractures
2. Tetanus prophylaxis is only effective against one infectious organism
3. Time to debridement was not shown to affect the rate of infection in the referenced articles
4. Time to soft-tissue coverage was not shown to have a statistically significant difference on the rate of infection in the level II
study by Pollak et. al
72. PREFERRED RESPONSE 5
The above clinical scenario describes a post-op superficial peroneal nerve (SPN) deficit following ORIF of a tibial fracture using
both external fixation and minimally invasive plate osteosynthesis (MIPO). The less invasive stabilization system (LISS) plate by
Synthes is a system which utlizes the MIPO technique. This minimally invasive technique can increase the risk of damage to the
SPN without careful identification of the nerve distally due to its close proximity to LISS plate holes 11-13.
Deangelis et al studied the anatomy of the superficial peroneal nerve in relation to fixation of tibia fractures with the LISS plate
using cadaveric dissections. They found that the superficial peroneal nerve is at significant risk during percutaneous screw
placement in holes 11 through 13 of the 13-hole proximal tibia LISS plates. They recommended using an incision and careful
dissection down to the plate in this region of distal locking screws to minimize the risk of damage to the nerve.
Roberts et al also studied neurovascular anatomy of the leg in relation to screw placement, but did it in relation to locking screws
used in intramedullary nailing. They concluded that intramedullary nail locking screws placed from a lateral-to-medial direction
minimized the risk of injuries to the SPN and tibial neurovascular bundle. A disadvantage of lateral-to-medial locking screw
placement was less resistance to nail bending forces.
73. PREFERRED RESPONSE 3
If a hypertrophic nonunion is present, it is most likley a mechanical issue. Tibial diaphyseal hypertrophic nonunions (Illustration
A) have approximately an 85-90% incidence of union with exchange reamed nailing. A nonunion that has bone loss or appears
atrophic (Illustration B) will usually require improved mechanical stability as well as biological stimulation in the form of either
autograft or an osteoinductive substance like BMP. A bone defect of up to 5-6cm in length can usually achieve union with bone
grafting. In the presence of an infected nonunion, the infectious process needs to be addressed prior to the introduction of any
revision hardware. If a patient does not show radiographic signs of tibial fracture union for 9 months and does not have
progression toward healing for 3 consecutive months, then revision surgery would be indicated.
Tempelman et al looked at 71 tibial shaft fractures treated with nonlocked or dynamically locked IM nails and found a loss of
alignment in 11% of the fractures that were not transverse in nature. They concluded that these nailing techniques should not be
used in the treatment of spiral or oblique tibial shaft fractures.
Incorrect Answers:
1-A 4cm bone defect could not be corrected with exchanged nailing alone, and would either need extensive grafting or bone
transport
2-An infected tibial shaft nonunion would require infection clearance prior to exchanged nailing
4-Atrophic nonunions typically need biologic stimulation in the form of acute grafting or insertion of a BMP type substance

5-Hypertrophic metadiaphyseal distal tibial nonunions can be treated with isolated exchanged nailing, however this does not have
the same success as diaphyseal injuries. It is difficult to acheive appropriate stability to allow for fracture healing in the
metadiaphyseal region, and other modalities such as plating need to be considered.

74. PREFERRED RESPONSE 5


The radiographs demonstrate a distal third spiral tibia shaft with a proximal fibula fracture. The coronal and sagittal alignments
are within acceptable limits. The oblique fracture is at risk of shortening, especially with a concomitant fibular fracture.
Acceptable alignment for non-operative treatment of tibia fractures is defined as:
<5 degrees varus-valgus angulation,
<10 degrees anterior/posterior angulation
>50% cortical apposition
<1 cm shortening
< 10 degrees rotational alignment
Sarmiento et al reviews fracture bracing for the treatment of long bones. With reference to tibial fractures, bracing is best for
transverse fractures. Shortening is difficult to control in oblique fractures. However, shortening is usually less than 15 mm and
does not result in functional limitations. He reports union in 97% of tibial fractures treated with bracing.
One year follow-up radiographs are provided in Illustrations A and B. The patient presented in this question went on to functional
healing.

75. PREFERRED RESPONSE 4


The clinical photo and radiograph are consistent with a Grade III open tibia fracture.
The referenced study by the LEAP group reviews 527 patients with severe lower extremity fractures and found that the most
important factor in determining the ability to salvage the extremity remains the severity of the soft tissue injury of that extremity.
Bone loss has been shown to have no effect on the eventual outcome (amputation versus salvage). Similarly, plantar sensation at
presentation has no bearing on final outcome, and in the LEAP study, often either partially or fully returned.
76. PREFERRED RESPONSE 3
Exchange nailing is indicated for nonunions of diaphyseal femoral and tibia fractures in the absence of infection, comminution, or

segmental bone loss. Hypertrophic nonunions need better stability (increased nail diameter) to achieve union. Where as atrophic
nonunions often need better biology (bone graft, flap coverage, etc.)
The referenced article by Brinker et al reviews the indications for exchange nailing. They argue, on the basis of the available
literature, that exchange nailing is an excellent choice for aseptic nonunions of noncomminuted diaphyseal femoral and tibia
fractures.
Zelle et al. demonstrated 95% success with reamed exchange nailing for the treatment of aseptic tibial shaft nonunions that were
initially treated with nonreamed intramedullary nailing.
77. PREFERRED RESPONSE 5
rhBMP-2 has been shown in two randomized controlled studies to have improved clinical outcomes in grade III open tibial
fractures.
Swiontkowski et al and Govender et al have shown in two separate clinical studies that use of this product has: significantly fewer
invasive interventions (e.g., bone-grafting and nail exchange), significantly faster fracture-healing than did the control patients,
increased healing (union) rates, fewer hardware failures, fewer infections, and faster wound-healing (83% compared with 65%
had wound-healing at six weeks).
78. PREFERRED RESPONSE 4
Placement of long lateral tibial plates have been shown to have a risk of iatrogenic injury to the superficial peroneal nerve, which
has a sensory distribution to the dorsal foot. This risk is seen especially with percutaneous approaches, such as those used with the
LISS plate.
The first reference by Deangelis et al found a risk of superficial peroneal injury with percutaneous screw placement of holes 11-13
in the LISS plate.
The second referenced article by Roberts et al noted a slightly increased distance to the neurovascular bundle when interlocking
tibial nails in a lateral to medial direction (compared to medial to lateral locking) and slightly increased biomechanical strength
when locking in a medial to lateral direction.
The third referenced article by Wolinsky et al notes a risk of iatrogenic injury to the deep peroneal nerve and anterior tibial artery
with an anterolateral approach to the distal tibia, but notes the superficial peroneal nerve is safe with an appropriate exposure.
79. PREFERRED RESPONSE 5
Reamed exchange intramedullary nailing of diaphyseal tibial shaft fractures in which there is less than 30% of cortical bone loss
can achieve union rates ranging between 76%-96%. In a review article, Brinker et al discusses the indications and limitations of
exchange nailing of ununited fractures. Biological as well as mechanical factors contribute to the healing of nonunions.
Anatomically, multiple studies cited in this review article demonstrate that distal femoral nonunions do not readily achieve union
following exchange nailing. Humerus nonunions, both diaphyseal and proximal locations, more readily achieve union with plate
fixation and bone grafting according to articles cited in this review as well. Banaszkiewicz et al also discusses the difficulties with
exchange nailing of femoral nonunions with a large percentage of patients requiring additional surgeries to achieve union.
Templeman et al discusses the successful results of reamed exchange intramedullary nailing of delayed union and nonunion of the
tibial shaft. The authors recommend the use of bone graft only when there is substantial bone loss, usually exceeding 30% of the
cortical diameter.
80. PREFERRED RESPONSE 1
The safe zone for tibial nail placement as seen on radiographs is just medial to the lateral tibial spine on the anteroposterior
radiograph and immediately adjacent and anterior to the articular surface as visualized on the lateral radiograph.

Tornetta et al specifically located the safe zone for nail entry in a study using fresh frozen cadaver knees. The authors found that
the safe zone for nail placement is located 9.1+/-5 millimeters lateral to the midline of the plateau and three millimeters lateral to
the center of the tibial tubercle. The width of the safe zone averaged 22.9 millimeters and was as narrow as 12.6 millimeters.
The starting point of the of the nail can be best viewed on the lateral knee radiograph, an example of which is shown in Illustration
A. Illustration B shows the "sweet spot" for nail insertion as defined by Tornetta.
Illustrations: A

81. PREFERRED RESPONSE 2


Osteogenic Protein-1 (OP-1), which is also known as BMP-7, has been evaluated in a randomized, prospective, multi-institution
study of tibial nonunions.
Between 5% to 10% of tibial shaft fractures progress to nonunion, causing substantial disability. Bone autografts, along with
internal fixation, are the usual treatment for these failures, but the morbidity associated with autogenous tissues remains
problematic. Bone morphogenetic proteins are currently available for clinical use and preclinical models, and an increasing
number of patients treated with these molecules demonstrate their safety and efficacy.
Friedlaender et al studied BMP-7 (Osteogenic Protein-1 or OP-1) in a randomized, prospective, multi-institution study of tibial
nonunions. Clinical and radiographic outcomes were statistically indistinguishable at 9 months following treatment and OP-1
avoided donor site morbidity.
Swiontkowski et al performed a level I study of patients with acute open tibial fractures randomized to treatment with or without
rhBMP-2. Interestingly, in their subgroup analysis the authors found no significant difference between the two groups when
patients were treated with reamed intramedullary nailing.
82. PREFERRED RESPONSE 1
Administration of rhBMP-2 at the time of definitive fixation has been shown to decrease the need for subsequent bone grafting
procedures in Gustilo-Anderson type IIIA and IIIB open tibia fractures.
Swiontkowski et al performed a subgroup analysis of two prospective randomized control studies regarding the use of rhBMP-2 in
open tibia fractures. The authors found a significant risk reduction in the need for secondary procedures, including bone grafting,
with the addition of rhBMP-2 for type IIIA and IIIB open tibia fractures.
Govender et al performed a randomized prospective RCT of 450 patients with open tibia fractures allocated to tibia nailing or
nailing with one of 2 different dosages of rhBMP-2. They found a 44% reduction in the need for secondary intervention as a result
of delayed union, better wound healing, and decreased infection in the higher dose rhBMP-2 group compared to controls.
Figure A demonstrates a severe soft tissue injury associated with an open tibia fracture. Figure B demonstrates a segmental tibial
shaft fracture with an associated fibula fracture.
83. PREFERRED RESPONSE 2
A significant malunion of the distal tibia has important consequences for patient outcome, including pain, gait changes, and
cosmesis.

The first referenced article by Milner et al looked at long-term outcomes of tibial malunions and noted that varus malunion led to
increased ankle/subtalar stiffness and pain regardless of the amount of radiographic degenerative changes.
The second referenced article by Puno et al reinforced the concept of decreased functional outcomes of the ankle with tibial
malunions, and noted that other lower extremity joints (ipsilateral and contralateral) do not have increased rates of degeneration
from such a malunion.
84. PREFERRED RESPONSE 4
Puloski et al determined the incidence and severity of tibial malrotation following reamed intramedullary nail fixation as
measured by computerized tomography. Malrotation was defined as an internal/external rotation deformity greater than 10
degrees. They found that 5 (22%) of the tibia were malrotated greater than 10 degrees, and of those 5 tibia, 4 were distal 1/3
fractures.
McKee et al randomized 85 patients undergoing intramedullary tibial nailing using manual traction with the leg draped free versus
standard fracture table positioning and traction. They found that manual traction is an effective technique that can save a
significant amount of time without sacrificing the quality of reduction or fixation of tibial shaft fractures.
85. PREFERRED RESPONSE 2
Post-operative gapping at the fracture site significantly increased the risk of reoperation due to nonunion or malunion.
Bhandari et al performed a retrospective study to identify which prognostic factors were associated with an increased risk of
reoperation for nonunion in surgically treated tibial shaft fractures. They examined over 200 fractures, and found the presence of
an open fracture wound (RR 4.32), lack of cortical continuity between the fracture ends following fixation (RR 8.33), and the
presence of a transverse fracture (RR 20.0) were the three variables most predicitive of reoperation.
Audige et al analyzed 416 patients with operatively treated tibial shaft fractures who were followed for at least 6 months. They
found that the greatest risk for delayed healing or nonunion was the presence of an open injury, fractures of the distal 1/3 of the
tibia, and postoperative gapping at the fracture site (The risk of healing problems was doubled for fractures of the distal shaft and
for fractures showing a postoperative diastasis).
86. PREFERRED RESPONSE 4
The patient in the scenario has a closed distal one-third tibia fracture. Canal reaming increases the biologic environment for
fracture healing but can potentially disrupt cortical blood flow. As such, many recommend canal reaming 1-2mm greater than the
canal width followed by insertion of a nail that matches the native canal width. Reamed and unreamed tibias have similar mineral
apposition rates.
In 1998, Hupel et al studied the effect of loose and tight unreamed, locked nails on cortical blood flow and strength of union in a
canine model. They found that loose nails allowed higher cortical reperfusion at the time of insertion and at eleven weeks.
In a later study by the same group in 2001, they studied the effect of non-reamed, limited reamed and standard reamed nails on
porosity, new bone formation and mineral apposition. They found the lowest porosity in the limited reaming group but found new
bone formation and mineral apposition rates similar at eleven weeks across the three groups. They concluded that limited reaming
is preferred in patients with vascular compromise to the tibia.
87. PREFERRED RESPONSE 4
Anterior knee pain is the most common complication after intramedullary nailing of the tibia. Dissection of the patellar tendon and
its sheath during transtendinous nailing was thought to be a contributing cause of chronic anterior knee pain.
The referenced paper by Toivanen et al. compared two different nail-insertion techniques in 50 patients who were randomized to
treatment with paratendinous or transtendinous nailing. Sixty-seven percent of the transtendinous and seventy-one percent of the
paratendinous approaches resulted in patients with postoperative anterior knee pain. The same authors published an 8 year followup which showed that the percentage dropped down to 29%, but there was still no advantage of paratendinous over the
transtendinous approach.

In the more recent study by Lefaivre with an average patient follow up of 14 years, knee pain was present in greater than 70% of
the respondents.
88. PREFERRED RESPONSE 2
The clinical scenario and radiographs are consistent with a Gustilo and Anderson type 3A open tibia fracture.
Melvin et al review the evidenced-based literature and make recommendations for the initial evaluation and management of open
tibial shaft fractures. The time elapsed before antibiotic administration and adequate surgical debridement of all contamination are
the only factors definitively shown to reduce infection and improve outcome. Traditional recommendations have suggested
surgical debridement of open fractures occur within 6 hours of injury. However, there is no literature to support this time window.
Certainly, open fractures should be addressed with urgency, but there is no evidence reporting a definitive time window. There is
insufficient data to recommend gram negative coverage with gentamicin for all open fractures although this is a common practice.
The addition of antibiotics to the irrigation solution has been shown to decrease bacterial load, but it has also demonstrated host
tissue necrosis and delayed wound healing. There is not sufficient data to support its use over a castile soap solution or normal
saline. Similarly, high pressure pulsatile lavage decreases bacterial load, but also seeds bacteria deeper within the soft tissues and
harms host tissues. There is no evidence to support pulsatile lavage over gravity flow.
89. PREFERRED RESPONSE 3
All of the answer choices are correct except #3. Intramedullary nailing can increase the risk of compartment syndrome.
In a study of 94 tibial fractures, Finkemeier reported 10 (11%) had compartment syndromes. Three of the 10 patients developed
the compartment syndrome postoperatively.
In comparing IM nailing to non-op, Bone et al showed that IM nailing had a shorter time to union (mean, 18 vs 26 weeks; p =
0.02), lower non-union rate (2% vs 10%), decreased incidence of shortening (2% vs 27%), and quicker return to work (mean, 4 vs
6.5 months), but no difference in compartment syndrome (0% in both groups).
The classic article cited by Sarmiento el al. reported that closed treatment with use of a prefabricated functional below-the-knee
brace was effective in a study of 1000 closed diaphyseal fractures of the tibia with an incidence of nonunion of only 1.1%.
However, those authors had very strict criteria for use of the fracture-brace (exclusion criteria included intact fibula and tibial
shortening >2cm).
90. PREFERRED RESPONSE 3
Superficial peroneal nerve (SPN) injury is a known complication of percutaneous plating of proximal tibial fractures with the
LISS system as seen in Figure A.
The Less Invasive Stabilization System (LISS) is a minimally invasive implant that uses indirect fracture reduction techniques.
When using the LISS system, percutaneous screw placement increases the risk of injury to nearby structures because they are not
necessarily visualized. The superficial peroneal nerve exits the superficial fascia of the leg approximately 8 cm above the tip of
the lateral malleolus. The nerve then travels from posterior to anterior in the vicinity of the distal aspect of the 13-hole proximal
tibia LISS plate (near holes 11-13). In a patient of shorter stature, the nerve could cross the distal portion of a 9-hole plate.
Deangelis et al. performed a cadaveric study using Less Invasive Stabilization System (LISS) plates and found that the average
distance from the SPN to the center of holes 11, 12, and 13 was 10.0 mm, 6.8 mm, and 2.7 mm respectively. They recommended
using a larger incision and careful dissection down to the plate in this region to minimize the risk of damage to the nerve.
Cole et al. retrospectively reviewed 77 tibia fractures treated with LISS and found that 91% healed without complication. In their
cohort, there were no superficial peroneal nerve palsies and one deep peroneal nerve palsy.
Figure A demonstrates AP and lateral x-rays of a tibial shaft fracture treated with a LISS plate.
Incorrect Answers:
A: compartment syndrome would have demonstrated pain out of proportion which the patient never complains of

B, D, and E are all less likely to be injured with LISS plate application than the superficial peroneal nerve.
91. PREFERRED RESPONSE 4
Valgus and flexion is the most common deformity seen after intramedullary nailing of proximal tibia fractures. The semi-extended
nailing position helps overcome the procurvatum or flexion deformity of the fracture.
Lang et al. reported in their study of 32 proximal third tibia fractures that 56% of the fractures had 5 degrees or more valgus
angulation and 28% had 10 degrees or more valgus angulation. Angulation in the AP plane ranged from 0 degrees to 20 degrees,
all of which was apex anterior. Nineteen (59%) fractures demonstrated 5 degrees or more angulation, and 7 (22%) fractures
demonstrated 10 degrees of more angulation.
Tornetta advocates use of extended medial parapatellar incision with the leg in a semiextended position to allow for a more
proximal and lateral starting point. This modified starting point forces the nail to overcome the tendency of the fracture to flex
(apex anterior) and go into valgus.
92. PREFERRED RESPONSE 3
Chronic anterior knee pain at the insertion site is the most common frequently reported complication of closed nailing of a tibial
shaft fracture. A high incidence of knee pain has been associated with IM nailing. The etiology of anterior knee pain remains
unclear. It had been previously thought that the incidence of pain is higher when the nail was inserted by a patellar tendon-spliting
approach versus a paratendon approach. According to the Keating paper, insertion of the nail through the patella tendon was
associated with a higher incidence of knee pain compared to the paratendon site of nail insertion (77% and 50% respectively).
Toivanen et al. investigated this question when the group randomized fifty patients with a tibial shaft fracture requiring
intramedullary nailing equally to treatment with paratendinous or transtendinous nailing. Fourteen (67%) of the twenty-one
patients treated with transtendinous nailing reported anterior knee pain at the final evaluation. Of these fourteen patients, thirteen
were mildly to severely impaired by the pain. Fifteen (71%) of the twenty-one patients treated with paratendinous nailing reported
anterior knee pain, and ten of the fifteen were impaired by the pain. The Lysholm, Tegner, and Iowa knee scoring systems;
muscle-strength measurements; and functional tests showed no significant differences between the two groups. Compared with a
transpatellar tendon approach, a paratendinous approach for nail insertion does not reduce the prevalence of chronic anterior knee
pain or functional impairment by a clinically relevant amount after intramedullary nailing of a tibial shaft fracture.
93. PREFERRED RESPONSE 3
The injury described above is a Type IIIB injury as per the Gustilo and Anderson classification. Type I injuries are low energy and
have small soft-tissue wounds (usually <1 cm in length) with minimal contamination. Type II injures have a wound >1 cm in
length, but do not have extensive soft-tissue damage, flaps, or avulsions. Type IIIA injuries are associated with soft-tissue damage
secondary to high-energy trauma but have adequate soft-tissue coverage. Type IIIB injures exhibit severe periosteal stripping and
bone exposure, often associated with massive contamination. These often require treatment with soft-tissue coverage procedures.
Type IIIC fractures require vascular repair. The risk for infection in this scenario is as high as 44%, so placing definitive plate
fixation is contraindicated when future debridement and soft tissue coverage procedures will be needed. External fixation provides
excellent stability, provisional skeletal alignment, and minimal additional soft tissue injury.
94. PREFERRED RESPONSE 5
At 9 months, observation is no longer an option, as the fracture is not healing and is adjacent to a arthrofibrotic joint. Plate
osteosynthesis has been shown to be an effective method of treatment for patients who have had an open fracture of the tibia that
has failed to unite after external fixation and/or immobilization in a cast.
Wiss et al reported a series of fifty tibial non-unions with a similar clinical scenario. He reported that, with compression plating,
92% of the nonunions healed without further intervention. In their study, 39/50 patients, had autogenous bone grafting in addition
to compression plating.
95. PREFERRED RESPONSE 4

Reamed nailing of closed tibial shaft fractures has been shown to lead to an earlier time to union without an increased rate of
complications when compared to unreamed nailing.
The referenced study by Finkenmeier et al is a randomized controlled study of reamed vs. unreamed nails in open and closed tibia
fractures (excluding Grades IIIB and IIIC). They found that the use of reamed insertion of IM nails for the treatment of closed
tibia fractures lead to an earlier time to union without increased complications. The authors reported no differences in infection
rate, compartment syndrome rate, or percent needing additional surgeries to obtain union. More secondary procedures were
needed with unreamed nails in closed fractures only. Increased rates of interlocking screw failure were seen if smaller screws were
needed for smaller, unreamed nails.
The referenced study by Keating et al reported that reaming for open tibia fractures is safe, with time to union and nonunion rate
increasing with increased soft tissue injury as classified by the Gustilo-Anderson classification.
Bhandari et al conducted a multicenter, blinded randomized trial of 1319 adults in whom a tibial shaft fracture was treated with
either reamed or unreamed intramedullary nailing. When comparing outcomes in open and closed injuries at twelve months, they
found a benefit for reamed intramedullary nailing in patients with closed fractures, but found no difference between approaches in
patients with open fractures.
Figure A shows a tibial shaft fracture, with intramedullary fixation shown in Figures B and C.
96. PREFERRED RESPONSE 1
By definition, with Type IIIB injuries, there is exposed bone after debridement which will require a local or a free flap for
coverage. Distal third IIIB tibial shaft fracture are unique in that they usually require a free flap or reverse sural rotational flap to
obtain adequate coverage. As stated in Skeletal Trauma, "As local donor muscles in the distal third of the tibia are almost nonexistent, closure of an open plafond fracture, or any extensive Type IIIB injury in this area will usually require free tissue transfer.
The primary options are latissimus dorsi or rectus abdominus for large defects, and gracilis for smaller wounds." In addition to the
flaps mentioned here, others, including fasciocutaneous flaps and radial forearm flaps, are also utilized with success in this area.
Typically, treatment of Type IIIB tibial shaft fractures should be staged. Initially tetanus prophylaxis, antibiotics with gram
negative and positive coverage, and application of an external fixator with repeat I&Ds are employed for immediate fracture care.
Plating is usually required in the presence of significant intra-articular fracture involvement.
Incorrect Answers: Typically, proximal third tibia fractures requiring soft tissue coverage can be treated with a gastrocnemius
rotation flap and middle third tibia fractures with soft tissue defects can be reliably covered with a soleus rotation flap. Therefore,
a free flap is less commonly indicated in the proximal and middle tibia.
97. PREFERRED RESPONSE 3
Figures A and B show a significantly comminuted, segmental tibial fracture. In this scenario, the fracture is appropriately
classified as a Grade III because there is a highly comminuted, segmental fracture which is always associated with significant
periosteal stripping. The patient's leg was able to be closed primarily, which means that it should be classified as a IIIA.
Gustilo et al reviews 18 years of open fracture treatment and outcomes. They reported that dbridement and copious irrigation,
with primary closure for type I and II fractures and secondary closure for type III fractures resulted in a 5% infection rate (9% for
type III fractures). Initial wound cultures were positive in 70.3% despite an infection rate of that patient group of only 2.5%.
Incorrect Answers:
Answer 1: Gustilo Type I wounds have a clean skin lesion that are < 1 cm, and a simple fracture with minimal comminution.
Answer 2: Gustilo Type II wounds have a skin lesion > 1 cm, no extensive soft tissue damage, minimal crushing, and moderate
comminution and contamination.
Answer 4: Gustilo Type IIIB are defined as wounds that require a flap for salvage.
Answer 5: Gustilo Type IIIC have an exposed fracture with arterial damage that requires repair.

Você também pode gostar